Está en la página 1de 127

Editorial

NOTA

La medicina es una ciencia sometida a un cambio constante. A medida que la investigación y la


experiencia clínica amplían nuestros conocimientos, son necesarios cambios en los tratamientos
y la farmacoterapia. Los editores de esta obra han contrastado sus resultados con fuentes
consideradas de confianza, en un esfuerzo por proporcionar información completa y general, de
acuerdo con los criterios aceptados en el momento de la publicación. Sin embargo, debido a la
posibilidad de que existan errores humanos o se produzcan cambios en las ciencias médicas, ni
los editores ni cualquier otra fuente implicada en la preparación o la publicación de esta obra
garantizan que la información contenida en la misma sea exacta y completaen todos los aspectos,
ni son responsables de los errores u omisiones ni de los resultados derivados del empleo de dicha
información. Por ello se recomienda a los lectores que contrasten dicha información con otras
fuentes. Por ejemplo, y en particular, se aconseja revisar el prospecto informativo que acompaña
a cada medicamento que deseen administrar, para asegurarse de que la información contenida
en este libro es correcta y de que no se han producido modificaciones en la dosis recomendada
o en las contraindicaciones para la administración. Esta recomendación resulta de particular
importancia en relación con fármacos nuevos o de uso poco frecuente. Los lectores también deben
consultar a su propio laboratorio para conocer los valores normales.

No está permitida la reproducción total o parcial de este libro, su tratamiento informático, la transmisión
de ningún otro formato o por cualquier medio, ya sea electrónico, mecánico, por fotocopia, por registro y
otros medios, sin el permiso previo de los titulares del copyright.

C/ Albarracín, 34; 28037 Madrid


Tfno.: (0034) 91 782 43 30 - Fax: (0034) 91 782 43 43
E-mail: editorial@grupocto.com
Página web: www.grupocto.com
Editorial
Digestivo
Info Pregunta: 190d58b0-56a0-4cae-b155-005ff11b9d2e

1. Paciente con antecedentes de etilismo crónico que ingresa en el hospital por astenia y
anemia intensas. En la exploración física se objetiva hepatoesplenomegalia y en los
estudios sanguíneos, acantocitos e hiperlipemia. Este cuadro es CARACTERÍSTICO
de:

1. Hemólisis por hiperesplenismo.


2. Enfermedad de Wilson.
3. Síndrome de Zieve.
4. Deficiencia de folato.

Resp. Correcta: 3

Comentario:

El síndrome de Zieve (respuesta 3 correcta) es una complicación extremadamente infrecuente de la


esteatosis hepática secundaria al alcohol (hígado graso alcohólico). Se define por la asociación de
hipertrigliceridemia, anemia hemolítica e ictericia en un paciente con esteatosis hepática alcohólica. Los
pacientes suelen tener además dolor abdominal. La plaquetopenia, cirrosis, hiperesplenismo e
hipergammaglobulinemia también pueden manifestarse como parte de este síndrome, aunque no forman
parte de la tríada que lo define.

-----------------------------------------------------------o--------------------------------------------------------------

Info Pregunta: dd8c5d68-cbc8-49e3-8112-009fb1b386d2

2. Joven de 18 años de edad, con antecedentes de asma, alergia a pólenes, ácaros y pelo
de gato, acude a urgencia refiriendo sensación de detención de alimentos a nivel
retroesternal con práctica incapacidad para deglutir su propia saliva. Refiere episodios
similares en otras ocasiones que han cedido de forma espontánea en pocos minutos.
¿Cuál de los siguientes es el diagnóstico más probable?

1. Esófago de Barrett.
2. Anillo esofágico distal (Schaztki).
3. Esofagitis infecciosa.
4. Esofagitis eosinofílica.

Resp. Correcta: 4

Comentario: Se trata de un caso clínico típico de esofagitis eosinofílica, en la que se describen tres
características de esta entidad: joven, asociación con procesos alérgicos (asma, alergia a polen…) e
impactaciones alimentarias de repetición. No hay datos que sugieran que el paciente pudiera tener un
esófago de Barrett, dado que no describen que presente pirosis. Aunque el anillo esófagico distal pudiera
justificar episodios de disfagia esofágica intermitente, no hay asociación del mismo con procesos alérgicos
que describe la pregunta. Por último, no pude ser una esofagitis infecciosa porque el paciente no refiere
odinofagia.

-----------------------------------------------------------o--------------------------------------------------------------
Info Pregunta: 06b55bca-489e-4fa9-b355-00bf55a0d7cb

3. Mujer de 56 años que acude con sensación de dolor urente postprandial que ha ido
empeorando en la ultimas semanas. La exploración física general fue normal. La
paciente se niega a realizarse gastroscopia pero aporta de un centro privado un estudio
de secreción áciga gástrica en la que se comprueba la existencia de hiperclorhidria.
Con la información de la que dispone en este momento, ¿cuál sería el diagnóstico más
probable si hubiere aceptado realizarse gastroscopia?:

1. Gastritis crónica atrófica multifocal.


2. Gastritis crónica activa.
3. Gastritis autoinmune (tipo A).
4. Gastrinoma con síndrome de Zollinger-Ellison.

Resp. Correcta: 2

Comentario:

La Gastritis crónica atrófica multifocal y la corporal difusa (o tipo A), son gastritis que cursan con
hipoclorhidria. La gastritis crónica activa y el gastrinoma cursan con hiperclorhidria. De esta 2, la más
frecuente es la gastritis crónica activa y por ello sería la opción más correcta.

-----------------------------------------------------------o--------------------------------------------------------------

Info Pregunta: e4a60d0d-ab42-4110-b122-00ece905948e

4. Varón de 56 años con consumo excesivo de alcohol, acude a su consulta por


hipertransaminasemia en analítica de rutina. La exploración física únicamente reveló
hepatomegalia leve. La analítica fue: Hb 13.1 g/dl, VCM 103.5 fl, leucocitos 7.600/mm3,
plaquetas 158.000/mm3, actividad de protrombina 10%, glucosa 100 mg/dl, urea 30
mg/dl, creatinina 1 mg/dl, albúmina 4.2 g/dl, GOT 200 U/L, GPT 100 U/L, GGT 228
U/L, bilirrubina total 1.1 mg/dl, fosfatasa alcalina 70 U/L, LDH 136 U/L, sodio 139,
potasio 4.2, antiHBs +, AgHBs -, AntiHBc -, AgHBe -, antiHBe-, DNA-VHB -, anti-
VHC -, AgVHD -. Se realiza una ecografía abdominal en la que se observa un hígado
algo heterogéneo y una lesión nodular de 4 cm en segmento V. Se realiza un TC con
contraste donde se observa dicho nódulo con realce precoz y homogéneo con contraste
y cicatriz estrellada central. ¿Cuál sería la actitud más adecuada en este paciente?:

1. Trasplante hepático.
2. Quimioembolización.
3. Cirugía resectiva.
4. Seguimiento evolutivo del paciente.

Resp. Correcta: 4

Comentario:

Se trata de un paciente con hepatopatía etílica que, independientemente de si fuere o no cirrótico, tiene una
lesión focal hepática pero que no cumple criterios de hepatocarcinoma sino de hiperplasia nodular focal con
su cicatriz estrellada central característica. Esta lesión no requiere tratamiento salvo si experimenta
crecimiento o es sintomática.
-----------------------------------------------------------o--------------------------------------------------------------

Info Pregunta: 4c2f1a68-6e5e-4ee9-a03b-01105e32e7cc

5. Varón de 60 años, cirrótico VHC en tratamiento con espironolactona y furosemida a


dosis máximas, sin otros antecedentes de interés. Acude a consulta por aumento de
perímetro abdominal y dolor en ambos flancos y espalda. En la exploración física se
observó abdomen distendido y oleada ascítica. En la analítica destacaba: Hb 11.0 g/dl,
VCM 80.5 fl, leucocitos 10.000/mm3 con 50% neutrófilos, plaquetas 78.000/mm3,
glucosa 89 mg/dl, urea 101 mg/dl, creatinina 2.5 mg/dl, proteínas totales 5.6 g/dl,
albúmina 3.3 g/dl. GOT 88 U/L, GPT 101 U/L, GGT 178 U/L, bilirrubina total 1.8
mg/dl, fosfatasa alcalina 108 U/L, LDH 187 U/L, sodio 133, potasio 4.8. ¿Cuál sería la
actitud para el control de este paciente?

1. TIPS.
2. Paracentesis evacuadora de repetición.
3. Aumentar la dosis de diuréticos.
4. Tratamiento con Peg-IFN y rivabirina.

Resp. Correcta: 1

Comentario: Se trata de un paciente que acude con ascitis importante no controlada a pesar de diurético a
dosis máximas. Se trata, por tanto, de una ascitis refractaria. Para ello puede plantearse la realización de
paracentesis evacuadoras o TIPS. En este caso nos decantaríamos por ésta última ya que tiene insuficiencia
renal y no se observa que sea un paciente con encefalopatía hepática previa ni un CHILD muy avanzado.

-----------------------------------------------------------o--------------------------------------------------------------

Info Pregunta: da617024-24c8-48b9-9c2c-014510d95708

6. Un hombre de 25 años presenta de forma brusca astenia e ictericia, detectándose


unas transaminasas (ALT y AST) > 2.000 UI/L. Los marcadores serológicos muestran
el siguiente patrón: AntiVHA IgM negativo, HBsAg negativo, antiHBc IgM positivo y
antiVHC negativo. ¿Cuál es su diagnóstico?

1. Hepatitis crónica B.
2. Hepatitis aguda no viral.
3. Hepatitis aguda B.
4. Hepatitis aguda A y B.

Resp. Correcta: 3

Comentario:

Pregunta interesante sobre marcadores serológicos de las hepatitis víricas. En este caso clínico nos presentan
a un paciente con una clínica y analítica típicas de una hepatitis viral (astenia, ictericia y transaminasas muy
elevadas). En cuanto a los marcadores serológicos, que por otra parte es lo realmente importante de la
pregunta, vemos que nos exponen un patrón compatible con una hepatitis aguda B. No te confundas con la
opción 2. En este caso al decirnos que el HBsAg es negativo puede llevar a pensar que no es una infección
por hepatitis B, ya que este marcador indica la presencia del VHB. Sin embargo, fíjate que la determinación
de anticuerpos anti-HBc IgM es positiva, y este marcador es el que resulta imprescindible para definir una
hepatitis B como aguda. Lo que probablemente suceda en el caso clínico de la pregunta es que el paciente se
encuentre en período ventana, es decir, entre la desaparición del HBsAg y la aparición de su anticuerpo,
momento en que el único marcador positivo puede ser la IgM anti- HBc.

-----------------------------------------------------------o--------------------------------------------------------------

Info Pregunta: 672ec7ac-6582-414a-a486-01dacd3167d4

7. Mujer de 30 años de edad, embarazada de 33 semanas, que consulta por prurito de


una semana de evolución. Las pruebas hepáticas muestran un leve incremento de los
valores de transaminasas (AST 77 UI/L y ALT 84 UI/L), así como de fosfatasa alcalina
(155 UI/L) y gammaglutamil-traspeptidasa (125 UI/L), con una cifra de bilirrubina
total de 2,3 mg/dL. El hemograma y el estudio de coagulación son normales, la
ecografía abdominal no muestra alteraciones hepáticas ni biliares y la serología de
virus de hepatitis, así como los autoanticuerpos son negativos. ¿Cuál de las siguientes
actitudes es la CORRECTA?

1. Realizar una biopsia hepática para establecer el diagnóstico de la enfermedad hepática.


2. Tratar a la paciente con ácido ursodesoxicólico, ya que probablemente se trata de una colestasis
intrahepática benigna del embarazo.
3. Interrumpir el embarazo ya que el diagnóstico más probable es el hígado graso agudo del embarazo.
4. Realizar una resonancia magnética para descartar una colangitis esclerosante primaria.

Resp. Correcta: 2

Comentario: Nos presentan un caso clínico de una mujer embarazada que en el tercer trimestre presenta un
cuadro con hipertransaminasemia leve y colestasis. Las dudas más razonables sería dudar en si se trata de
una colestasis intrahepática benigna del embarazo o un hígado graso agudo del embarazo. Una de las claves
fundamentales es la presencia de prurito muy típica y frecuente en la colestasis intrahepática benigna del
embarazo y mucho más rara en la segunda entidad. A esto se le suman los datos de transaminasas
ligeramente elevadas y, sobre todo, predominio de colestasis (aumento de bilirrubina, fosfatasa alcalina y
bilirrubina); ambos apoyan igualmente la primera entidad. La biopsia hepática en esta situación aporta poco
y tampoco es el caso típico de una colangitis esclerosante. Por todo ello, la opción correcta sería la segunda.
El tratamiento con ácido ursodesoxicólico, si bien no está ensayado en el embarazo, se puede utilizar,
habiendo múltiples casos descritos en la literatura que muestran su utilidad

-----------------------------------------------------------o--------------------------------------------------------------

Info Pregunta: 1181024f-6019-49fc-a3e6-020dd3467f26

8. La hepatitis A es una de las hepatitis más frecuentes en nuestro medio y debe


conocerse en todo medico que se precie y trabaje en el sistema de salud. Respecto a esta
entidad señale la afirmación que considere incorrecta:

1. En la infección aguda los anticuerpos de tipo IgM suelen ser detectables durante unos 4 meses.
2. La infección aguda sobre una hepatopatía crónica virus B de base puede precipitar un fallo hepático.
3. La mayor parte de las veces cursa en la infancia como una hepatitis colestásica.
4. La determinación de RNA-VHA es muy poco utilizado en la práctica diaria.

Resp. Correcta: 3

Comentario:
La hepatitis aguda por VHA en la infancia cursa en la mayor parte de las ocasiones de forma asintomática.
La determinación del RNA es muy poco usado dada la alta utilidad de los anticuerpos de tipo IgM e IgG
para precisar el diagnóstico.

-----------------------------------------------------------o--------------------------------------------------------------

Info Pregunta: 08062102-e17a-4417-a879-02182c64696f

9. ¿Cuál de estas opciones es FALSA respecto al pseudoquiste pancreático?:

1. La mayoría de los pseudoquistes se localizan en cabeza de páncreas.


2. Debe sospecharse su presencia ante la persistencia o aumento de la hiperamilasemia.
3. La rotura y la hemorragia son las principales causas de mortalidad.
4. La ecografía y la TC son los métodos diagnósticos de elección.

Resp. Correcta: 1

Comentario:

Los pseudoquistes pancreáticos se localizan con mayor frecuencia en el cuerpo y cola del páncreas variando
su frecuencia en estas localizaciones entre el 55 % y el 85 % de todos los pseudoquistes según diferentes
series. Deben sospecharse por la persistencia del dolor abdominal con intolerancia a la dieta oral y la
persistencia o aumento de las cifras de amilasa. El método diagnóstico de elección es el TAC Abdominal
siendo también útil, sobre todo en el seguimiento la ECO Abdominal. En general el tratamiento es
conservador, inicialmente porque con el tiempo una gran parte de ellos desaparecen o disminuyen de tamaño
y posteriormente porque sólo requerirán una actitud quirúrgica aquellos que por su tamaño o complicación
den clínica al paciente.

-----------------------------------------------------------o--------------------------------------------------------------

Info Pregunta: e822e060-bad2-4192-ac5e-024825c63200

10. Mujer de 76 años que acude a su consulta por dificultad para la deglución; refiere
que esto le viene pasando desde hace años, pero que en los últimos meses ha
empeorado, por lo que solo tolera alimentación líquida. No presenta fiebre ni pérdida
de peso. Mantiene tratamiento de forma habitual con furosemida y digoxina ("por algo
de corazón") y ranitidina (para sus ardores). ¿Cuál es el diagnóstico MÁS probable?

1. Acalasia.
2. Espasmo esofágico difuso.
3. Estenosis péptica.
4. Neoplasia esofágica.

Resp. Correcta: 3

Comentario:

Ante una disfagia progresiva en un anciano, el cáncer sería una opción a tener en cuenta. Sin embargo, la
larga duración (años) y la ausencia de pérdida de peso hacen más probables otros diagnósticos. La acalasia
también puede producir un cuadro similar, pero entonces no tomaría ranitidina “para sus ardores”, ya que los
pacientes con acalasia no tienen por qué tenerlos. Los “ardores” (pirosis) nos hacen pensar en reflujo
gastroesofágico, que posiblemente se habrá complicado con una estenosis péptica (respuesta 3 correcta). La
opción del espasmo esofágico difuso no es muy adecuada; ten en cuenta que, en esa enfermedad, la queja
fundamental es el DOLOR, no la disfagia, y tampoco tendría relación con la pirosis.

-----------------------------------------------------------o--------------------------------------------------------------

Info Pregunta: 10c81dd5-c099-4047-97f9-02858ef9b1f8

11. En relación con los pólipos del colon, indique la afirmación que le parece
VERDADERA:

1. La mayoría de los cánceres colorrectales derivan de un pólipo adenomatoso.


2. El síndrome de Muir Torre asocia los pólipos del colon a neoplasias del sistema nervioso central.
3. La polipectomía con colonoscopia no es tratamiento suficiente de un pólipo con carcinoma in situ (no
sobrepasa la membrana basal glandular).
4. Los pólipos adenomatosos tubulares son los que más tienden a malignizarse.

Resp. Correcta: 1

Comentario:

La opción correcta es la 1, ya que la mayoría de los cánceres colorrectales provienen de un pólipo previo
(90% de los casos). Repasemos las demás opciones de respuesta:
2.- El síndrome de Muir Torre es una variante del síndrome de Lynch con tumores cutáneos.
3.- En caso de carcinoma in situ, como el tumor no sobrepasa la membrana basal, la polipectomía
endoscópica es curativa, ya que, al no haber llegado a la submucosa, las células tumorales no diseminan a
través de los linfáticos situados en esta capa.
4.- La probabilidad de malignización de un pólipo es < 1%, siendo los que más riesgo tienen los
adenomatosos con componente velloso.

-----------------------------------------------------------o--------------------------------------------------------------

Info Pregunta: 23649c23-cac9-4c26-aa8c-0329ef3d3917

12. Paciente que acude por hipertransaminasemia en el que se objetiva en sangre la


siguiente serología: Ac HBc IgG+, Ag HBs +, AgHBe- DNA +. Cuál es el diagnóstico?

1. Hepatitis aguda
2. Hepatitis B crónica AgHBe positivo
3. Mutación precore
4. Portado inactivo

Resp. Correcta: 3

Comentario: La cepa mutante precore se caracteriza porque los 2 marcadores habituales de realización no
coinciden y, por tanto, el AgHBe es negativo y el DNA-VHB es positivo.

-----------------------------------------------------------o--------------------------------------------------------------

Info Pregunta: a9de57cc-afd8-4dcf-b96e-0346feabe315

13. Varón de 35 años con antecedentes de dispepsia, motivo por el que ha sido
estudiado recientemente, que acude a Urgencias con un cuadro de fiebre de 39 ºC,
leucocitosis, dificultad respiratoria y dolor torácico. Presenta en la Rx de tórax una
imagen compatible con neumomediastino. Con respecto al cuadro que usted sospecha
que tiene el paciente, ¿cuál considera su origen MÁS probable?

1. Vómitos.
2. Traumatismo.
3. Ingesta de algún tóxico químico.
4. Enfermedad iatrogénica.

Resp. Correcta: 4

Comentario:

La dificultad de esta pregunta radica en que debemos identificar la patología que presenta el paciente para
poder resolver la pregunta que se nos hace sobre su tratamiento. Nos hablan de un estudio reciente por
dispepsia y una clínica aguda de dolor torácico y signos de sepsis acompañados de una imagen característica
de neumomediastino. Todo ello es compatible con una perforación esofágica. Pese a que la rotura esofágica
por vómitos, síndrome de Boerhaave, es la causa más grave de perforación, la etiología más frecuente de
una perforación esofágica es la iatrogénica (respuesta 4 correcta), siendo la endoscopia la causa del 59% de
las perforaciones esofágicas. Probablemente durante el estudio reciente por dispepsia se produjo una
pequeña perforación al hacer la endoscopia, que ha pasado desapercibida hasta que se ha desarrollado una
mediastinitis.

-----------------------------------------------------------o--------------------------------------------------------------

Info Pregunta: 0e1f7ec0-354b-4e2c-b592-04cd4203c522

14. Con respecto a la prueba de imagen consistente en la realización de un


esofagograma con bario, indique qué patología de las siguientes considera que tiene
MENOR sensibilidad que la endoscopia para su diagnóstico:

1. Hernia de Hiato.
2. Alteraciones motoras.
3. Divertículos esofágicos.
4. Esofagitis.

Resp. Correcta: 4

Comentario:

Pese a que la endoscopia digestiva alta aclara el origen de muchos trastornos esofágicos, fundamentalmente
malignos, el estudio de imagen mediante papilla de bario tiene más sensibilidad que la endoscopia para el
diagnóstico de la hernia de hiato como alteración anatómica, así como para mostrar estenosis ligeras,
alteraciones motoras y divertículos esofágicos. En el caso de la esofagitis, el diagnóstico se lleva a cabo
mediante endoscopia (el esofagograma de bario, además de no indicado para esta patología, tiene menor
sensibilidad en el diagnóstico de esta que la endoscopia - respuesta 4 correcta -).

-----------------------------------------------------------o--------------------------------------------------------------

Info Pregunta: 1c746487-8782-44c3-9205-04f064447125

15. Una de las siguientes sustancias NO produce o NO agrava el reflujo


gastroesofágico:

1. Nifedipino.
2. Sildenafilo.
3. Levosulpiride.
4. Nicotina.

Resp. Correcta: 3

Comentario:

Los calcioantagonistas y nitritos, o los fármacos con un comportamiento similar al de los nitritos, como es el
caso del sildenafilo, relajan la musculatura del cuerpo esofágico, alivian el dolor de las contracturas o
espasmos, y relajan el esfínter esofágico inferior, por lo que aumenta el reflujo. Las dietas ricas en grasas,
chocolate, café, o el uso de nicotina también relajan el esfínter. El Levosulpiride es un potente procinético,
que estimula la contractilidad del cuerpo esofágico y contrae el esfínter inferior. Por ello, no agravaría el
reflujo, sino todo lo contrario: podría emplearse como tratamiento médico.

-----------------------------------------------------------o--------------------------------------------------------------

Info Pregunta: d141684a-55c6-4f96-8241-04fc34249a14

16. Paciente con pancreatitis aguda. En Tomografía Axial Computerizada (TAC)


realizada a las 72 horas de ingreso se aprecia una necrosis del 50% del páncreas. A la
tercera semana de ingreso, el paciente comienza con fiebre elevada y leucocitosis. Se
solicita radiografía de tórax urgente y sedimento urinario, siendo ambos normales.
Señale el siguiente paso a seguir:

1. Punción aspiración con aguja fina de necrosis pancreática guiada por ecografía o TAC.
2. Aspirado broncoalveolar, cultivo de orina y hemocultivos.
3. Resonancia Magnética Nuclear abdominal.
4. Colecistectomía urgente.

Resp. Correcta: 1

Comentario:

Paciente con necrosis pancreática y cuadro infeccioso asociado. El riesgo de infección de la necrosis
aumenta en la primera semana teniendo su pico máximo en la tercera semana de evolución. Sospecha de
infección de dicha necrosis. Es necesario confirmar la infección de la misma con una PAAF del material
necrótico y cultivo microbiológico. Dicha PAAF clásicamente se ha realizado con control de TC o
ecografía, no obstante, tras la aparición de la ecoendoscopia, esta última se considera el gold stándar dada su
mayor rentabilidad y menor morbimortalidad.

-----------------------------------------------------------o--------------------------------------------------------------

Info Pregunta: 4841605e-fbc8-4299-870a-057db9427f0e

17. Catalina es una paciente de 55 años que acaba de ser diagnosticada de enfermedad
de reflujo gastroesofágico por la clínica de pirosis y va a comenzar un tratamiento
empírico con omeprazol de 20 mg al día. Para el diagnóstico de ERGE, ¿en cuál de las
siguientes situaciones NO sería necesario realizar una medición del pH de 24 horas
para confirmarlo?

1. Diagnosticar la ERGE en presencia de tos crónica sin otros síntomas.


2. Antes de una cirugía correctora (funduplicatura).
3. Confirmar el diagnóstico de ERGE en paciente con pirosis antes de comenzar el tratamiento con IBP.
4. Descartar la ERGE en paciente con erosión del esmalte.

Resp. Correcta: 3

Comentario:

La realización de una pHmetría 24 horas es la prueba más exacta y objetiva para el diagnóstico de RGE, sin
embargo solo se indica en situaciones como:

- Pacientes con síntomas atípicos para determinar si se relacionan con el RGE. Recuerda la regla
mnemotécnica ¡vaya TELA con los otorrinos! Tos-Erosión del esmalte-Laringitis-Asma. Puede debutar sin
síntomas digestivos y solo presentar alguno de estos.

- Pacientes con ausencia de respuesta al tratamiento.

- Pacientes para valorar la eficacia del tratamiento, sobre todo en pacientes con esófago de Barret en los que
son necesarias ajustar las dosis.

- Valoración pre y postquirúrgica de una cirugía antirreflujo.

Un paciente con pirosis el diagnóstico es clínico y está confirmado, no hay que hacer pHmetría (marcamos
la opción de respuesta 3).

-----------------------------------------------------------o--------------------------------------------------------------

Info Pregunta: 11de815d-5150-4d24-8693-05e42fff30be

18. Un paciente diagnosticado de enfermedad celíaca no mejora clínicamente tras 5


meses de habérsele indicado una dieta carente de gluten. Los estudios analíticos
muestran persistencia de títulos altos de anticuerpos antiendomisio tipo IgA. ¿Cuál es
la causa más probable de esa falta de respuesta a la dieta sin gluten?:

1. Que no sigue correctamente la dieta.


2. Que no es una enfermedad celíaca lo que padece ese enfermo.
3. Que la enfermedad celíaca se asocia con una inmunodeficiencia común variable.
4. Que ha desarrollado un linfoma.

Resp. Correcta: 1

Comentario: La primera causa de remisión incompleta o persistencia de los síntomas y títulos elevados de
anticuerpos es el incumplimiento dietético. Sólo excluyendo esta causa cabría plantearse, en segundo lugar,
un linfoma.

-----------------------------------------------------------o--------------------------------------------------------------

Info Pregunta: 2c242524-5bb9-4489-ad7e-060978b544a6

19. Un hombre de 67 años con cirrosis alcohólica en seguimiento por el servicio de


hepatología es diagnosticado de un hepatocarcinoma de 3 cm de diámetro localizado en
el lóbulo izquierdo, segmento 3. Sometido a estudio para valoración del tratamiento a
seguir se demuestra que el paciente no tiene hipertensión portal, presenta una función
hepática normal, un MELD score de 7 y un Child-Pugh grado A. La alfa-fetoproteína
es inferior a 50 nf/mL y no se han objetivado metástasis a distancia. ¿Cuál es el
tratamiento MÁS indicado en este paciente?

1. Aplicar inyección de etanol percutánea.


2. Resección hepática.
3. Trasplante hepático.
4. Quimioembolización transarterial.

Resp. Correcta: 2

Comentario: Decidimos el tratamiento del carcinoma hepatocelular según el Sistema de estadificación


Barcelona-Clinic-Liver-Cancer (BCLC) que toma en cuenta tres parámetros (estadiaje de tumor, función
hepática y presencia de síntomas) de acuerdo a 5 estadios. Los mejores candidatos a resección quirúrgica
son aquellos con tumores únicos, que presentan una bilirrubina normal, ausencia de hipertensión portal
clínicamente relevante (no ascitis, no várices esofágicas y/o gástricas) con Child Pugh A o B. Recuerda que
en tumores únicos está indicada la resección quirúrgica independientemente del tamaño de la lesión si se
cumplen los criterios antes mencionados. La pregunta nos comenta a un paciente en estadio inicial (A) de la
BCLC(tumor único, sin hipertensión portal, función hepática conservada definido por Child A, sin
metástasis a distancia) con todos los criterios para indicar la cirugía

-----------------------------------------------------------o--------------------------------------------------------------

Info Pregunta: 4737ea1b-0b51-4854-8fa1-060f466b5bb8

20. Mujer de 35 años, diagnosticada de carcinoma de endometrio en estadio avanzado


por el que recibe quimioterapia, que acude a Urgencias por presentar desde hace una
semana odinofagia. Una radiografía de tórax resultó normal. Se le realizó una
endoscopia donde, desde el tercio medio del esófago, la mucosa presentaba importante
afectación con vesículas y ulceraciones, siendo informada como esofagitis necrotizante.
¿Cuál es su impresión diagnóstica?

1. Esofagitis por Candida.


2. Esofagitis por virus varicela zoster.
3. Esofagitis por CMV.
4. Esofagitis por VHS.

Resp. Correcta: 2

Comentario:

Pregunta de considerable dificultad. La causa más frecuente de esofagitis infecciosa es la candidiásica, pero
en este caso no existen hallazgos sugerentes (no hay lesiones blanquecinas en la mucosa esofágica). La
presencia de vesículas y ulceraciones orienta a etiología vírica. Estas esofagitis son más frecuentes en
pacientes inmunocomprometidos, como es el caso de la paciente por la quimioterapia. El CMV no es una
buena opción de respuesta, puesto que suele producir una úlcera única y de gran tamaño, y aquí son
múltiples. Solo nos queda el VHS y el virus varicela zoster, siendo los dos capaces de producir vesículas y
ulceraciones. La razón para decantarse por el VVZ es el carácter NECROTIZANTE de la esofagitis, que no
sería típica por el virus (respuesta 2 correcta).

-----------------------------------------------------------o--------------------------------------------------------------

Info Pregunta: 9b732bba-f3a2-4747-bcaa-060f6a5561cb


21. Paciente de 47 años, en diálisis desde los 38 por insuficiencia renal terminal
secundaria a poliquistosis renal del adulto, con infección crónica por el VHC genotipo
3 y carga viral de 4 millones UI/mL. ¿Cuál sería el tratamiento más adecuado?

1. Daclatasvir + Ribavirina.
2. Glecaprevir + Pribrentasvir.
3. Sofosbuvir + Velpatasvir.
4. Sofosbuvir + Ledispasvir.

Resp. Correcta: 2

Comentario: En un paciente con infección crónica por el VHC genotipo 3 serían válidas tanto la opción 2
como la 3. Pero al tratarse de un paciente con insuficiencia renal severa no podemos usar el sofosbuvir por
lo tanto, la única opción correcta sería la 2. La combinación elbasvir+grazoprevir se puede usar para los
genotipos 1 y 4 pero no para el 3; la combinación daclatasvir + ribavirina no se usa.

-----------------------------------------------------------o--------------------------------------------------------------

Info Pregunta: 0ee34a12-26c2-4b14-98b5-06c191acef8e

22. Acude a consulta un hombre de 70 años refiriendo que hace una hora se ha
pinchado de forma accidental en el parque con una aguja conectada a una jeringa
abandonada con restos de sangre. El paciente no ha padecido hepatitis B ni ha recibido
vacunación. ¿Cuál es la actitud MÁS correcta?

1. Administrar una dosis de gammaglobulina frente a VHB y una dosis de vacuna.


2. Administrar una dosis de gammaglobulina frente a VHB y vacunación completa.
3. Administrar una dosis de gammaglobulina frente a VHB y realizar antisepsia de la zona.
4. Realizar antisepsia de la zona y vacunación completa.

Resp. Correcta: 2

Comentario: Ante una exposición del VHB en un individuo no vacunado que puede ocurrir de forma
accidental, por pinchazo con material contaminado con antígeno de superficie del VHB (HBsAg), por vía
sexual, tras el contacto sexual con una persona positiva al HBsAg o perinatal, en recién nacidos cuyas
madres son portadoras del VHB se debe realizar inmunoprofilaxis tanto activa como pasiva. La
inmunoprofilaxis pasiva se realiza con la administración intramuscular de inmunoglobulina específica anti-
VHB y la activa con la vacunación completa con vacunas recombinantes que se administran en 3 dosis a los
0 (momento de la exposición), 1 y 6 meses por vía intramuscular en el deltoides. Tras ésta, se detectan
niveles protectores de anticuerpos (anti-HBs > 10 mUI/ml) en más del 90% de los adultos y del 95% de
niños y adolescentes.

-----------------------------------------------------------o--------------------------------------------------------------

Info Pregunta: aff2d38c-a1b1-4940-86de-06d3fa27761f

23. Paciente colecistectomizada hace 6 años por colelitiasis. Desde hace 6 meses
presenta dolor cólico en flanco derecho a temporadas. En la analítica hay valores de
colestasis moderada. La ecografía abdominal sugiere coledocolitiasis. ¿Cuál, entre las
siguientes, es la prueba indicada para realizar en primer lugar, en este caso?:

1. Tomografía computerizada abdominal con contraste i.v.


2. Colangiografía retrógrada endoscópica.
3. Colangioresonancia magnética.
4. Colangiografía isotópica.

Resp. Correcta: 2

Comentario:

Ante un cuadro de cólico biliar e imagen sugestiva de coledocolitiasis en un paciente colecistectomizado se


debe pensar siempre en coledocolitiasis residual. La Ecografía confirma el diagnostico. La prueba de
elección para la extracción de las litiasis es la colangiografía retrógrada endoscópica (CPRE).

-----------------------------------------------------------o--------------------------------------------------------------

Info Pregunta: 53db7243-f795-4581-92c1-075c6d25c539

24. Un paciente de 45 años de edad es diagnosticado de esófago de Barrett largo


(segmento de 6 cm) y sigue tratamiento con inhibidores de la bomba de protones (IBP)
en dosis estándar. ¿Cuál de las siguientes afirmaciones es cierta?:

1. Mientras se mantenga el tratamiento continuo con IBP, se elimina el riesgo de cáncer.


2. Se debe asegurar un control adecuado de la secreción ácida con dosis altas de IBP o, preferiblemente,
ajustando la dosis mediante pHmetría.
3. El seguimiento sólo es necesario si existe displasia de alto grado.
4. La inhibición adecuada de la secreción ácida logra la regresión del epitelio metaplásico.

Resp. Correcta: 2

Comentario: Paciente con Barrett en tratamiento con IBP. Debemos asegurar la eficacia de los mismos ya q
si no el paciente sería candidato a cirugía anti reflujo

-----------------------------------------------------------o--------------------------------------------------------------

Info Pregunta: 0d8596c5-6372-48bc-a660-0761939e9c8a

25. ¿Qué proteína es codificada por la región preS2 + S del genoma del VHB?

1. HBcAg.
2. HBeAg.
3. Proteína mediana.
4. Proteína grande.

Resp. Correcta: 3

Comentario:

Las proteínas de VHB se codifican por:


- HBcAg: gen C (si la transcripción se inicia a nivel de la región Core).
- HBeAg: gen C (si la transcripción se inicia en la región precore).
- Proteína principal: región S.
- Proteína mediana: preS2 y región S (respuesta 3 correcta).
- Proteína grande: preS1, preS2 y región S.

-----------------------------------------------------------o--------------------------------------------------------------
Info Pregunta: f9c835c9-4a4d-4187-bccf-076d94cb19cf

26. Señale la afirmación INCORRECTA en relación con el estudio del paciente con
colelitiasis:

1. La primera técnica de imagen que debemos realizar es la radiografía simple de abdomen.


2. La colangiorresonancia presenta una sensibilidad y una especificidad similares a la CPRE.
3. La ecografía endoscópica es altamente rentable para estudiar las zonas distales del colédoco.
4. La CPRE puede producir, como efecto secundario, un aumento leve y transitorio de la amilasa.

Resp. Correcta: 1

Comentario:

La primera técnica de imagen que debemos realizar en el estudio del paciente con colestasis es una ecografía
para valorar si existe, o no, dilatación de la vía biliar y la presencia, o no, de colelitiasis (opción 1 incorrecta,
por lo que la marcamos).

-----------------------------------------------------------o--------------------------------------------------------------

Info Pregunta: 47330ed8-25b0-4892-ad4b-08da8e70bdaf

27. Una mujer de 49 años acude de Urgencias por presentar tiritona, fiebre de 39 ºC,
dolor en hipocondrio derecho, ictericia y vómitos. La exploración revela TA 100/50
mmHg, FC 110 lpm. Postración y dolor a la palpación en cuadrante derecho, con
Murphy positivo. La analítica muestra leucocitosis con desviación izquierda y la
ecografía abdominal, colelitiasis, coledocolitiasis y dilatación de la vía biliar
extrahepática. Se inicia tratamiento empírico con antibiótico y fluidoterapia. ¿Cuál es
el procedimiento MÁS eficaz que podemos realizar a continuación?

1. Mantener tratamiento antibiótico y medidas de soporte con colecistectomía reglada posterior.


2. Colecistectomía laparoscópica.
3. Colecistectomía urgente con canulación del colédoco.
4. Colangiopancreatografía retrógrada endoscópica con esfinterotomía.

Resp. Correcta: 4

Comentario:

Esta paciente cumple los tres criterios clínicos de la Triada de Charcot para el diagnóstico de colangitis
aguda: fiebre, dolor abdominal en hipocondrio derecho e ictericia. Además, presenta también datos de
síndrome de respuesta inflamatoria sistémica por la existencia de fiebre, taquicardia y leucocitosis,
presumiblemente de causa infecciosa; es decir, estamos ante un caso clínico de sepsis biliar secundaria a
coledocolitiasis demostrada por ecografía abdominal. Para el manejo es necesario fluidoterapia,
antibioterapia empírica y desobstrucción de vía biliar mediante colangiopancreatografía retrógrada
endoscópica con esfinterotomía para extracción de la litiasis, que constituye la alternativa terapéutica más
eficaz y de elección ante una obstrucción biliar.

-----------------------------------------------------------o--------------------------------------------------------------

Info Pregunta: b897c8ef-c045-4f9e-9252-09504cdb090c


28. La manifestación clínica más frecuente en la hepatitis fulminante es:

1. Coagulación intravascular diseminada.


2. Encefalopatía hepática.
3. Hipotensión arterial.
4. Ascitis.

Resp. Correcta: 2

Comentario: La manifestación clínica más frecuente de la hepatitis fulminante y síntoma "princeps" a partir
del cual deberemos sospecharlo es la encefalopatía hepática. Otras manifestaciones que pueden orientarnos a
este proceso es el deterioro de la coagulación y la hipoglucemia pero son menos frecuentes que el anterior.

-----------------------------------------------------------o--------------------------------------------------------------

Info Pregunta: be4b70a5-220f-4a4d-80a7-0965ec50c66f

29. ¿Cuál de las siguientes manifestaciones NO es característica de una colestasis


prolongada?:

1. Hipercolesterolemia.
2. Osteopenia.
3. Degeneración neuromuscular.
4. Estreñimiento.

Resp. Correcta: 4

Comentario: En el síndrome colestático, parte de la sintomatología se debe al déficit de vitaminas


liposolubles. Así, puede surgir osteopenia u osteomalacia por alteraciones en la absorción proteica y de la
vitamina D; y alteraciones neuromusculares o ataxia si hay déficit de vitamina E, situación de baja
frecuencia y que se observa básicamente en niños. Por otra parte, hay síntomas derivados de la retención de
bilis como ictericia, hipercolesterolemia, prurito y en ocasiones puede encontrarse diarrea tipo esteatorrea
por falta de absorción de las grasas al no aportarse sales biliares. No se ha descrito estreñimiento.

-----------------------------------------------------------o--------------------------------------------------------------

Info Pregunta: bd6d6709-b19d-48d6-8bdd-096d6a3b1b6e

30. La vitamina B12 es producida por microorganismos que viven en simbiosis en las
raíces de las plantas. Las concentraciones de vitamina B12 que están presentes en los
tejidos animales son demasiado bajas para su uso en la producción comercial. La
síntesis química tampoco es práctica, ya que requiere 70 etapas de reacción. La
producción comercial se lleva a cabo en la actualidad enteramente por fermentación.
La producción industrial de vitamina B12 está destinada a la elaboración de
suplementos y para enriquecer alimentos. Existen diferentes cantidades de vitamina
B12 de manera natural en huevo, aves, carnes, marisco y lácteos. La vitamina B12 se
absorbe:

1. En el estómago, ayudado por el factor intrínseco.


2. En el duodeno cuando el pH es ácido.
3. En las últimas porciones del íleon.
4. En el ciego.
Resp. Correcta: 3

Comentario:

La vitamina B12, se encuentra en las proteínas animales, de donde es liberada por acción del ácido
clorhídrico y la pepsina. La cobalamina se une inicialmente a una proteína ligadora de cobalamina (R-
binder o cobalofilina), también secretada por el estómago. El complejo cobalofilina-cobalamina es
degradado por las proteasas pancreáticas dentro de la luz duodenal con liberación de la cobalamina, que se
une al FI gástrico. Este complejo alcanza los últimos 60 cm del íleon, donde se une a un receptor específico
que absorbe la vitamina B12.

-----------------------------------------------------------o--------------------------------------------------------------

Info Pregunta: 24cdeecb-ff52-4119-90c6-09ca8c483aea

31. ¿Cuál de las siguientes sustancias NO disminuye la presión del esfínter esofágico
inferior?:

1. Atenolol.
2. Diacepam.
3. Teofilina.
4. Barbitúricos.

Resp. Correcta: 1

Comentario:

El efecto del atenolol es justamente el contrario al que nos indican, es decir, lo que hace es incrementar la
presión del esfínter. El resto de las opciones son conocidos estímulos que inducen su relajación y, por lo
tanto, facilitan el reflujo gastroesofágico.

-----------------------------------------------------------o--------------------------------------------------------------

Info Pregunta: 8459b56d-6343-4866-a159-09cfba6cee42

32. Comparados con la cimetidina, la ranitidina y famotidina en el tratamiento de la


úlcera péptica:

1. Producen efectos anticolinérgicos.


2. Neutralizan el ácido gástrico.
3. Producen menos efectos secundarios.
4. Eliminan la necesidad de prescribir antiácidos.

Resp. Correcta: 3

Comentario:

Los tres fármacos mencionados son antagonistas del receptor histamínico H2, al que bloquean, con lo que
disminuyen la secreción ácida gástrica. La diferencia entre ellos no es la potencia, sino el perfil de efectos
secundarios, que son menos frecuentes con la ranitidina y la famotidina. Recuerda que la cimetidina puede
producir ginecomastia, aparte de arritmias cardíacas, por lo que hoy prácticamente ya no se emplea.

-----------------------------------------------------------o--------------------------------------------------------------
Info Pregunta: 91b1d280-9a75-4337-b10e-0a14a97e29f3

33. Mujer de 26 años con disfagia a líquidos y solidos desde hace un año. Ha
presentado varios episodios de sensación de impactación esofágica y pérdida de 15 kg
de peso. Asimismo refiere levantarse por la mañana con halitosis importante. La
gastroscopia demostró la existencia de retención alimentaria esofágica que impedía
continuar. En relación con la entidad que debería sospechar, señale la afirmación que
considere INCORRECTA:

1. La causa más frecuente en nuestro medio es la esclerosis múltiple.


2. El hallazgo de presurización panesofágica condiciona buena respuesta al tratamiento.
3. Contracciones prematuras de gran amplitud en el 50% de las degluciones condicionan peor respuesta
al tratamiento.
4. La presión del esfínter esofágico inferior puede ser normal.

Resp. Correcta: 1

Comentario:

La causa más frecuente en nuestro medio es la primaria y el adenocarcinoma gástrico. La Acalasia de tipo II
caracterizada por ausencia de peristaltismo con presurización panesofágica es el subtipo que mejor responde
a cualquier tipo de tratamiento), mientras que la espástica (con contracciones prematuras en más del 20% de
las degluciones líquidas estudiadas, es el subtipo que peor responde a cualquier modalidad de tratamiento.
En la Achalasia el esfínter esofágico inferior puede ser hiperpresivo o normopresivo.

-----------------------------------------------------------o--------------------------------------------------------------

Info Pregunta: dfe465f9-7c58-4dc8-84da-0a5010820d3b

34. Paciente con colitis ulcerosa desde hace 11 años que es sometido a colonoscopia de
despistaje rutinaria, con toma de múltiples biopsias. El informe de anatomía patológica
revela displasia de alto grado en una de las biopsias, confirmada por un segundo
patólogo. Interrogando al paciente, este revela que se encuentra “fenomenal”, sin
clínica digestiva ni alteraciones analíticas. Actualmente está en tratamiento de
mantenimiento con mesalazina y azatioprina. ¿Cuál será la actitud correcta ante
nuestro paciente?

1. Aumentar la periodicidad del despistaje endoscópico, repitiendo la exploración cada 3 meses y actuar
según hallazgos.
2. Resección quirúrgica limitada al segmento colónico cuya biopsia resultó positiva para displasia de alto
grado.
3. Valorar la adición al tratamiento de mantenimiento de infliximab y realizar nueva colonoscopia de
control en 6 meses.
4. Valoración quirúrgica para realización de panproctocolectomía.

Resp. Correcta: 4

Comentario:

Todo paciente con colitis ulcerosa en cuyo despistaje de cáncer colorrectal se halle displasia de alto grado y
esta sea confirmada por un segundo patólogo con experiencia, debe ser sometido a panproctocolectomía
precoz.
-----------------------------------------------------------o--------------------------------------------------------------

Info Pregunta: 8d17af27-2a28-4231-a25a-0acf03977fa9

35. ¿Cuál es el mecanismo fundamental por el que los síndromes de hipercrecimiento


bacteriano causan malabsorción intestinal?

1. Efecto tóxico de las toxinas bacterianas sobre la mucosa.


2. Inhibición de las disacaridasas de la mucosa.
3. Hidrólisis de las sales biliares en el intestino.
4. Consumo bacteriano de las enzimas proteolíticas.

Resp. Correcta: 3

Comentario:

Lo que se produce fundamentalmente en el sobrecrecimiento bacteriano es consumo de sustancias nutrientes


(como vitaminas y azúcares) por parte de los microorganismos. Además, los productos metabolizados por
las bacterias producen una desconjugación de las sales biliares. Esto provoca que las grasas ingeridas con la
dieta no puedan ser absorbidas, dando lugar a malabsorción intestinal con esteatorrea.

-----------------------------------------------------------o--------------------------------------------------------------

Info Pregunta: 2d9a6b90-3830-47c5-b70c-0add52f9bbe4

36. ¿Qué papel tiene la CPRE en el diagnóstico de los pseudoquistes?:

1. Es la primera prueba a realizar ante la sospecha de pseudoquiste.


2. Debe realizársele a la mayoría de los pacientes después de un TC.
3. Se debe realizar en la mayoría de los pseudoquistes que no se resuelven en dos semanas.
4. Es conveniente llevarla a cabo inmediatamente antes de la cirugía de un pseudoquiste.

Resp. Correcta: 4

Comentario: La utilidad de la CPRE en el manejo clínico de los pseudoquistes de páncreas viene dada por
ser una técnica de imagen que nos va a permitir demostrar o no la comunicación entre el pseudoquiste y los
conductos pancreáticos lo cual es útil en el momento de decidir entre las opciones de tratamiento del
pseudoquiste. Así pues en los casos en los que se demuestre la existencia de una comunicación entre el
pseudoquiste y los conductos pancreáticos la opción de realizar una punción drenaje debe ser evitada pues el
riesgo de originar una fístula pancreática después de la resolución del pseudoquiste es muy alta así como el
de que este recurra. Sólo en los casos en los que no exista comunicación podremos optar por un drenaje
percutáneo como opción de tratamiento del pseudoquiste con una garantía relativa de no crear fístula
pancreática..

-----------------------------------------------------------o--------------------------------------------------------------

Info Pregunta: b1266fe7-235e-487c-85be-0aeafd3ef10c

37. Varón de 33 años que refiere sensación de presión en región hipofaríngea y tos de
predominio matutino. Tras consultar con servicio de ORL y descartarse patología a ese
nivel, se remite para continuar el estudio puesto que refiere desde hace meses
regurgitaciones de comida postprandiales ocasionales y pirosis. Se realizó una
gastroscopia en la que se observa únicamente una pequeña hernia de hiato. ¿Cuál sería
su actitud a continuación?:

1. Solicitaría una pHmetría de 24 horas de doble canal para descartar reflujo esofágico con síntomas
faríngeos.
2. Iniciaría una prueba con Nifedipino y nitritos.
3. Repetiría la gastroscopia con toma de biopsias para descartar esofagitis eosinofílica.
4. Manometría esofágica para descartar una Achalasia cricofaríngea y poder realizar dilatación a
continuación del esfínter superior esofágico.

Resp. Correcta: 1

Comentario:

Se nos presenta un típico caso de posible reflujo gastroesofágico con síntomas ORL. Ante la no existencia
de lesiones en la gastroscopia, la siguiente exploración sería una pHmetría de 24 horas de doble canal (bajo
y alto) para descartar un reflujo patológico y correlacionarlo con los síntomas que refiere el paciente. La
hernia de hiato iría favor también de un posible reflujo gastroesofágico. Como el paciente no tiene síntomas
alérgicos previos y no relata disfagia, no sospecharíamos una esofagitis Eosinofílica. Tampoco podríamos
sospechar un trastorno motor, por lo que realizar una prueba con Nifedipino o pedir manometría no sería
adecuado.

-----------------------------------------------------------o--------------------------------------------------------------

Info Pregunta: 1f0c8457-9b87-4ed0-bce7-0b63de3ec8e0

38. ¿Cuál de estos fármacos se utiliza en el tratamiento de mantenimiento de la colitis


ulcerosa?:

1. Omeprazol.
2. Anti-H2.
3. Metronidazol.
4. Sulfasalazina.

Resp. Correcta: 4

Comentario: Se trata de una pregunta directa. En el tratamiento de mantenimiento de la colitis ulcerosa, se


emplean los 5'ASA (sulfasalazina). No se utilizarían como tratamiento de mantenimiento (sólo para los
brotes), los glucocorticoides, debido a los importantes efectos secundarios de estos fármcos si se emplean de
forma prolongada. El metronidazol queda reservado para los casos de enfermedad de Crohn con afectación
fistulosa perianal. El Omeprazol y los Anti-H2 no forman parte del tratamientro de mantenimiento de la
colitis ulcerosa.

-----------------------------------------------------------o--------------------------------------------------------------

Info Pregunta: 569aa157-3c62-49a2-8719-0baf2a48a92c

39. Varón de 22 años, con relaciones sexuales con otros hombres de riesgo, acude por
astenia e ictericia. En la exploración física únicamente destacaba ligera hepatomegalia
y abdomen globuloso. La analítica realizada mostraba los siguientes resultados: Hb
14.5 g/dl, leucocitos 7400/mm3, plaquetas 258.000/mm3, glucosa 102 mg/dl, urea 46
mg/dl, creatinina 0.6 mg/dl, GOT 160 U/L, GPT 285 U/L, GGT 200 U/L, bilirrubina
total 3.1 mg/dl, fosfatasa alcalina 55 U/L, LDH 160 U/L, anti-VHA IgM -, anti-VHA
IgG +, AgHBs +, antiHBs -, antiHBc +, AgHBe +, antiHBe -, DNA-VHB 1100 UI/ml,
Ag-VHD -, Ac-VHC -, RNA-VHC indetectable. Señale a cuál de las siguientes actitudes
sería más correcta:

1. Remitir a un centro trasplantador dado que posiblemente se instaure un fallo hepático agudo.
2. Iniciar tratamiento con Entecavir oral.
3. Iniciar tratamiento con Interferón pegilado.
4. Vigilancia analítica cada 6 meses.

Resp. Correcta: 3

Comentario:

El paciente no presenta hepatitis C dado que los anticuerpos y el RNA son negativos. La serología de VHA
corresponde a una hepatitis A pasada. Dado que presenta antígeno de superficie positivo y anticore positivo
de tipo IgG, puede considerarse una hepatitis crónica B. El Antígeno-e y el DNA son positivos, lo que
corresponde a una hepatitis crónica B por cepa salvaje replicativa. Dado que las transaminasas están
elevadas más de 2 veces lo normal estaría indicado tratamiento, siendo el más adecuado en la cepa salvaje
intentarlo con Interferón pegilado salvo que tenga otros factores de riesgo claramente de mal pronóstico de
respuesta.

-----------------------------------------------------------o--------------------------------------------------------------

Info Pregunta: d57bfcee-2857-4053-80d9-0beba6a782b4

40. Una de las siguientes afirmaciones es FALSA respecto a la gastritis crónica A.


Señálela:

1. Existe hiperplasia de las células G del antro.


2. La anemia perniciosa se produce al poco tiempo de originarse la enfermedad.
3. Existe hipo o aclorhidria.
4. Se han encontrado Ac contra factor intrínseco y células parietales.

Resp. Correcta: 2

Comentario:

La anemia perniciosa se debe a un déficit de vitamina B12. Las reservas hepáticas de vitamina B12, tardarían
varios años en agotarse desde el comienzo de la gastritis autoinmune porque son ciertamente abundantes
(opción 2 falsa, por lo que la marcamos). Sobre la anemia perniciosa, debes tener muy claro que, como
consecuencia de la destrucción de las células parietales, disminuye la secreción de ácido clorhídrico. Esto
hace que llegue al duodeno una menor cantidad de ácido, produciéndose una hiperplasia de células G que
sintetizarán una mayor cantidad de gastrina. Por tanto, siempre debes relacionar la anemia perniciosa con
hipoclorhidria e hipergastrinemia, igual que ocurre con los pacientes en tratamientos prolongados con
omeprazol.

-----------------------------------------------------------o--------------------------------------------------------------

Info Pregunta: dbb18257-f711-4bd3-87c4-0c7a8d13419b

41. En cuanto a la profilaxis de la infección por el virus de la hepatitis A, señalar la


FALSA:
1. La Ig en contactos con enfermos disminuye la frecuencia de enfermedad en un 80-90%.
2. Se debe administrar la Ig en las dos primeras semanas tras la exposición.
3. La vacunación está especialmente indicada en países subdesarrollados.
4. La Ig se recomienda en contactos personales estrechos.

Resp. Correcta: 3

Comentario: La vacunación del VHA está especialmente indicada ante estancias a paises subdesarrolados de
adecuadas medidas higienico- dietéticas hacen más posible su infección. Sin embargo la inmunización en
estos paises de forma rutinaria y en paises subdesarrolasos no está indicada dada la escasa agresividad del
mismo. La inmunización secundaria con Ig deberá realizarse dentro de las 2 semanas postexposición,
estando sólo recomendad en contactos estrechos. Esta disminuye la probabilidad de infección en un 80-
90%.

-----------------------------------------------------------o--------------------------------------------------------------

Info Pregunta: ab723997-ee5e-4801-9636-0cbfda2c9357

42. Mujer de 64 años, obesa, con dolor epigástrico intenso irradiado en cinturón,
febrícula, hiperamilasemia marcada, hematocrito del 48% y creatinina sérica
discretamente elevada. En la TC abdominal se evidencia un páncreas con
agrandamiento difuso, contorno irregular, inflamación alrededor de la glándula y una
acumulación de líquido intrapancreática. ¿Cuál de estas opciones es CIERTA en
relación con el manejo del paciente?

1. La reanimación mediante fluidoterapia enérgica es básica en el tratamiento.


2. Debe administrarse desde el inicio antibioterapia con fines profilácticos para evitar la necrosis
infectada.
3. La nutrición parenteral total está indicada a partir de las 48-72 horas.
4. El uso de inhibidores de la proteasa (aprotinina) ha demostrado un efecto beneficioso en estos casos.

Resp. Correcta: 1

Comentario: Pregunta respecto al manejo terapéutico de la pancreatitis aguda que no entraña dificultad
alguna. El pilar fundamental al inicio es el soporte volumétrico, dado que en el contexto de la inflamación
pancreática se produce extravasación del líquido intravascular. La reposición con fluidoterapia será
fundamental. No hay indicación de profilaxis antibiótica y de elección es la nutrición enteral. No hay
fármacos específicos para la inflamación del páncreas con eficacia demostrada.

-----------------------------------------------------------o--------------------------------------------------------------

Info Pregunta: 2af5957c-62e5-4a45-b7fb-0ce74d840622

43. Varón de 47 años, bebedor 50 g/día alcohol y fumador de 20 cig/día que acude por
dolor abdominal intermitente de meses de evolución pero que ha ido empeorando
progresivamente y diarrea intermitente de 3 meses de evolución. La exploración física
demostró la existencia de dolor abdominal en hemiabdomen superior a la palpación sin
peritonismo. La radiografía simple de abdomen mostraba un patrón gaseoso
inespecífico con pequeñas calcificaciones en región central de abdomen superior. La
analítica realizada fue la siguiente: Hb 11.9 g/dl, leucocitos 6.700/mm3, plaquetas
318.000/mm3, actividad de protrombina 70%, glucosa 144 mg/dl, urea 52 mg/dl,
creatinina 1.1 mg/dl, albúmina 3 g/dl, GOT 60 U/L, GPT 40 U/L, GGT 115 U/L,
bilirrubina total 1.2 mg/dl, fosfatasa alcalina 80 U/L, LDH 90 U/L, amilasa 18 U/ml.
Ante la entidad que debería sospechar, ¿qué tratamiento resultaría menos adecuado
actualmente?:

1. Suplementación con enzimas pancreáticos.


2. Suplementación con nutrición enteral.
3. Fitomenadiona oral.
4. Tramadol oral.

Resp. Correcta: 3

Comentario:

Se trata de un paciente bebedor y fumador que tiene clínica y radiología simple diagnósticas de pancreatitis
crónica. El paciente esta desnutrido y con dolor abdominal recurrente. La administración de vitamina K oral
(Fitomenadiona Oral) no sería útil ya que el paciente no presenta colestasis ni deterioro de coagulación.

-----------------------------------------------------------o--------------------------------------------------------------

Info Pregunta: b9994f4d-489b-4089-b1c7-0d1d347392f6

44. El medicamento de elección en la esofagitis grave por reflujo es:

1. Cimetidina.
2. Omeprazol.
3. Ranitidina.
4. Sucralfato.

Resp. Correcta: 2

Comentario: El uso de anti- H2 en enfermedad por reflujo es útil, pero la eficacia es inferior cuando se
compara con un inhibidor de la bomba de protones como es el omeprazol. El empleo de sucralfato no ha
demostrado utilidad. Los procinéticos tienen un nivel de eficacia semejante al de los anti- H2. La
combinación procinético - anti- H2 no mejora el resultado de cualquiera de estos fármacos en forma
independiente y el empleo de alcalinos sólo tiene interés como alivio sintomático y no aporta beneficio
terapéutico en la solución de la esofagitis.

-----------------------------------------------------------o--------------------------------------------------------------

Info Pregunta: cc0d368f-de7b-4466-9b43-0d2e92781ad7

45. Indique la variante genética del HLA que expresa el 90% de los pacientes celíacos:

1. DQ8.
2. DQ2.
3. B27.
4. B12.

Resp. Correcta: 2

Comentario:
Pregunta fácil sobre la predisposición genética en la enfermedad celiaca La enfermedad celiaca es una
intolerancia al gluten que aparece en individuos genéticamente susceptibles. En la base genética de la
enfermedad celiaca está implicado el sistema HLA (conjunto de genes localizados en el cromosoma 6, que
se encarga de vigilar que las células del organismo sean propias, es decir, que sepan diferenciar lo propio de
lo ajeno). Los HLA con mayor asociación con la predisposición a la celiaquía son DQ2 y DQ8, estando
presente HLA-DQ2 en el 90% de los celiacos y HLA-DQ8 en el 5%. Estos marcadores genéticos poseen
alto valor predictivo negativo, por lo que tenerlos no implica enfermedad, pero su ausencia sí descarta la
misma.

-----------------------------------------------------------o--------------------------------------------------------------

Info Pregunta: 71223788-75a7-4b65-9b85-0d5ce9318297

46. ¿Cual de los siguientes factores no es de alto riesgo para desarrollo de displasia
grave y adenocarcinoma en un paciente con colitis ulcerosa?:

1. Presencia de pseudopólipos inflamatorios.


2. Asociación con colangitis esclerosante primaria.
3. Afectación extensa de colon.
4. Antecedentes familiares en primer grado de cáncer colorrectal en menores de 50 años.

Resp. Correcta: 1

Comentario:

Los pseudopólipos inflamatorios en la colitis ulcerosa representan un riesgo moderado de desarrollo de


displasia grave y adenocarcinoma por lo que estos pacientes deben de ser revisados cada 2-3 años en vez de
forma anual. El resto de factores que se expresan si son de alto riesgo y requerirían de revisión con
colonoscopia anual.

-----------------------------------------------------------o--------------------------------------------------------------

Info Pregunta: 45df78b6-eb91-479a-89f9-0e18b29eef63

47. Mujer de 66 años que acude a Urgencias por dolor en epigastrio, así como náuseas
y vómitos alimentario-biliosos. La exploración física es de dolor epigástrico a la
palpación, con defensa y ruidos hidroaéreos negativos. La analítica fue: Hb 13,2 g/dL,
leucocitos 19.500/mm³, plaquetas 195.000/mm³, glucosa 111 mg/dL, urea 40 mg/dL,
creatinina 0,7 mg/dL, GOT 478 U/L, GPT 190 U/L, GGT 200 U/L, bilirrubina total 1,4
mg/dL, fosfatasa alcalina 311 U/L, amilasa 1.080 U/L. Se realiza un TC abdominal con
contraste i.v. en el que se observa un páncreas edematoso con colecciones
peripancreáticas y ausencia de captación de contraste en el 65% de la glándula. ¿Cuál
sería la actitud a continuación?

1. Necrosectomía profiláctica.
2. Imipenem i.v.
3. Carbapenem i.v. solo en caso de infección de necrosis infectada.
4. CPRE.

Resp. Correcta: 3

Comentario:
Según las últimas guías y recomendaciones, ya no se recomienda sistemáticamente la profilaxis antibiótica
en pacientes con pancreatitis graves. Solo debe ponerse en caso de infección demostrada de la necrosis. La
necrosectomía solo se realiza en caso de infección confirmada, por lo que no sería profiláctica. La paciente
tiene bilirrubina de 1,4 (casi normal) y en la prueba de imagen no nos dicen q haya coledocolitiasis, por lo
que en este momento no está indicada la CPRE. En caso de confirmar infección de la necrosis, ahí si estaría
indicado el tratamiento antibiótico (respuesta 3 correcta) y necrosectomía.

-----------------------------------------------------------o--------------------------------------------------------------

Info Pregunta: fd7572b6-9a17-4f8e-b75d-0e3eaf7c540e

48. ¿Cuál de las siguientes pruebas es MÁS útil para el diagnóstico de colangitis
esclerosante primaria?

1. ColangioRM.
2. TC (tomografía computarizada).
3. Biopsia hepática.
4. Colangiografía retrógrada endoscópica (CPRE).

Resp. Correcta: 1

Comentario:

La colangitis esclerosante primaria suele asociarse a la colitis ulcerosa, cosa que ocurre en aproximadamente
un 70% de los casos. El método diagnóstico de elección es la colangioRM, que mostraría una imagen con
múltiples estenosis y dilataciones a lo largo de los conductos biliares. Anteriormente al desarrollo de la RM,
se realizaba CPRE para objetivar dichas alteraciones, aunque al ser una técnica invasiva con tasa de
complicaciones no despreciable, ha sido relegada a un segundo plano, aplicándose con intención terapéutica
(dilataciones, colocación de prótesis, etc.). Entre las opciones que nos ofrecen, la biopsia puede resultar
tentadora, pero es una trampa. En la biopsia de la colangitis esclerosante primaria podríamos encontrar un
dato que se considera patognomónico: la lesión en piel de cebolla. El problema es que solo lo encontramos
en un 30% de los pacientes, por lo que su interés diagnóstico es escaso. En cambio, las estenosis y
dilataciones de la colangioRM aparecen en la mayoría de los casos.

-----------------------------------------------------------o--------------------------------------------------------------

Info Pregunta: c484e944-f483-4601-8cd2-0e3f635746c9

49. Señale el signo serológico que predice más tempranamente una evolución ulterior
favorable en la hepatitis aguda B, HBsAg seropositiva, de curso común:

1. La aparición de anti-HBc de clase IgM.


2. La negativización del antígeno HBs.
3. La negativización del antígeno HBe.
4. La aparición de anti-HBs.

Resp. Correcta: 3

Comentario: La negativización del AgHBe es el primer acontecimiento que ocurre en los pacientes que se
están curando ya que lo primero que ocurre es el cese de replicación, para posteriormente producirse la
negativización del antígeno de superficie (AgHBs), momento en el cual se considera que la infección está
curada.

-----------------------------------------------------------o--------------------------------------------------------------
Info Pregunta: 0f305954-59e0-4480-92d2-0e468015a81c

50. Se sabe que todos los procesos siguientes predisponen a la formación de cálculos de
colesterol, EXCEPTO:

1. Obesidad.
2. Edad avanzada.
3. Tratamiento con ácido ursodesoxicólico.
4. Resección quirúrgica del íleon.

Resp. Correcta: 3

Comentario:

El ácido ursodesoxicólico se emplea en la cirrosis biliar primaria. En esta enfermedad, ha demostrado ser el
fármaco más eficaz de los disponibles hasta el momento, aumentando el tiempo hasta el que se hace
necesario el trasplante hepático. Debe administrarse desde las fases iniciales de la enfermedad, aunque se
trate de pacientes asintomáticos. Otra aplicación del ácido ursodesoxicólico es el tratamiento médico de los
cálculos biliares. Por ello, la respuesta opción de respuesta que hay que marcar es la 3, ya que precisamente
hacen lo contrario: oponerse a la formación de estos cálculos. Este tratamiento se reserva para pacientes
asintomáticos, con cálculos de pequeño tamaño y vesícula funcionante. Presenta el inconveniente de que se
producen recidivas con mucha frecuencia, siendo más eficaz la colecistectomía.

-----------------------------------------------------------o--------------------------------------------------------------

Info Pregunta: 5580df08-f736-4b41-ba18-0e4fb4e44f9b

51. Mujer de 51 años, antecedentes personales de consumo de tabaco de hasta 20


cigarrillos al día, que acude refiriendo un cuadro de hemorragia digestiva alta, sin
relación con la ingesta. Refiere que ha venido padeciendo un dolor en epigastrio tras
las comidas durante varias semanas que se aliviaba con la administración de
antiácidos. Se realiza estudio con contraste, en el que no se observan imágenes
patológicas, por lo que se decide realizar estudio endoscópico que demuestra la
existencia de una úlcera de 7 mm de diámetro, con restos mínimos de sangre oscura
alrededor, en la primera porción del duodeno. Asimismo, las muestras confirman la
existencia de lesión por Helicobacter pylori. Indique, en relación con las medicinas
utilizadas en el tratamiento de la úlcera duodenal, la afirmación INCORRECTA:

1. Mientras que el hidróxido de aluminio puede producir estreñimiento, el hidróxido de magnesio tiene
efecto laxante.
2. Los antiácidos se emplean en la erradicación de Helicobacter pylori.
3. El sucralfato actúa protegiendo la mucosa gástrica.
4. El bismuto coloidal tiene acción frente a Helicobacter pylori.

Resp. Correcta: 2

Comentario:

Ten mucho cuidado con esta pregunta. Lo que se utiliza en la erradicación del H. pylori son los inhibidores
de la bomba de protones (IBP), como el omeprazol o los inhibidores del receptor de histamina tipo 2 (anti
H2), como la Ranitidina. Antiácidos son, por ejemplo, el bicarbonato sódico, el hidróxido de aluminio o el
de magnesio. Otra cosa es que los IBP reduzcan la secreción ácida gástrica, pero su mecanismo de acción no
es antiácido, sino inhibir la citada bomba de protones.

El Sucralfato es el protector de mucosa por excelencia y el Bismuto coloidal tiene actividad anti-
Helicobacter Pylori (de hecho se utiliza en la cuádruple terapia con Doxiciclina y Metronidazol).

-----------------------------------------------------------o--------------------------------------------------------------

Info Pregunta: 097ab3f4-c30a-4718-bf53-0f331180614d

52. Varón de 25 años que acude a su consulta por anemia. La exploración física era
normal salvo ligera palidez de mucosa conjuntival. Tras estudio analítico destacaban
Hb 10,3, VCM 74.3 fL, HCM 24 pg, ferritina 4, leucocitos 6.500 con 59% neutrófilos,
plaquetas 400.000, GOT 12, GPT 23, GGT 32, bilirrubina total 0,8, LDH 168, sodio
142, potasio 3,8. Los anticuerpos antitransglutaminasa y antigliadina fueron negativos.
Ya tuvo episodios similares a los 18 y 22 años. Aporta estudio de heces con sangre
oculta negativa, y además se realizaron estudios con gastroscopia, colonoscopia y
cápsula endoscópica normales, salvo test de ureasa positivo. ¿Qué cree
RECOMENDADO realizar a continuación?

1. Tratamiento antibiótico por sospecha de sobrecrecimiento bacteriano.


2. Solicitar estudio genético de enfermedad celíaca (HLA-DQ2 / DQ8).
3. Tratamiento erradicador de H. pylori.
4. Dieta exenta de gluten de forma empírica.

Resp. Correcta: 3

Comentario:

Se trata de un paciente con anemia ferropénica. Tras ser estudiado ampliamente, no se observa causa. Por
ello, al serH. pyloripositivo, puede probarse a erradicar (indicación relativa de consenso Maastricht III) ya
que algunos pacientes mejoran de la misma y no vuelven a presentarla.

-----------------------------------------------------------o--------------------------------------------------------------

Info Pregunta: d681e6a2-be74-4d6d-b8ca-0f4ba863428d

53. Paciente con molestias epigástricas, hipo, estado nauseoso y edema generalizado.
Analiticamente tiene marcada hipoproteinemia. En la gastroscopia se observan
pliegues gástricos pseudolipoides, gruesos y con erosiones varioliformes. ¿Cuál es su
diagnóstico?:

1. Cáncer gástrico.
2. Cirrosis hepática con varices esofágicas.
3. Poliadenomatosis.
4. Enfermedad de Ménétrier.

Resp. Correcta: 4

Comentario: En la enfermedad de Ménétrier se observan pliegues gruesos a nivel gástrico, con formaciones
que asemejan pequeños pólipos y alguna erosión por la hiperplasia de las células mucosas. Es inhabitual la
ulceración péptica. Con frecuencia se asocia a una pérdida luminal de proteínas, que pueden condicionar una
hipoalbuminemia con los consiguientes edemas. El cáncer gástrico en su variedad difusa provocaría pliegues
gástricos gruesos, pero, al igual que el linfoma, rígidos, y la posibilidad de una enteropatía pierdeproteínas
es menor.

-----------------------------------------------------------o--------------------------------------------------------------

Info Pregunta: 06a74605-a0fe-4dc5-bf70-0fb4e8e7a2e4

54. Todos, MENOS uno de los siguientes procedimientos, pueden ser adecuados para el
tratamiento del pseudoquiste de la cabeza del páncreas. Señálelo:

1. Quistogastrostomía.
2. Duodenopancreatectomía cefálica.
3. Quistoyeyunostomía.
4. Drenaje externo.

Resp. Correcta: 2

Comentario:

Debemos recordar que el pseudoquiste es la complicación más frecuente de la pancreatitis aguda, y que una
vez establecido el pseudoquiste una gran parte de ellos se resuelven espontáneamente (25- 40%), y que
además la localización más frecuente de los mismos es el cuerpo- cola del páncreas (hasta el 85%). Cuando
se establece la indicación quirúrgica de un pseudoquiste la técnica habitualmente empleada es la del drenaje
interno (a una víscera hueca del tubo digestivo alto, en forma de quistogastrostomía, quistoduodenostomía o
quistoyeyunostomía) y que en casos excepcionales por imperativo de la clínica se puede utilizar el drenaje
externo (con alta frecuencia de fístulas pancreáticas posteriores). Sólo en casos seleccionados de
pseudoquistes localizados en la cola del páncreas optaremos por una extirpación del mismo. En el caso que
nos ocupa, el pseudoquiste se localiza en la cabeza y la extirpación pasaría por realizar una
duodenopancreatectomía cefálica, actitud excesivamente agresiva y con alta frecuencia de complicaciones
teniendo en cuenta que existen otras técnicas menos agresivas y más sencillas de realizar.

-----------------------------------------------------------o--------------------------------------------------------------

Info Pregunta: fc66e83d-f073-4f98-a926-102a3d114b8f

55. ¿Cuál de las siguientes afirmaciones relativas al tratamiento de la hepatitis crónica


viral es FALSA?

1. Una respuesta viral mantenida (negativización del ARN) en la hepatitis crónica C se considera
curación de la infección viral.
2. La ribavirina produce típicamente anemia hemolítica, síndrome pseudogripal y alteraciones tiroideas.
3. La ausencia de respuesta viral temprana permite interrumpir el tratamiento en pacientes con hepatitis
crónica C, evitando efectos secundarios y costes económicos innecesarios.
4. La duración del tratamiento antiviral en la hepatitis crónica C depende del genotipo viral.

Resp. Correcta: 2

Comentario:

Tratan de confundirte entre los dos fármacos principales en el tratamiento de la hepatitis C. La ribavirina
produce típicamente anemia hemolítica, pero es el interferón el que produce síndrome pseudogripal y
alteraciones tiroideas.

-----------------------------------------------------------o--------------------------------------------------------------
Info Pregunta: 73a14290-1a0d-44f1-bb10-1049a1b80ca8

56. Señale la afirmación FALSA con respecto a la enfermedad por reflujo


gastroesofágico:

1. Debe estudiarse la infección por H. pylori y, en caso de ser positivo, erradicar.


2. La mayoría de estos pacientes no tienen esofagitis en la endoscopia.
3. Los síntomas cardinales de la enfermedad por reflujo gastroesofágico son la pirosis y regurgitación.
4. El test de Bernstein puede ser útil en algunos casos de hipersensibilidad visceral.

Resp. Correcta: 1

Comentario:

El reflujo esofágico no es una patología que deba erradicarse aun teniendo infección por H. pylori, y, de
hecho, en algunos casos, puede asociarse a empeoramiento del reflujo (opción 1 falsa, por lo que la
marcamos). La enfermedad por reflujo gastroesofágico es una patología cuyos síntomas cardinales son la
pirosis y la regurgitación; asimismo, se asocia a dolor torácico no cardiaco, asma y laringitis posterior. La
mayoría de los pacientes con enfermedad por reflujo gastroesofágico no presentan datos de esofagitis, de
forma que hay que recurrir a una pHmetría de 24 horas. En casos de no existir reflujo patológico y tener
síntomas, puede ser útil el uso del test de Bernstein para descartar un trastorno de hipersensibilidad visceral
subyacente.

-----------------------------------------------------------o--------------------------------------------------------------

Info Pregunta: 296f45af-b298-48a7-b035-10817ddf43bb

57. Mujer de 60 años sin antecedente de interés que acude por diarrea liquida de hasta
12 deposiciones al día matutinas, líquidas y sin productos patológicos acompañadas de
dolor abdominal cólico leve. La exploración física era normal. La analítica mostraba
los siguientes resultados: Hb 13.5 g/dl, leucocitos 6500/mm3, plaquetas 275.000/mm3,
glucosa 108 mg/dl, urea 34 mg/dl, creatinina 1 mg/dl, sodio 141 mEq/L, potasio 3.9
mEq/L, VSG 6 mm, calprotectina fecal normal. Se realizó una colonoscopia en la que
no se objetivaron alteraciones relevantes. Las biopsias por tramos únicamente
mostraron infiltrado llamativo de predominio linfocitico con células maduras. En
relación con la entidad que debería considerar, señale que característica consideraría
incorrecta:

1. No suele ser frecuente la esteatorrea.


2. Suele cursar con perdida de peso progresiva.
3. Tiene una etiología desconocida.
4. Su prevalencia es igual en ambos sexos.

Resp. Correcta: 2

Comentario:

La colitis linfocítica es una entidad de origen desconocido, igual de prevalente en los 2 sexos y no suele
acompañarse de rectorragia ni esteatorrea. La perdida de peso que aunque inicialmente acontece luego suele
estabilizarse.

-----------------------------------------------------------o--------------------------------------------------------------
Info Pregunta: 2642e37f-10af-473f-91ff-114a4ef5e0ab

58. Enfermo de 48 años diagnosticado de cirrosis hepática alcohólica al que venimos


controlando desde hace años. No ha conseguido dejar de beber alcohol. Desde hace 2
años presenta ascitis, que responde cada vez peor al tratamiento diurético. En el
momento actual toma 120 mg/día de furosemida, 300 mg/día de espironolactona y tiene
ascitis a tensión. Analíticamente destaca Na 121 mEq/1, K 4,7 mEq/l y creatinina 1,6
mg/dl. ¿Cuál es la actitud terapéutica más correcta en el momento actual?:

1. Proponer entrada preferente en un programa de trasplante hepático.


2. Establecer un régimen de paracentesis evacuadoras periódicas con reposición coloidal.
3. Insertar un TIPS.
4. Administrar una sobrecarga de suero salino hipertónico para forzar la diuresis.

Resp. Correcta: 2

Comentario:

Resumiendo la pregunta, se trata de una ascitis resistente a diuréticos. Cuando se produce esta situación,
existen varias posibilidades:

- Paracentesis evacuadoras: Incluso se podría extraer todo el líquido ascítico en una sola paracentesis. No
obstante, se debe tener la precaución de reexpandir el volumen plasmático con albúmina o dextranos, para
evitar complicaciones. En este caso, es la respuesta 2 la que debemos considerar correcta, dado que nos
dicen que es ascitis a tensión

- Shunt peritoneo- venoso de LeVeen: No ha demostrado prolongar la supervivencia, pero puede mejorar la
sintomatología. No obstante, tiene un riesgo muy alto de complicaciones, como infección, aparición de
coagulación intravascular diseminada, trombosis del shunt…. No obstante, en esta pregunta ni siquiera
aparece entre las posibles opciones.

- Trasplante hepático: La supervivencia al año de los pacientes con ascitis refractaria a diuréticos es del
25%. Por ello, plantear directamente un trasplante no es descabellado. Sin embargo, en nuestro caso nos
especifican que el paciente no ha dejado de beber, por lo que por el momento no estaría indicado el
trasplante.

- TIPS: Recientemente, se ha introducido la colocación del TIPS para pacientes con ascitis refractaria a
diuréticos. No obstante, la principal indicación de esta técnica es la prevención de la hemorragia digestiva
alta por varices en pacientes que están esperando un trasplante hepático, y en los que han fallado los
tratamientos endoscópicos y farmacológicos. Por ello, por el momento debemos preferir la respuesta 2.

-----------------------------------------------------------o--------------------------------------------------------------

Info Pregunta: 2334836a-6324-49e3-9277-115459286745

59. El test de la D-xilosa alterado indica:

1. Síndrome de malabsorción por defecto de la pared del intestino.


2. Esteatorrea pancreática.
3. Déficit congénito de disacaridasas.
4. Obstrucción biliar.

Resp. Correcta: 1
Comentario:

La D- xilosa se absorbe en el yeyuno por difusión pasiva, sin necesidad de que intervengan las enzimas
pancreáticas o las sales biliares. Por ello, está alterada en procesos que afectan a la pared del tubo digestivo
(respuesta 2 correcta), sin verse influida por alteraciones pancreáticas ni biliares.

El test de la D- xilosa consiste en administrar este azúcar en ayunas, y luego se mide su presencia en orina,
cuando pasan unas horas. Si la encontramos en orina, quiere decir que se ha absorbido a nivel intestinal.

-----------------------------------------------------------o--------------------------------------------------------------

Info Pregunta: 581e8ba0-3e46-4888-9974-1175fd07c661

60. Paciente que acude por hipertransaminasemia en el que se objetiva en sangre la


siguiente serología: Ac HBc IgM+, Ag HBs +, AgHBe +, DNA +. ¿Cuál es el
diagnóstico?

1. Hepatitis aguda.
2. Hepatitis B crónica AgHBe positivo
3. Mutación precore.
4. Portador inactivo

Resp. Correcta: 1

Comentario: La presencia de Ag de superficie denota la presencia del virus en el cuerpo. Al tener los anti-
core de tipo IgM indican infección reciente (hepatitis aguda).

-----------------------------------------------------------o--------------------------------------------------------------

Info Pregunta: abf19443-44c0-4c6d-8bcf-11f31f78faee

61. Mujer de 65 años sin antecedentes de interés que consulta porque desde hace un
año presenta diarrea. En la analítica : las grasas fecales son de 10 gr/24 h y se detecta
proteinuria de 3 gr/24 h. En el estudio radiológico se aprecia derrame plural derecho y
en la ecografía hepatomegalia. Entre las siguientes pruebas, ¿cual sería la más rentable
para obtener el diagnóstico?:

1. Test de Trioleína.
2. Biopsia de submucosa rectal.
3. Biopsia pleural.
4. Aclaramiento de alfa 1 antitripsina.

Resp. Correcta: 2

Comentario: Al observarse alteración en la absorción (con esteatorrea) y afectación de la función renal (con
proteinuria en rango nefrótico), ya se debe sospechar el diagnóstico de amiloidosis. Además, la presencia de
derrame pleural y de hepatomegalia, lo han todavía más sugestivo. Por tanto, la prueba diagnóstica que sería
inicialmente más rentable es la biopsia de grasa rectal. En ella esperaríamos encontrar depósito de sustancia
amiloide, causante de todo el cuadro clínico.

-----------------------------------------------------------o--------------------------------------------------------------

Info Pregunta: 9f0e824a-7187-4f9d-a5d1-12c78910b759


62. Señale, de entre los siguientes, cuál es el tratamiento MÁS adecuado para un
paciente con ingesta de 14 gramos de paracetamol:

1. Alopurinol.
2. Estreptomicina.
3. Resincolestiramina.
4. N-acetilcisteína.

Resp. Correcta: 4

Comentario:

El tratamiento de elección de la toxicidad por paracetamol es la N-acetilcisteína. Aunque es más eficaz en


las primeras horas tras la ingestión del paracetamol, se debe administrar en el momento en el que
diagnostiquemos la intoxicación independientemente del tiempo que haya pasado desde la ingesta.

-----------------------------------------------------------o--------------------------------------------------------------

Info Pregunta: 0c86de01-d121-4a33-873b-1316d88387b9

63. Con respecto a las lesiones agudas de la mucosa gástrica (LAMG), es


INCORRECTO que:

1. Las úlceras de Cushing aparecen como consecuencia de lesiones del SNC.


2. Las úlceras de Curling aparecen en los grandes quemados.
3. La hematemesis es una forma común de presentación.
4. Las úlceras de Cushing cursan con normo o hiposecreción.

Resp. Correcta: 4

Comentario:

Las gastritis agudas de estrés suelen expresarse clínicamente en forma de hemorragia digestiva, ya sea en
forma de hematemesis, melenas o anemización súbita. Aunque su localización puede ser difusa, predominan
en fundus. El mecanismo de la lesión es por isquemia y alteración de la barrera mucosa gástrica, como
sucede en las úlceras de Curling (grandes quemados). La excepción a esta regla son las úlceras de Cushing,
que se relacionan con la hipertensión intracraneal, sobre todo cuando es aguda, ya que produce una
hiperestimulación vagal y, como resultado, hipersecreción ácida gástrica (opción 4 incorrecta, por lo que la
marcamos).

-----------------------------------------------------------o--------------------------------------------------------------

Info Pregunta: ccb97234-a924-4ec1-a2f6-135dd81f93fa

64. Varón que fue diagnosticado hace seis meses de úlcera duodenal, Helicobacter
pylori +, instaurándose tratamiento médico. Acude de nuevo a la consulta por
persistencia de la sintomatología con persistencia endoscópica de la lesión ulcerosa,
para valorar indicación quirúrgica. Previo a la misma, debemos investigar posibles
causas de fracaso del tratamiento, entre las que usted indicaría todas las siguientes,
EXCEPTO:

1. Un tratamiento inadecuado del Helicobacter pylori: no impuesto o incumplido.


2. La persistencia de la toma de AINEs.
3. Toma de comidas picantes y abundantes.
4. Hábito tabáquico importante no abandonado.

Resp. Correcta: 3

Comentario:

Cuando existe persistencia de una úlcera secundaria a Helicobacter pylori tras el tratamiento, en la mayoría
de los casos el tratamiento ha sido inadecuado, por incumplimiento del paciente o por la utilización de
protocolos no suficientemente consensuados. Una vez descartada adecuadamente la presencia del
microorganismo, se deberá insistir en la posibilidad de la persistencia de la toma de AINEs o en el hábito
tabáquico que altera el proceso cicatricial y lo retrasa, precisando tratamientos más largos y a veces más
intensos; por su parte, el consumo de comidas copiosas o picantes no se ha relacionado con la persistencia
ulcerosa (marcamos la opción de respuesta 3).

-----------------------------------------------------------o--------------------------------------------------------------

Info Pregunta: 2ae5b09e-9241-4f4f-b112-136b397a89de

65. Todas las siguientes están descritas como causa de pancreatitis aguda, EXCEPTO:

1. Hipertrigliceridemia.
2. Alcohol.
3. Hipercolesterolemia.
4. Traumatismo abdominal.

Resp. Correcta: 3

Comentario:

La mayoría de las pancreatitis agudas (60-85%) se presentan en pacientes con colelitiasis y abuso de alcohol
(agudo y crónico). En un 10% se encuentra una posible causa identificada de las siguientes:
- Postoperatoria (abdominal, bypass aortocoronario).
- CPRE.
- Traumatismos (sobre todo abdominales no penetrantes).
- Metabólicas.
- Hipertrigliceridemia.
- Deficiencia de CII apoproteína.
- Hipercalcemia.
- Insuficiencia renal.
- Trasplante renal (multifactorial).
- Embarazo.
- Pancreatitis hereditaria.
- Infecciones.
- Virus: parotiditis, hepatitis vírica.
- Otros virus: Coxsackievirus,echovirus.
- Ascaridiasis.
- Micoplasma,Salmonella,Campylobacter jejuni.
- SIDA (multifactorial).
- Fármacos.
- Trastornos del tejido conectivo con vasculitis.
- Lupus eritematoso sistémico.
- Angeítis necrotizante.
- Púrpura trombocitopénica trombótica.
- Úlcera péptica penetrante.
- Obstrucción de la ampolla de Vater.
- Enteritis regional.
- Divertículo duodenal.
- Tumores pancreáticos: primarios o metastáticos.
- Páncreas dividido.
Entre un 8 y un 24% no se encuentra una explicación causal. En este caso, la hipercolesterolemia no se
contempla dentro de las causas identificadas de pancreatitis aguda.

-----------------------------------------------------------o--------------------------------------------------------------

Info Pregunta: 0d27aeda-3428-415a-883f-13951e259c32

66. Varón de 24 años que acude a Urgencias por disfagia súbita (mientras comía carne)
y sialorrea. En la endoscopia practicada en Urgencias tras extracción del bolo cárnico
impactado en esófago medio, se observa una mucosa normal. Se toma una biopsia que
únicamente demuestra la existencia de 45 eosinófilos por campo en la mucosa. La
analítica era la siguiente: Hb 13,6 g/dL, 11.300 leucocitos/mm³ con neutrófilos 54%,
linfocitos 25%, monocitos 13%, eosinófilos 2%, plaquetas 165.000/mm³, creatinina 1,0,
urea 66, GOT 25, GPT 38, GGT 13, bilirrubina 1,1, sodio 147, potasio 4,4. ¿Cuál es el
diagnóstico MÁS probable?

1. Síndrome hipereosinófilo.
2. Acalasia.
3. Esofagitis eosinofílica.
4. Esofagitis candidiásica.

Resp. Correcta: 3

Comentario:

La esofagitis eosinofílica (respuesta 3 correcta) es un trastorno caracterizado por la existencia de disfagia e


incluso impactaciones de repetición con una mucosa normal o un aspecto ligeramente traquealizado del
esófago y, por tanto, el diagnóstico es histológico (marcada infiltración de la mucosa por eosinófilos). La
diferencia con el síndrome hipereosinofílico radica en que no tiene eosinofilia plasmática marcada como
sucede así en este último.

-----------------------------------------------------------o--------------------------------------------------------------

Info Pregunta: cf63692f-2d73-4123-ac40-13ab5296e931

67. Paciente de 22 años que acude a usted por diarrea de tres semanas de evolución con
expulsión de sangre, fiebre de hasta 38ºC y dolor abdominal. No refiere antecedentes
de interés y en los últimos meses no ha viajado fuera de su localidad habitual. El
coprocultivo es negativo. ¿Cuál sería el PRIMER proceso que deberíamos descartar?

1. Amebiasis.
2. Enfermedad inflamatoria intestinal.
3. Diarrea enteroinvasiva por Shigella.
4. Neoplasia.
Resp. Correcta: 2

Comentario:

Una pregunta relativamente sencilla. Dada la negatividad del coprocultivo y la ausencia de viajes recientes,
se descartan las opciones 1 y 3. No parece probable una neoplasia, dada la edad del paciente y la ausencia de
síndrome constitucional, con lo que también descartamos la opción 4. La única opción razonable sería la 2,
cuya edad pico de presentación oscila entre los 20-30 años, y cuyas manifestaciones clínicas son
compatibles con el cuadro descrito.

-----------------------------------------------------------o--------------------------------------------------------------

Info Pregunta: c08a066c-25ca-4461-ac46-14136fdddb63

68. Varón de 38 años, profesional del taxi y con antecedente de pancolitis ulcerosa que
precisó colectomía urgente por perforación en el contexto de una colitis fulminante.
Tras mejoría clínica y nutricional deseaba volver a operarse para crear un reservorio.
Se procedió a ello contrayéndose un reservorio en J con anastomosis ileorrectal. A los 5
años comenzó con aumento d enumero de deposiciones líquidas con sangre hasta 15 al
día, fiebre y dolor pélvico. La analítica mostraba los siguientes resultados: Hb 10 g/dl,
leucocitos 12000/mm3, plaquetas 310.000/mm3, glucosa 102 mg/dl, urea 42 mg/dl,
creatinina 1.4 mg/dl, sodio 143 mEq/L, potasio 3.7 mEq/L, VSG 48 mm, calprotectina
fecal 400 mg/g. Se realizó una colonoscopia en la que se observaban múltiples
ulceraciones en el muñón rectal con neoíleon integro. Con la información de la que
disponemos, ¿cuál sería el tratamiento más adecuado?:

1. Budesonida en enemas y antibioterapia.


2. Infliximab i.v.
3. Mesalazina tópica.
4. Extirpación del reservorio e ileostomía terminal.

Resp. Correcta: 3

Comentario:

Se trata de un paciente con actividad en el muñón rectal y, por consiguiente, se trata de una Cuffitis y no una
reservoritis. Esta es una entidad que se comporta como una colitis ulcerosa al quedar recto residual. El
tratamiento por lo tanto será el mismo que una colitis ulcerosa.

-----------------------------------------------------------o--------------------------------------------------------------

Info Pregunta: 1b25b6b5-3ea4-4503-8b65-1433bd6369c9

69. Mujer de 75 años de edad, sin antecedentes de interés, que acude a urgencias por
dolor abdominal cólico difuso y vómitos alimentarios y biliosos. Se diagnostica cólico
biliar y se pauta tratamiento espasmolítico, pero la paciente empeora clínicamente,
apareciendo febrícula, distensión abdominal y disminución del peristaltismo. En la
radiografía de abdomen se observa: asas de intestino delgado dilatadas con
distribución en patrón "en escalera" y niveles hidroaéreos, pequeña imagen de
densidad calcio en cuadrante inferior derecho y aire en la vía biliar. Su sospecha
diagnóstica es:
1. Ileo biliar.
2. Colangitis.
3. Colecistitis enfisematosa.
4. Trombosis de la vena mesentérica.

Resp. Correcta: 1

Comentario: Se trata de una paciente que asocia síntomas típicos de cólico biliar con un cuadro
posteriormente de obstrucción intestinal, tanto clínico como radiológico. La presencia de imagen cálcica en
la fosa iliaca derecha (ileon terminal), asociada a niveles hidroaéreos por la obstrucción y la aerobilia, hacen
que el cuadro más probable sea el íleo biliar, es decir, la obstrucción intestinal por un cálculo biliar
impactado en ileon terminal, que ha conseguido pasar al tubo digestivo por una fístula colecistoduodenal.

-----------------------------------------------------------o--------------------------------------------------------------

Info Pregunta: fef90b13-0ff4-4bb0-9e28-1455b267ace3

70. ¿A qué diagnóstico corresponde el siguiente patrón manométrico? En el cuerpo


esofágico distal, las contracciones son de baja amplitud y son simultáneas, el esfínter
esofágico inferior es hipotenso:

1. Acalasia vigorosa.
2. Espasmo esofágico difuso.
3. Acalasia clásica.
4. Esclerodermia.

Resp. Correcta: 4

Comentario:

Esta pregunta parece más difícil de lo que es en realidad. Aunque aparentemente nos piden un dato
memorístico, podemos ir descartando opciones a partir de conocimientos básicos.

Las opciones 1 y 3 nos hablan de dos tipos de acalasia: la vigorosa y la clásica. La acalasia viene definida
por una ausencia de relajación del esfínter esofágico inferior con la deglución, y aquí nos cuentan todo lo
contrario: esfínter esofágico inferior hipotenso. Por tanto, descartaríamos estas dos opciones, teniendo en
cuenta que la única diferencia entre ellas es la presencia de ondas de mayor amplitud y repetitivas, similares
a las que aparecen en el espasmo esofágico difuso, en la acalasia vigorosa. Tampoco pensaríamos en la
opción 2, por ser en nuestro caso ondas de baja amplitud. En la esclerodermia se afectan especialmente los
dos tercios inferiores del esófago, por infiltración de la musculatura lisa, lo que hace que disminuya el
peristaltismo y la presión del esfínter esofágico inferior.

-----------------------------------------------------------o--------------------------------------------------------------

Info Pregunta: 6a7a8cc7-0465-41de-9027-14aed86b0cac

71. El esófago de Barrett es una entidad clinicopatológica estrechamente relacionada


con el reflujo gastroesofágico. Señale cuál de las siguientes afirmaciones es verdadera:

1. Es una metaplasia del epitelio escamoso del esófago, que es sustituido por un epitelio columnar de
tipo intestinal.
2. Se produce en un 30% de los pacientes con reflujo gastroesofágico crónico.
3. Tiene un alto índice de malignización, pudiendo alcanzar hasta un 30% de los casos.
4. La presencia de esófago de Barrett en un paciente es indicación de cirugía, dado el alto potencial
maligno de esta entidad.

Resp. Correcta: 1

Comentario:

El esófago de Barrett es una metaplasia intestinal que surge en esófagos con esofagitis por reflujo, en
general severas y de larga duración. No se conoce con precisión la incidencia, pero se observa en
aproximadamente entre el 8-20% de los pacientes con reflujo. Incrementa en riesgo de desarrollar
malignización en forma de adenocarcinoma, pero en un porcentaje pequeño, con una tasa inferior al 5%
anual. Es indicación de cirugía (esofaguectomía) cuando el esófago de Barret se acompaña de displasia
severa, porque en ese caso el riesgo aumenta mucho. En referencia al tratamiento del esófago de Barret a dia
de hoy, es controvertido si el tratamiento con IBP evita la aparición del esófago de Barret, su progresión o el
desarrollo de cáncer . No obstante, y puesto que el único factor conocido actualmente al esófago de Barret es
el reflujo, se recomienda a todos estos pacientes el tratamiento con IBP, a la misma dosis recomendada en
ERGE, o bien cirugía antirreflujo si son pacientes de bajo riesgo quirúrgico.

-----------------------------------------------------------o--------------------------------------------------------------

Info Pregunta: 692c6f74-068f-4bfc-afec-14db8b0a90b6

72. El marcador serológico más útil para el diagnóstico de una pancreatitis autoinmune
tipo I es:

1. El anticuerpo anticitoplasma de los neutrófilos (ANCA).


2. El anticuerpo antimitocondrial (AMA).
3. El anticuerpo antimicrosomal.
4. El nivel de Ig G4.

Resp. Correcta: 4

Comentario: Se trata de una pregunta sencilla en relación con la pancreatitis autoinmune (PAI). La elevación
de IgG4 es el parámetro más sensible (95%) y específico (97%) para el diagnóstico de la PAI tipo I. Además
es el parámetro más útil para determinar la actividad y predecir el curso de la enfermedad. Es importante
recordar que puede encontrarse elevado en el cáncer de páncreas o la pancreatitis crónica.Las otras opciones
hacen referencia a autoanticuerpos detectados en colitis ulcerosa y CEP (ANCA), cirrosis biliar primaria
(CBP) y enfermedades tiroideas (antimicrosomales).

-----------------------------------------------------------o--------------------------------------------------------------

Info Pregunta: 65a06131-0534-41cb-a774-14e248fe8f2f

73. La poliposis adenomatosa familiar (PAF) y el síndrome de Gardner se han puesto


en relación con la alteración de uno de los siguientes genes. Señálelo:

1. C-yes.
2. APC.
3. C-fos.
4. C-mos.

Resp. Correcta: 2

Comentario:
Pregunta sobre la genética de las poliposis hereditarias. A simple vista, puede parecer difícil, pero si
sabemos que APC significa Adenomatosis Polyposis Coli, y que este nombre se le puso por su relación con
la poliposis adenomatosa familiar, es evidente que la opción de respuesta correcta es APC (respuesta 2
correcta).

-----------------------------------------------------------o--------------------------------------------------------------

Info Pregunta: 207a8e78-debc-4ae8-81d6-157b95a3d9f6

74. El gastrinoma es un tumor neuroendocrino productor de gastrina originado en las


células G y que pude condicional un síndrome de hipersecreción ácida. Señale la
afirmación que considera incorrecta respecto a esta entidad:

1. Puede aparecer en el contexto de una neoplasia endocrina múltiple tipo 1.


2. La gammagrafía con octreótido tiene unos sensibilidad en el diagnóstico inferior a la ecoendoscopia
en los tumores localizados en duodeno y páncreas.
3. El tratamiento quimioterápico y con IBPs constituye el tratamiento de elección.
4. La esteatorrea es un síntoma que con frecuencia acompaña a la dispepsia ulcerosa que pueden padecer
estos enfermos.

Resp. Correcta: 3

Comentario:

El tratamiento elección es el quirúrgico con exéresis del tumor, y posteriormente tratamiento oncológico con
quimioterapia. El tratamiento médico es paliativo y consiste en un abordaje sintomático con IBP a dosis
altas. La esteatorrea es relativamente frecuente. El aumento de la secreción ácida por la hipergastrinemia
impide la activación de los enzimas pancreáticos por la gran cantidad de ácido presente en el duodeno, que
es incapaz de neutralizarse por el bicarbonato pancreático. La ecoendoscopia es superior a la gammagrafía
con octreótido. Puede presentarse aislada o en el contexto de una neoplasia endocrina múltiple tipo 1.

-----------------------------------------------------------o--------------------------------------------------------------

Info Pregunta: 86955ffb-26d3-45cf-9de4-15ae4632a1e9

75. Natalia es una chica universitaria de 22 años que acude a su consulta de digestivo
por dolor abdominal. Al interrogarla, le cuenta que estos cuadros de dolor suelen
durarle 3 días aproximadamente y a menudo se acompañan de diarrea. Además, la
frecuencia de los episodios se incrementa en períodos de exámenes. Usted tiene un
mente un diagnóstico. ¿Cuál de los siguientes síntomas/signos iría en contra de este?

1. El dolor abdominal mejora con la defecación.


2. Existe un cambio en la frecuencia de las deposiciones.
3. En ocasiones, la diarrea se acompaña de grandes cantidades de moco.
4. 4. Es frecuente la pérdida de peso en los períodos diarreicos.

Resp. Correcta: 4

Comentario:

Cuadro clínico de una paciente con síndrome de intestino irritable. Se trata de la enfermedad gastrointestinal
más frecuente. El hallazgo clínico más frecuente es la alteración del ritmo intestinal, asociado siempre a
dolor abdominal que mejora con la defecación. La pérdida de peso es un dato de alarma que debe
orientarnos a patología orgánica y que, por tanto, va en contra del diagnóstico de intestino irritable.

-----------------------------------------------------------o--------------------------------------------------------------

Info Pregunta: e42cc5e1-8e5a-451d-8887-15b3c285347e

76. Mujer de 37 años, afecta de una colitis ulcerosa extensa, presenta un brote grave
por el que se inicia tratamiento con prednisona en dosis de 1 mg/kg. Tras una semana
de tratamiento, la paciente no presenta mejoría. ¿Cuál es la siguiente medida
terapéutica a realizar?

1. Colectomía subtotal de urgencia y, en un segundo tiempo, proctectomía y reservorio ileoanal.


2. Asociar un inmunosupresor como azatioprina.
3. Asociar mesalazina en dosis de 4 gramos al día oral y triamcinolona rectal, 1 aplicación cada 12 horas.
4. Ciclosporina endovenosa 2 mg/kg.

Resp. Correcta: 4

Comentario:

Nos presentan a un paciente con colitis ulcerosa sin mejoría tras 7 días de tratamiento esteroideo en dosis
estándar (brote corticorresistente).

El tratamiento del mismo a día de hoy se recomienda con ciclosporina IV en dosis de 2mg/kg o bien con
Infliximab.

-----------------------------------------------------------o--------------------------------------------------------------

Info Pregunta: d840d681-0fbf-4bcc-a115-15e35541bf43

77. Ante un paciente con enfermedad de Crohn que va a comenzar tratamiento con
terapia biológica antiTNF (anticuerpos antifactor de necrosis tumoral alfa), ¿cuál de
las siguientes pruebas diagnósticas NO es necesaria antes de iniciar dicha terapia?

1. Valoración del estadio inmunitario mediante contaje linfocitario.


2. Serología del virus de hepatitis B (VHB).
3. Prueba de la tuberculina.
4. Radiografía de tórax.

Resp. Correcta: 1

Comentario:

Los anticuerpos contra el factor de necrosis tumoral son inhibidores potentes de los linfocitos T y con su
administración puede incrementar el riesgo de infecciones, motivo por el que antes de administrarlos es
necesario destacar la existencia de hepatitis B, Hepatitis C, VIH y tuberculosis. Para ello, se solicitan
serologías virales, Rx Tórax y Mantoux (PPD, prueba de la tuberculina).

-----------------------------------------------------------o--------------------------------------------------------------

Info Pregunta: dcce61fb-9143-48d9-b7fa-16052d05f3d0


78. Una mujer de 49 años acude a consulta por presentar rectorragia de heces de
aspecto mucoide. En el examen endoscópico se encuentra una tumoración plana de
superficie vellosa, de 4 x 3 cm, que se reseca endoscópicamente al parecer de forma
completa. Histológicamente se objetiva un adenoma velloso, descartándose infiltración
neoplásica del tallo vascular. ¿Cuál es la actitud que debe adoptar respecto a esta
paciente?

1. Resección segmentaria de la zona donde implantaba el pólipo, con anastomosis término-terminal


posteriormente.
2. Realizar los test genéticos oportunos para descartar una poliposis colónica familiar entre sus
familiares.
3. Realizar una TC cerebral para descartar tumores endocraneales, ya que los pólipos adenomatosos
pueden verse en el síndrome de Turcot.
4. Observación periódica.

Resp. Correcta: 4

Comentario:

Se trata de un adenoma velloso resecado completamente. El siguiente paso es el seguimiento endoscópico


periódico.

-----------------------------------------------------------o--------------------------------------------------------------

Info Pregunta: 9e92a2ed-8ae0-48f5-984d-16a3b65853fa

79. ¿Qué patología cutánea suele asociarse a la enfermedad celíaca?:

1. Dermatitis herpetiforme.
2. Pioderma gangrenoso.
3. Necrosis grasa metastásica.
4. Necrólisis epidérmica.

Resp. Correcta: 1

Comentario: Se trata de una pregunta directa. Y para contestarla , la opción más fácil y razonable es la
siguiente: La enfermedad celíaca es una enfermedad con una base autoinmune. Cualquier paciente que
presente una enfermedad con alteración en la autoinmunidad, tiene un riesgo aumentado de presentar otras
patologías o síndromes autoinmunes. Así, de las opciones que nos dan, la que tiene una base autoinmune
clara es la dermatitis herpetiforme.

-----------------------------------------------------------o--------------------------------------------------------------

Info Pregunta: c1694220-cffc-4f90-960e-16c3b7444951

80. En la peritonitis bacteriana esponténea del cirrótico no esperaría encontrar en el


líquido ascítico:

1. cultivo monomicrobiano
2. LDH baja
3. proteinas bajas
4. glucosa próxima a cero
Resp. Correcta: 4

Comentario:

El líquido ascítico en la peritonitis bacteriana espontánea se caracteriza:

- cultivo monomicrobiano

- proteinas < 1g/dl

- glucosa > 50mg/dl (similar a la plasmática)

- LDH < 225 U/l

La glucosa próxima a cero es características de las peritonitis bacterianas secundarias

-----------------------------------------------------------o--------------------------------------------------------------

Info Pregunta: 8db2b77f-7ed7-4694-be7c-16fc5323f7dd

81. Una mujer de 23 años, diagnosticada de colitis ulcerosa dos años antes, es ingresada
por un cuadro de fiebre, dolor, distensión abdominal y diarrea de múltiples
deposiciones líquidas. Se instaura un tratamiento con prednisona a razón de 1
mg/kg/día, pero después de tres días de tratamiento, no se observa una mejoría
significativa. ¿Cuál debe ser la actitud terapéutica en este caso?

1. Doblar la dosis de esteroides y, si no se aprecia mejoría, preparar a la paciente para cirugía.


2. Realizar una pancolectomía urgente ante la sospecha de megacolon tóxico.
3. Añadir ciclosporina i.v. a la pauta de tratamiento.
4. Pautar enemas de sulfasalacina.

Resp. Correcta: 3

Comentario:

Caso clínico sobre brote agudo de colitis ulcerosa. Es un brote grave, cursa con fiebre, dolor abdominal y
diarrea intensa. Además, no responde a corticoides a dosis adecuadas (1 mg/kg/día). Por tanto, sería
necesario ahora iniciar tratamiento con ciclosporina i.v. (respuesta 3 correcta) o antiTNF (infliximab). En
caso de fracaso al tratamiento, plantearíamos como última opción la cirugía.

-----------------------------------------------------------o--------------------------------------------------------------

Info Pregunta: b92bbb21-9986-4b99-b7c9-177f7f54117c

82. Durante una guardia, usted observa que uno de sus compañeros tiene un tono
amarillento. Él le explica que ya se lo han comentado en alguna ocasión durante las
guardias, y también cuando realiza mucho ejercicio. Sin embargo, la revisión médica
que le hicieron hace unos meses, al comenzar la residencia, no reveló ninguna
alteración en las enzimas hepáticas ni hiperbilirrubinemia. Su serología para virus
hepatotropos es negativa. Si en este momento solicitase una analítica con perfil
hepático, señale lo que esperaría encontrar:
1. GOT y GPT discretamente elevadas.
2. Fosfatasa alcalina elevada y, en menor proporción, GGT.
3. Hiperbilirrubinemia, acompañada de cierto grado de colestasis.
4. Hiperbilirrubinemia, probablemente inferior a 5 mg/dl, con transaminasas normales.

Resp. Correcta: 4

Comentario:

Nos describen un caso típico de síndrome de Gilbert. Pacientes jóvenes, en los que en situaciones de estrés
se produce un aumento exclusivamente de bilirrubina (elevación de < 5 mg/dl) a expensas de la fracción
indirecta.

-----------------------------------------------------------o--------------------------------------------------------------

Info Pregunta: 0d85f705-d9c0-4b8d-a9d0-17fde7fc0412

83. ¿cuál es el mecanismo de acción de los fármacos betabloqueantes en la hipertensión


portal cirrótica?

1. aumentan la presión portal por vasoconstricción esplácnica


2. disminuyen la presión portal por vasoconstricción esplácnica
3. aumentan la presión portal por vasoconstricción sistémica
4. aumentan la presión portal por vasodilatación sistémica

Resp. Correcta: 2

Comentario: Los betabloqueantes reducen la presión portal al disminuir el flujo sanguíneo esplácnico por
vasoconstricción de la circulación esplácnica.

-----------------------------------------------------------o--------------------------------------------------------------

Info Pregunta: 30e1d737-1b45-4aea-a9e6-186aa9aa778d

84. A un paciente de unos 30 años se le descubre una hernia hiatal. El paciente no


refiere síntomas ni molestias relacionadas. En sus antecedentes personales tan solo
aparece una colitis ulcerosa de la que está siendo tratado. ¿Qué debería hacer con el
paciente?

1. No hacer nada.
2. Programar una pHmetría ambulatoria de 24 horas.
3. Tratar al paciente con IBP durante 4 semanas y citar para revisión.
4. Implantar medidas higiénico-dietéticas como disminución del tabaco, alcohol, chocolate, menta y
comidas grasas. Evitar hacer comidas copiosas y elevar el cabecero de la cama.

Resp. Correcta: 1

Comentario:

La hernia hiatal (protrusión del estomago a través del hiato diafragmático) se relaciona con la aparición de
ERGE al favorecer la hipotonía del EEI, sus relajaciones anómalas y el inadecuado aclaramiento de la parte
más distal del esófago. No obstante, la mera presencia de hernia hiatal no implica necesariamente que exista
RGE: puede haber hernia hiatal sin RGE y RGE sin hernia. Por consiguiente, el hallazgo de una hernia
asintomática no requiere tratamiento.

-----------------------------------------------------------o--------------------------------------------------------------

Info Pregunta: 953e6819-f252-4167-be8c-1933df1da133

85. Se identificó un pólipo de aspecto benigno de 2,5 cm de diámetro, en el colon


sigmoide de una mujer de 55 años mediante el enema de bario. Aunque la escisión
intentada con la sigmoidoscopia flexible fue incompleta, se extrajeron varias porciones
de tejido adenomatoso que, en el examen histológico, no pareció maligno. El plan
terapéutico MÁS apropiado sería:

1. Colectomía sigmoide.
2. Colostomía y polipectomía sigmoides.
3. Resección segmentaria del pólipo con escisión de un margen de 3 cm de colon normal.
4. Sigmoidoscopia repetida para completar la escisión del pólipo.

Resp. Correcta: 4

Comentario:

Existen varios factores que influyen en la probabilidad de malignización de los pólipos adenomatosos. Uno
de ellos es el tamaño, con el que existe una relación casi directa. Cuando un pólipo supera los 2 cm, el riesgo
de malignización supera el 50%. Por tanto, si persisten restos de pólipo, sería como dejarle en el abdomen
una bomba de relojería. Está claro que habrá que marcar una opción que permita extirparlo y, entre las
opciones de respuesta que nos ofrecen, la única correcta es la 4. Dado que el estudio histológico no ha
demostrado malignidad, habría que hacer una resección lo más conservadora posible (escisión del pólipo,
simplemente, y vía endoscópica al ser la opción menos agresiva). En caso de que los restos del pólipo fuesen
tumorales, posteriormente podría plantearse una ampliación en función del resultado.

-----------------------------------------------------------o--------------------------------------------------------------

Info Pregunta: c449f2c8-db97-41bd-849f-195bfae0df90

86. En relación al adenoma hepático, señale la opción que considere CORRECTA:

1. Tiene bajo potencial de malignización.


2. Es más frecuente en el lóbulo hepático izquierdo.
3. La suspensión de anticonceptivos orales puede conseguir disminuir su tamaño.
4. Es un tumor poco vascularizado, por lo que no es útil el TC abdominal, sino la RMN.

Resp. Correcta: 3

Comentario:

El adenoma hepatocelular es un tumor que puede malignizar, siendo más frecuente en mujeres jóvenes y en
el lóbulo hepático derecho. Son tumores muy vascularizados y dentro de su tratamiento debe considerarse la
suspensión de anticonceptivos orales, a cuyo uso se asocian, ya que puede disminuir el tamaño de los
mismos.

-----------------------------------------------------------o--------------------------------------------------------------

Info Pregunta: 05a4132b-d4c0-4232-9f2b-19ae99e38f1e


87. La presencia de úlceras gástricas o duodenales puede observarse en las siguientes
entidades, EXCEPTO:

1. Colitis ulcerosa.
2. Enfermedad de Behçet.
3. Zollinger-Ellison.
4. Sífilis.

Resp. Correcta: 1

Comentario: En la colitis ulcerosa, la afectación del tubo digestivo queda limitada al colon. Puede afectar a
todo el territorio colónico (pancolitis), o sólo a una parte, normalmente la más distal (proctitis). Sin
embargo, no se observan lesiones ulcerosas a otros niveles del tracto digestivo, a diferencia de la
enfermedad de Crohn, que puede afectar, de forma parcheada, desde la boca hasta el ano. El resto de las
entidades descritas sí pueden manifestarse con úlceras gástricas o duodenales.

-----------------------------------------------------------o--------------------------------------------------------------

Info Pregunta: bae3f33d-b5ec-43c5-b304-19dfafd84d4e

88. Un paciente con cirrosis hepática grado C de Child acude a Urgencias por
presentar febrícula de 37,8 ºC, dolor abdominal localizado en ambos vacíos, continuo,
sin náuseas ni vómitos. A la exploración destaca la presencia de ictericia
cutaneomucosa, un hematoma en muslo derecho que el paciente relaciona con una
caída casual hace dos días, el abdomen es blando y depresible, doloroso a la palpación
de forma difusa sin signos de irritación peritoneal, con matidez a la percusión en
ambos flancos. Se le realizó una paracentesis diagnóstica en la que la cifra de leucocitos
era de 1.334/mm³ con 45% de neutrófilos. Señale la afirmación CORRECTA respecto a
la patología que presenta este paciente:

1. Se debe esperar al resultado del cultivo de líquido ascítico para iniciar el tratamiento.
2. La presencia de esta complicación no empeora el pronóstico del enfermo.
3. La coexistencia de insuficiencia renal aumenta la mortalidad.
4. La presencia en líquido ascítico de proteínas > 1 g/dL, aumenta el riesgo de presentar esta
complicación.

Resp. Correcta: 3

Comentario:

Una pregunta evidente, que podrías haber resuelto incluso sin saber nada del tema. Como sabes, la
insuficiencia renal por sí sola, puede ser suficiente como para comprometer la vida del que la padece. Pues
bien, si tienes un cuadro como el que nos plantean (peritonitis bacteriana espontánea), ¿cuándo tendrás más
probabilidades de morir? ¿Con insuficiencia renal o sin ella? Aunque no supieses de qué enfermedad se
trata, la insuficiencia renal es un agravante, con lo que la opción de respuesta 3 es correcta de forma forzosa.

-----------------------------------------------------------o--------------------------------------------------------------

Info Pregunta: ca8bf6d3-d93b-4bf9-8dd8-19ffa7e53b58

89. Varón de 48 años de origen nigeriano y con cirrosis por hepatitis crónica B acude a
consulta de revisión. Se encontraba prácticamente asintomático. En la exploración
física destacaba ictericia conjuntival y leves edemas de miembros inferiores. La
exploración cardiopulmonar y abdominal fueron normales salvo leve hepatomegalia.
La analítica de rutina mostraba los siguientes resultados: Hb 11.2 g/dl, VCM 101 fl,
leucocitos 9.000/mm3 con 60% neutrófilos, plaquetas 120.000/mm3, actividad
protrombina 60%, glucosa 89 mg/dl, urea 55 mg/dl, creatinina 1.1 mg/dl, albúmina 3.0
g/dl, GOT 96 U/L, GPT 175 U/L, GGT 308 U/L, bilirrubina total 2.6 mg/dl, fosfatasa
alcalina 58 U/L, LDH 107 U/L, sodio 138 mEq/L, potasio 4.2 mEq/L. Se realiza una
ecografía abdominal en la que se observa un hígado heterogéneo e irregular con una
lesión nodular de 8 cm. Se realiza un TC donde se confirman estos hallazgos siendo la
lesión de 82 mm con realce en fase arterial y lavado precoz en fase venosa. ¿En que
estadío de la BCLC se encontraría este paciente?:

1. Estadio A1 - BCLC.
2. Estadío A3 - BCLC.
3. Estadío B - BCLC.
4. Estadío C – BCLC.

Resp. Correcta: 3

Comentario:

Se trata de un paciente hepatópata con una lesión única de más de 5 cm (no cumple criterios de Milán) y que
además presenta ictericia. Su estado general es bueno dado que se encuentra asintomático (Performance
Status de la OMS de 0). Por todo ello, se encuentra en un estadío B de la BCLC. Si no hubiera presentado
ictericia o hipertensión portal, hoy día se trataría de un estadio A1.

-----------------------------------------------------------o--------------------------------------------------------------

Info Pregunta: 94e7aa90-e779-4513-93db-1a1df0888dd9

90. Señale cuál de las siguientes enfermedades se caracteriza por presentar en la


lámina propia macrófagos PAS + con bacilos intracelulares:

1. Enfermedad de Crohn.
2. Colitis ulcerosa.
3. Enfermedad de Whipple.
4. Histiocitosis duodenal.

Resp. Correcta: 3

Comentario: La biopsia es necesaria para realizar el diagnóstico de la enfermedad de Whipple. Se


caracteriza por presentar macrófagos PAS + en la lámina propia. Hay bacilos intracelulares Zhiel – Nissen
negativos, a diferencia de las infecciones por Micobacterium Avium Intracellulare.

-----------------------------------------------------------o--------------------------------------------------------------

Info Pregunta: 0d86abce-2e16-4c05-baef-1ad39ecbb680

91. Paciente de 67 años hipertenso, diabético e intervenido de hipertrofia benigna de


próstata, acude por vómitos de repetición y sensación continua de plenitud abdominal.
Relataba pérdida de 12 kg en los últimos 3 meses y estreñimiento. Se realizó una
gastroscopia que tuvo que interrumpirse por bezoar y restos alimentarios abundantes
en cavidad gástrica. La colonoscopia únicamente demostró la existencia de divertículos
aislados en sigma. Con la información de la que disponemos, señale cual de las
siguientes afirmación consideraría correcta:

1. La causa más frecuente suele ser idiopática.


2. Se recomienda el uso de refresco de cola o celulasa para pacientes con sintomatología leve.
3. El TC puede claramente descartar neoplasia subyacente.
4. Actualmente el uso de Eritromicina esta contraindicado por la alta incidencia de colestasis hepática
que provocaba.

Resp. Correcta: 2

Comentario: Por los datos de diabetes (causa más frecuente) y restos alimentarios en la gastroscopia debería
sospecharse una gastroparesia. Esta puede a su vez llevar al desarrollo de bezoares. En técnicas de imagen
estos bezoares pueden confundirse con neoplasias. La Eritromicina sigue utilizándose como procinético en
estos pacientes. La correcta sería la segunda opción ya que tanto el refresco de cola como la celulasa pueden
utilizarse para bezoares con síntomas leves.

-----------------------------------------------------------o--------------------------------------------------------------

Info Pregunta: 05e489bd-f58b-4485-83c9-1ae7711e78fe

92. El marcador serológico MÁS útil para el diagnóstico de una pancreatitis


autoinmune tipo I es:

1. El anticuerpo anticitoplasma de los neutrófilos (ANCA).


2. El anticuerpo antimitocondrial (AMA).
3. El anticuerpo antimicrosomal.
4. El nivel de Ig G4.

Resp. Correcta: 4

Comentario:

Pregunta sencilla en relación con la pancreatitis autoinmune (PAI). La elevación de IgG4 es el parámetro
más sensible (95%) y específico (97%) para el diagnóstico de la PAI tipo I. Además es el parámetro más útil
para determinar la actividad y predecir el curso de la enfermedad. Es importante recordar que puede
encontrarse elevado en el cáncer de páncreas o la pancreatitis crónica (respuesta 4 correcta). Las demás
opciones de respuesta hacen referencia a autoanticuerpos detectados en colitis ulcerosa y CEP (ANCA),
cirrosis biliar primaria (CBP) y enfermedades tiroideas (antimicrosomales).

-----------------------------------------------------------o--------------------------------------------------------------

Info Pregunta: fc36f788-fcd0-4ab7-a5b1-1b03877c114b

93. Varón de 36 años que acude por diarrea de hasta 12 deposiciones al día con sangre,
moco y pus de 3 meses de evolución. Asimismo refería pérdida marcada de peso y
fiebre de hasta 39.4 ºC. En la exploración física destacaba ligera palidez de mucosas y
bajo peso. Los coprocultivos y parásitos en heces fueron negativos. La analítica
mostraba anemia ferropénica moderada, VSG 70 mm, y calprotectina 980 mg/g. Se
realizó una colonoscopia donde se observaba una mucosa con patrón vascular ausente,
friable, algunas ulceras profundas y con sangrado espontáneo desde recto hasta ciego.
Tras tratamiento con esteroides y mesalazina mejoró significativamente quedando
prácticamente asintomático y con tratamiento de mantenimiento con mesalazina. La
colonoscopia de control a los 6 meses mostraba actividad muy leve a lo largo de todo el
trayecto y pseudopólipos inflamatorios residuales postcicatrización. Le pregunta dado
que su padre a los 62 años tuvo cáncer de colon izquierdo y porque ha leído que existe
un riesgo incrementado de neoplasia de colon en esta entidad. Señale de entre los
siguientes, cuál sería la actitud correcta respecto al cribado de displasia:

1. Iniciar cribado con colonoscopias a los 8 años de enfermedad y de forma anual.


2. Iniciar cribado con colonoscopias a los 14 años de enfermedad y de forma anual.
3. Iniciar cribado con colonoscopias en este momento y cada 2-3 años.
4. Iniciar cribado con colonoscopias a los 8 años de enfermedad y cada 2-3 años.

Resp. Correcta: 1

Comentario:

El paciente dado que tiene actividad persistente en todo el colon. Dado que tiene una pancolitis, este es un
factor de alto riesgo para desarrollo de displasia y, por consiguiente debe iniciar cribado con colonoscopias a
los 8 años de enfermedad y de forma anual.

-----------------------------------------------------------o--------------------------------------------------------------

Info Pregunta: 4eba8f77-2e2c-46e9-bfe6-1b216f09e3ef

94. Un paciente de 48 años, etílico crónico y diagnosticado de cirrosis hepática ingresa


en Urgencias con un cuadro de confusión moderada y asterixis de varias horas de
evolución. ¿Cuál de estas circunstancias cree que NO influyó en el desencadenamiento
de esta complicación?

1. Tratamiento con un ansiolítico los días previos.


2. Tratamiento diurético por su hepatopatía.
3. Tratamiento con betabloqueantes para profilaxis de hemorragias por varices.
4. Estreñimiento pertinaz en los últimos días.

Resp. Correcta: 3

Comentario:

Un paciente cirrótico con confusión y asterixis padece, casi con total seguridad, un cuadro de encefalopatía
hepática. Lo que nos preguntan, por tanto, es que señalemos el factor que no es capaz de desencadenarla.
Algunos tratamientos relativamente frecuentes en los pacientes cirróticos, como los diuréticos del asa (que a
veces se usan para el control de la ascitis) pueden producir encefalopatía hepática por las alteraciones del
medio interno que producen (alcalosis metabólica). El estreñimiento permite una mayor producción de
amonio, ya que permite un mayor contacto de las bacterias y las sustancias nitrogenadas. Sin embargo, el
uso de betabloqueantes para disminuir la presión portal no se relaciona con este problema (respuesta 4).

-----------------------------------------------------------o--------------------------------------------------------------
Info Pregunta: f04486bd-47cc-4ece-b958-1b99b2fd09c7

95. Un paciente de 28 años, adicto a drogas por vía parenteral presenta un cuadro
gripal seguido de ictericia. La analítica muestra ALT 950 U/l AST 825 U/l, fosfatasa
alcalina y gamma GT dentro de la normalidad, bilirrubina total 4,64 mg/dl directa 3,94
mg/dl. Serologías: HBs Ag negativo, Anti HBc IgM negativo. Anti VHC negativo, Anti
VHA IgM negativo. ¿Cuál de los siguientes exámenes debe incluirse en la siguiente
aproximación diagnóstica?

1. Anti VHA IgG.


2. Anti Hepatitis D IgM.
3. Anti HBc IgG.
4. RNA del virus C.

Resp. Correcta: 4

Comentario:

El cuadro que nos describen corresponde a una hepatitis aguda. En realidad, esta pregunta no ofrece ninguna
dificultad, si tienes unas pocas nociones sobre la interpretación de las distintas serologías para virus
hepatotropos.

- Dado que el HBsAg es negativo, este paciente no puede tener, en este momento, una hepatitis B. Por tanto,
carece de sentido solicitar anticuerpos IgG anti- HBc: Se descarta respuesta 3

- Donde no hay virus B, no puede haber nunca virus D. Opción 2 incorrecta.

- Si no existe IgM positivo para VHA, difícilmente tendrá una hepatitis A. Se excluye, en consecuencia, la
opción 1.

La respuesta correcta es la 4. A pesar de que los anticuerpos anti- VHC son negativos, esto no descarta una
hepatitis C. Recuerda que este virus tarda semanas, e incluso meses, en producir la seroconversión.
Teniendo esto en cuenta, habría que demostrar la infección buscando el genoma del virus, que en este caso
sería RNA.

Si además consideras que el paciente es ADVP (factor de riesgo para contraer esta infección), no queda otra
respuesta posible.

-----------------------------------------------------------o--------------------------------------------------------------

Info Pregunta: 3aeb444a-416f-42f5-af7f-1bb30f1b9789

96. Señale, de entre las siguientes, cuál es causa de diarrea motora:

1. Hidróxido de magnesio.
2. Enfermedad de Crohn.
3. Vipoma.
4. Hipertiroidismo.

Resp. Correcta: 4

Comentario: El hipertiroidismo, dado que aumenta la velocidad de tránsito intestinal es una causa de diarrea
motora. El hidróxido de magnesia provocaría diarrea osmótica. La enfermedad de Crohn es causa de diarrea
inflamatoria. El Vipoma es un tumor que produce una diarrea secretora.

-----------------------------------------------------------o--------------------------------------------------------------

Info Pregunta: 8c33ac88-c50c-4893-8774-1bdb4b7c3ca7

97. ¿Cuál de estas alteraciones NO se asocia a un aumento aislado de la fracción


indirecta de la bilirrubina del suero?

1. Ictericia fisiológica del recien nacido.


2. Ictericia hemolítica.
3. Síndrome de Gilbert.
4. Síndrome de Dubin-Johnson.

Resp. Correcta: 4

Comentario:

De entre las que se citan, el síndrome de Dubin- Johnson no cursa con aumento de la fracción indirecta, sino
de la directa, como consecuencia de una alteración en la excreción de la bilirrubina ya conjugada.

La ictericia fisiológica del recién nacidos se produce por inmadurez del sistema enzimático encargado de la
glucuronoconjugación, mientras que la producida por lactancia por la secreción en la leche de pregnano- 3-
beta- 20- alfa- diol, que inhibe dicho sistema.
El síndrome de Gilbert sería un trastorno caracterizado por una alteración cualitativa de la UDP-
glucuroniltransferasa, que eleva la bilirrubina indirecta.
La ictericia hemolítica aumentaría el pool de hemoglobina destruida y por tanto de bilirrubina indirecta, que
llega a saturar los sistemas enzimáticos y, por tanto, a acumularse.

-----------------------------------------------------------o--------------------------------------------------------------

Info Pregunta: 2446736c-d460-4bda-93bd-1c0075ad11b4

98. Varón de 40 años, cirrótico por VHC con varios episodios de encefalopatía hepática
y en tratamiento con espironolactona 200 mg/día y furosemida 80 mg/día. Acude a
Urgencias por fiebre de 38 ºC y tos productiva herrumbrosa. En la exploración física
destacaba disminución de murmullo vesicular en base pulmonar derecha que, tras la
realización de una Rx de tórax, se confirmó como un foco neumónico a ese nivel. A las
48 horas comienza con oliguria, y en la analítica practicada en planta se observaba: Hb
11,5 g/dL, VCM 81,5 fL, leucocitos 18.000/mm³ con 92% neutrófilos, plaquetas
68.000/mm³, glucosa 101 mg/dL, urea 214 mg/dL, creatinina 3,1 mg/dL, proteínas
totales 4,4 g/dL, albúmina 2,7 g/dL. GOT 92 U/L, GPT 168 U/L, GGT 121 U/L,
bilirrubina total 3,8 mg/dL, fosfatasa alcalina 148 U/L, LDH 387 U/L, sodio 149,
potasio 3,5. Tras retirada de diurético y administración de sueroterapia 3.000 cc/24
horas, continúa con diuresis 300 cc/día y la analítica no demostró cambios relevantes.
¿Cuál sería la actitud MENOS adecuada a continuación?

1. Terlipresina i.v.
2. Albúmina i.v.
3. Reiniciar diurético para reiniciar diuresis.
4. Sedimento orina.
Resp. Correcta: 3

Comentario:

Ante un caso de síndrome hepatorrenal (paciente con insuficiencia renal que no mejora con expansión de
volumen y con escaso sodio en orina), lo menos indicado sería reiniciar los diuréticos para intentar reiniciar
la diuresis, ya que provocaría empeoramiento de la insuficiencia renal. El sedimento permitiría comprobar la
escasa cantidad de sodio en la orina y descartar otras causas de insuficiencia renal, y las demás opciones de
respuesta son tratamientos aceptados del síndrome hepatorrenal tipo 1.

En el síndrome hepatorrenal, el tratamiento de primera elección es la expansión de volumen con albúmina


combinado con fármacos vasoconstrictores, siendo de elección la terlipresina

-----------------------------------------------------------o--------------------------------------------------------------

Info Pregunta: 71508eeb-8379-42cb-842a-1c796381555a

99. Mujer de 54 años con cuadro de tres meses de evolución manifestado por prurito,
ictericia, coluria y pérdida de peso de aproximadamente 9 kg en el mismo período. Se
le realizan pruebas de laboratorio que muestran: bilirrubina 9 mg/dL, bilirrubina
directa 6 mg/dL, GOT 53 U/L, GPT 60 U/L, GGT 66 U/L, FA 301 U/L, Amilasa 189
U/L, LDH 257 U/L. En la ecografía abdominal se observó dilatación de vía biliar intra
y extrahepática, por lo que se realizó TAC abdominal que objetivó la presencia de una
masa en cabeza de páncreas. Se decide laparotomía exploradora evidenciándose
tumoración de consistencia dura de 3-4 cm en cabeza de páncreas que, biopsiada
intraoperatoriamente, se informa de: marcada fibrosis interlobular y prominente
infiltración de linfocitos y células plasmáticas IgG4+ en el área periductal. ¿Cuál sería
su actitud INICIAL?

1. Resección quirúrgica de la masa.


2. Prednisona.
3. Ácido ursodesoxicólico.
4. Somatostatina.

Resp. Correcta: 2

Comentario:

Caso clínico sobre la pancreatitis autoinmune (PAI), preguntada en la última convocatoria. Recuerda su
asociación a otras enfermedades autoinmunes y, como dato especifico, el incremento de la IgG4. El
tratamiento inicial de la PAI se basa en corticoides, con una respuesta espectacular a estos. No hay un
consenso acerca de la dosis y el tiempo de tratamiento. Se recomienda iniciar con prednisona 40 mg/día
durante 4 semanas, seguido de un descenso progresivo de 5 mg/semana.

-----------------------------------------------------------o--------------------------------------------------------------

Info Pregunta: f9fc859e-9b27-4bf3-9aed-1c99d13d79ab

100. En el síndrome de Gardner NO suelen hallarse:

1. Quistes epidermoides.
2. Fibromas.
3. Tumores desmoides.
4. Papilomas vellosos múltiples.

Resp. Correcta: 4

Comentario:

El síndrome de Gardner cursa con pólipos adenomatosos como la poliposis colónica familiar pero también
con manifestaciones extraintestinales como osteomas en cráneo, mandíbula y huesos largos, tumores
desmoides, anormalidades dentales, quistes epidermoides y sebáceos, lipomas, fibromas, tumores de
tiroides, glándulas suprarrenales arbol biliar e hígado pero no cursa con papilomas vellosos.

-----------------------------------------------------------o--------------------------------------------------------------

Info Pregunta: 9f5126d2-15c1-457a-9b36-1cc32ab2472f

101. Con respecto a las enfermedades por reflujo gastroesofágico, todas las
afirmaciones siguientes son falsas, EXCEPTO:

1. Cuando existe una esofagitis química, se debe colocar una sonda nasogástrica para aspirar el
contenido gástrico.
2. Los pacientes con acalasia presentan con frecuencia síntomas de reflujo gastroesofágico.
3. El principal síntoma inicial de la esofagitis química es el dolor retroesternal.
4. Uno de los tratamientos más eficaces de los cuerpos extraños en el esófago es la administración de
sustancias gastrocinéticas (por ejemplo: metoclopramida).

Resp. Correcta: 3

Comentario:

En esta pregunta es importante que sepas el motivo que invalida cada una de las opciones de respuesta.
Veámoslas:
1.- Debes evitar el sondaje nasogástrico ante una esofagitis química, debido al riesgo que implica producir el
vómito, lo que podría implicar una segunda exposición de la mucosa esofágica al cáustico o al ácido
ingerido.
2.- La acalasia se caracteriza por una falta de relajación del esfínter esofágico inferior. Sería extraordinario
que se produjese reflujo, cuyo fundamento es, precisamente, lo contrario.
3.- Es la opción de respuesta correcta.
4.- El uso de procinéticos podría incrementar la motilidad esofágica y, en presencia de un cuerpo extraño,
sería hasta peligroso.

-----------------------------------------------------------o--------------------------------------------------------------

Info Pregunta: e5c55449-9df9-4ce4-a8fc-1cd8e4da9cc4

102. ¿En qué enfermedad pensaría usted ante una enferma de 50 años, diagnosticada
de hipotiroidismo hace 8 años, y que presenta sequedad de boca, picor de ojos, de
palmas de las manos y plantas de los pies?

1. El diagnóstico más probable es un fracaso renal agudo.


2. Pensaría que son síntomas inexplicables y que corresponden a una enferma funcional.
3. Sería interesante la determinación de fosfatasa alcalina.
4. Debe realizarse un TC cerebral con urgencia, ya que hay tumores cerebrales que se presentan con
prurito.

Resp. Correcta: 3

Comentario:

La causa más frecuente de hipotiroidismo en nuestro medio es autoinmune… Y las enfermedades


autoinmunes se asocian entre sí con cierta frecuencia. Teniendo en cuenta la clínica, que es una mujer de
mediana edad y que tiene antecedentes de hipotiroidismo, habría que pensar en una posible cirrosis biliar
primaria. El prurito puede ser el síntoma inicial, y la sequedad de boca y picor de ojos pueden orientar a un
síndrome seco asociado, que no es infrecuente. Todos los pacientes con esta enfermedad presentan elevación
de la fosfatasa alcalina desde el principio de la enfermedad, por lo que la determinación de ésta sería la
primera prueba de laboratorio que solicitaríamos, antes de realizar otras más específicas (como los AMA).

-----------------------------------------------------------o--------------------------------------------------------------

Info Pregunta: a5aa5d34-edf4-4dc1-92e5-1d552fefa999

103. Señale la afirmación que considere correcta en relación con la hepatitis


fulminante:

1. La causa más frecuente en nuestro medio es la intoxicación por paracetamol.


2. No puede considerarse indicación de trasplante hepático ortotópico hasta llegar a una encefalopatía
grado IV.
3. El tiempo de protrombina suele estar alargado en consonancia con una actividad de protrombina
inferior al 80%.
4. Ante un paciente con hepatitis fulminante y una anemia hemolítica coombs negativa debe sospecharse
en primer lugar una enfermedad de Wilson.

Resp. Correcta: 4

Comentario:

En hepatitis fulminante es característica la presencia de encefalopatía y disminución de la actividad del


tiempo de protrombina por debajo del 40% en un hígado previamente sano. La causa mas frecuente en
nuestro medio es la hepatitis B (sobre todo, sobreinfección D e infección B por mutantes precore). Estos
pacientes deben remitirse a un centro de trasplante a la mayor brevedad para instaurar tratamiento intensivo
de mantenimiento e indicar trasplante urgente (urgencia cero) cuando tienen encefalopatía grado II-III.

-----------------------------------------------------------o--------------------------------------------------------------

Info Pregunta: b7ea035e-45ff-44b4-9039-1d57f8e4c1c8

104. El diagnóstico de gastritis crónica se realiza FUNDAMENTALMENTE mediante:

1. Valoración de Helicobacter pylori.


2. Determinación de la vitamina B12.
3. Estudio histológico.
4. Estudios radiológicos.

Resp. Correcta: 3

Comentario: La gastritis es un diagnóstico histológico que requiere la realización de endoscopia con toma de
biopsias.
-----------------------------------------------------------o--------------------------------------------------------------

Info Pregunta: 089a341c-149f-4a6a-87df-1d83632a98a0

105. Varón de 45 años que acude a su consulta al observarse en analítica de rutina de


empresa la siguiente analítica: Hb 16,5 g/dL, VCM 80,5 fL, HCM 30,1 pg, leucocitos
7.100/mm³ con 62% neutrófilos, plaquetas 199.000/mm³, glucosa 102 mg/dL, urea 23
mg/dL, creatinina 0,7 mg/dL, proteínas totales 7,6 g/dL, albúmina 4,5 g/dL, GOT 47
U/L, GPT 45 U/L, GGT 67 U/L, bilirrubina total 0,9 mg/dL, fosfatasa alcalina 158 U/L,
LDH 190 U/L, sodio 139, potasio 3,9, ferritina 654 ng/ml, IST 49%, ANA-, anti-LKM-,
AMA-, antimúsculo liso-, anti-Sm-. Serología viral: antiHBs+, AgHBs-, AntiHBc+,
AgHBe-, antiHBe+, anti-VHC-, AgVHD-, anti-VHD IgM-, anti-VHD IgG-, IgG VHA+.
Sobre el diagnóstico que usted sospecha, señale la opción que considere CORRECTA:

1. El hipogonadismo es un síntoma constate en esta entidad.


2. Es más frecuente en África que en Europa.
3. Un análisis genético posible podría ser el siguiente: H63D+/C282Y+.
4. Presenta característicamente HLA-DQ2.

Resp. Correcta: 3

Comentario:

Se trata de una paciente con hipertransaminasemia leve en probable relación con una hemocromatosis (dado
que tiene aumento de ferritina con IST > 45%). Los genotipos posibles que confieren la enfermedad en el
análisis del gen HFE son la homocigosis C282Y/C282Y y la heterocigosis compuesta C282Y/H63D. La
mutación C282Y (la más frecuente) es más frecuente en poblaciones nórdicas de origen celta. El
hipogonadismo es un síntoma típico, pero no aparece en el 100% de los pacientes. El HLA-DQ2 es típico de
la enfermedad celíaca.

-----------------------------------------------------------o--------------------------------------------------------------

Info Pregunta: 90cb55f4-e121-4df2-813c-1da263bec855

106. En relación con los efectos secundarios del tratamiento médico de la enfermedad
inflamatoria intestinal, es INCORRECTO:

1. La azatioprina puede asociase a pancreatitis aguda que, en caso de producirse, precisa suspenderse,
aunque puede reiniciarse una vez solucionado el proceso.
2. El metronidazol debe evitarse a largo plazo por asociarse a neuropatía periférica irreversible.
3. Los biológicos pueden provocar una reactivación de tuberculosis latente.
4. La mesalazina puede asociarse a insuficiencia renal.

Resp. Correcta: 1

Comentario: La azatioprina puede asociarse a pancreatitis aguda por mecanismo inmunoalérgico por lo que
no puede volverse a reiniciar nunca más en caso de producirse. Los inmunosupresores como la azatioprina/6
mercaptopurina en estudios recientes se han demostrado que son seguros durante el embarazo. Antes de
utilizar un fármaco biológico es preciso investigar el estado portador de tuberculosis para evitar el desarrollo
de una tuberculosis clínica grave.

-----------------------------------------------------------o--------------------------------------------------------------
Info Pregunta: a53bdb2d-d3ed-4797-ada9-1da7ea51f23b

107. Una mujer de 46 años acude al servicio de Urgencias por presentar desde hace 4
días deposiciones negras. Refiere en la última semana intensa pirosis y ocasionalmente
dolor retroesternal. En la exploración presenta palidez cutáneo-mucosa y en la
analítica, anemia microcítica, Hb 8,9, hto 26% VCM 70. La endoscopia evidencia
erosiones longitudinales confluentes en el tercio inferior del esófago con alguna
ulceración cubierta de fibrina. ¿Cuál de las siguientes afirmaciones es FALSA?:

1. Iniciar tratamiento con omeprazol.


2. No es necesario realizar endoscopia de control para toma de biopsias.
3. Reposo relativo.
4. Dieta blanda durante los primeros días.

Resp. Correcta: 2

Comentario:

La presencia de una hemorragia digestiva obliga siempre a una endoscopia urgente. Esta técnica será
diagnóstica y, en ocasiones, terapéutica. Esta situación también obliga a endoscopias de control. El
tratamiento médico de elección es el omeprazol a alta dosis y la presencia de anemia, si se constata
ferropénica, requiere tratamiento con suplementos de hierro.

-----------------------------------------------------------o--------------------------------------------------------------

Info Pregunta: f477897d-f48b-49d1-ac43-1e61b7c629aa

108. Una mujer de 47 años acude por presentar ascitis de nueva aparición, junto con
ictericia y encefalopatía. Las pruebas de laboratorio revelaron los siguientes
resultados: GOT 756, GPT 987, bilirrubina 7,6 mg/dl, INR 1,7; Ig M Hbc negativo, Ag
Hbs negativo, RNA-VHC negativo, IgM VHA negativo, AMA 1:40, ANA 1:40, ASMA
1:320, LKM-1 1:40. ¿Cuál sería su actitud terapeútica?:

1. Propiltiuracilo.
2. Prednisona.
3. D-penicilamina.
4. Colchicina.

Resp. Correcta: 2

Comentario: Nos presentan un paciente con un cuadro de hepatopatía con cierto grado de evolución y con
serología viral negativa y que se asocia a determinados autoanticuerpos, lo que nos hará pensar en una
hepatitis autoinmune. La hepatitis autoinmune tipo 1 cursa con ANA y anti- músculo liso positivos mientras
que la tipo 2 cursa con anticuerpos anti- KLM y anti- citosol hepático. El tratamiento de las hepatitis
autoinmunes serían, como en el resto de procesos autoinmunitarios en general, con corticoides y en caso de
falta de respuesta podríamos añadir inmunosupresores.

-----------------------------------------------------------o--------------------------------------------------------------

Info Pregunta: 1e2bb9dc-f66d-4698-b7d4-1e73e7ca7166

109. Mujer de 67 años, bebedora de 65 g/día de alcohol y fumadora, que acude a su


consulta por aumento de perímetro abdominal e ictericia conjuntival leve. En la
exploración física se observó ligera disminución de murmullo vesicular en ambos
campos pulmonares inferiores, leve dolor en hipocondrio derecho a la palpación
profunda y matidez en región inferior de ambos flancos que desaparece con el cambio
a decúbito lateral. En la analítica destacaba: Hb 14.0 g/dL, VCM 88.5 fl, HCM 31.1 pg,
leucocitos 8000/mm³ con 65% neutrófilos, plaquetas 78.000/mm³, glucosa 105 mg/dl,
urea 78 mg/dl, creatinina 2.1 mg/dL, GOT 175 U/L, GPT 98 U/L, GGT 301 U/L,
bilirrubina total 2.7 mg/dL, proteínas totales 5.9 g/dL, albúmina 3.1 g/dL, fosfatasa
alcalina 188 U/L, LDH 87 U/L, sodio 149, potasio 3.4. ¿Cuál sería la actitud MÁS
correcta a continuación?

1. Paracentesis evacuadora.
2. Tratamiento diurético con espironolactona 300 mg/día oral.
3. Paracentesis diagnóstica.
4. TIPS.

Resp. Correcta: 3

Comentario:

Ante esa exploración física, debe sospecharse una ascitis. En caso de paciente que acude con ascitis
(matidez cambiante en flancos), debe realizarse siempre una paracentesis diagnóstica para descartar
peritonitis bacteriana espontánea. No debe realizarse en este momento ni una paracentesis evacuadora
(porque no tiene ascitis a tensión), ni iniciar diuréticos a dosis altas, ya que tiene además insuficiencia renal
(por ello sería recomendable iniciar tratamiento con albúmina i.v. ahora para mejorar la función renal). El
TIPS, pese a tener deteriorada la función renal, no se coloca en este momento al no considerarse una ascitis
refractaria.

-----------------------------------------------------------o--------------------------------------------------------------

Info Pregunta: aa411ec2-4c32-4a9c-a1c4-1fe55f7e9a3a

110. Señale la asociación FALSA entre el fármaco utilizado en el tratamiento de la


úlcera péptica y su mecanismo de acción:

1. Omeprazol - inhibidor de la bomba de protones.


2. Hidróxido de aluminio-protector de la mucosa gástrica.
3. Famotidina - antagonista de los receptores histamínicos.
4. Sucralfato - protector de la mucosa gástrica.

Resp. Correcta: 2

Comentario: El hidróxido de aluminio es un fármaco antiácido, que únicamente es útil para el alivio
sintomático, fundamentalmente de la pirosis. No tiene efecto protector de la mucosa gástrica, ni efecto
antisecretor. Por tanto, no puede ser empleado en las terapias erradicadoras del Helicobacter pylori, ni en la
cicatrización de la úlcera.

-----------------------------------------------------------o--------------------------------------------------------------

Info Pregunta: d1e36f9e-98a4-4481-b0bf-1feb2de55d22

111. ¿Cuál de los siguientes fármacos NO se emplea en el tratamiento de la hepatitis


autoinmune?

1. Prednisona.
2. Budesonida.
3. Larnivudina.
4. Micofenolato mofetil.

Resp. Correcta: 3

Comentario:

Se trata de una pregunta fácil de resolver aunque no conozcamos todas las opciones de tratamiento de la
hepatitis autoinmune. Al ser una enfermedad de origen autoinmune, utilizaremos fármacos con efecto
inmunosupresor, siendo los más comunes los que se describen en las opciones de respuesta 1, 2 y 4. Sin
embargo la lamivudina es un antiviral utilizado en la hepatitis B (opción 3 falsa, por lo que la marcamos).

-----------------------------------------------------------o--------------------------------------------------------------

Info Pregunta: 8a2da3f7-f393-48a3-b4ec-2083950456be

112. Un varón de 32 años es remitido para estudio diagnóstico al presentar en una


analítica de rutina, en el reconocimiento de su empresa, unas pruebas de función
hepática alteradas. Está asintomático y no toma medicamentos. Presenta durante más
de seis meses discreta hipertransaminasemia GOT 75, GPT 89, colesterol 221, el resto
de la analítica fue normal. Una correcta evaluación del paciente incluiría lo siguiente,
EXCEPTO:

1. Determinación de alfa-1 antitripsina.


2. CPRE.
3. Ecografía abdominal.
4. Serología virus hepatitis.

Resp. Correcta: 2

Comentario:

En este caso, la prueba diagnóstica que no tiene sentido es la CPRE. Date cuenta de que la analítica revela
cierto grado de citolisis, pero no de colestasis, por lo que resultaría muy improbable encontrarnos una
alteración morfológica de las vías biliares extrahepáticas.

-----------------------------------------------------------o--------------------------------------------------------------

Info Pregunta: 8a20fab9-a375-4cd1-8b3d-214f925c9519

113. Varón de 60 años con EPOC que ha ingresado por neumonía y reagudización
grave de la insuficiencia respiratoria, precisando tratamiento con esteroides e ingreso
en UCI. En el 5º día de ingreso, el paciente presenta episodio de hemorragia digestiva
alta. Lo más probable es que la endoscopia muestre:

1. Erosiones gástricas que afectan a cuerpo y fundus.


2. Erosiones gástricas en antro.
3. Úlcera duodenal.
4. Tumor gástrico.

Resp. Correcta: 1

Comentario:

Las gastritis agudas de estrés suelen expresarse clínicamente en forma de hemorragia digestiva, sea en forma
de hematemesis, melenas o anemización súbita. Aunque su localización puede ser difusa, predominan en
fundus. El mecanismo de la lesión es por isquemia y alteración de la barrera mucosa gástrica, como sucede
en las úlceras de Curling (grandes quemados) y en las relacionadas con otras causas. La excepción a esta
regla son las úlceras de Cushing. Éstas se relacionan con la hipertensión intracraneal, sobre todo cuando es
aguda, ya que produce una hiperestimulación vagal y, como resultado, hipersecreción ácida gástrica.

Debes recordar que, aunque se habla coloquialmente de “úlceras de estrés”, en realidad se trata de erosiones
(la profundidad de la lesión no sería, propiamente, una úlcera).

-----------------------------------------------------------o--------------------------------------------------------------

Info Pregunta: 436a53c2-719c-4c10-bbfe-21ba9fd89d74

114. ¿De cuál de los siguientes trastornos es característica la hipertrofia del lóbulo
caudado hepático?

1. Hepatitis crónica persistente.


2. Hepatitis crónica activa.
3. Cirrosis biliar primaria.
4. Síndrome de Budd-Chiari.

Resp. Correcta: 4

Comentario:

El síndrome de Budd- Chiari se debe a una trombosis de las venas suprahepáticas. Dado que el lóbulo
caudado drena directamente a cava inferior, a diferencia del resto del hígado, puede hipertrofiarse con el
paso del tiempo debido a la necrosis del resto del parénquima.

-----------------------------------------------------------o--------------------------------------------------------------

Info Pregunta: 743a3fb1-a03a-4a1f-86f7-21bd3b51ee3f

115. Un enfermo con cirrosis hepática, con ascitis importante, que está ingresado desde
ayer en el servicio de Digestivo, se encuentra en encefalopatía hepática grado 3. ¿Cuál
de las siguientes actitudes le parece INCORRECTA?

1. Realizar al enfermo un tacto rectal.


2. Realizar una paracentesis diagnóstica.
3. Incrementar la dosis de furosemida.
4. Iniciar un tratamiento con enemas cada 12 horas.

Resp. Correcta: 3

Comentario:

Los diuréticos del asa son una de las posibles causas que pueden descompensar la encefalopatía hepática.
Ten en cuenta que, desde el punto de vista del equilibrio ácido base, tienden a aumentar la pérdida urinaria
de potasio y cloro. Esto conlleva una alcalosis metabólica y, como consecuencia de ello, un incremento de la
amoniogénesis renal, lo que contribuye a empeorar la encefalopatía hepática.

El resto de las opciones son correctas. Ante un paciente con encefalopatía hepática, estaría indicado
descartar sangrado digestivo, para lo que puede ser útil el tacto rectal (respuesta 1). Otro factor que habría
que evitar es el estreñimiento, ya que es un posible desencadenante de la encefalopatía hepática, y para ello
tienen sentido los enemas y la paromomicina (aminoglucósido, que disminuirá la carga intestinal de
bacterias). Por último, la paracentesis diagnóstica también debería realizarse, ya que una peritonitis
bacteriana espontánea podría descompensar una encefalopatía hepática.

-----------------------------------------------------------o--------------------------------------------------------------

Info Pregunta: ba9bf24f-712d-4e85-abd0-222d74f76b80

116. El misoprostol está indicado en:

1. Prevención de lesiones agudas de la mucosa gástrica por AINE.


2. Tratamiento erradicador del Helicobacter pylori.
3. Prevención de cualquier tipo de úlcera duodenal.
4. En la profilaxis de úlceras de Curling.

Resp. Correcta: 1

Comentario:

El misoprostol es un fármaco análogo de prostaglandinas. Por tanto, se puede emplear en el tratamiento de la


gastropatía por AINEs, e incluso del ulcus provocado por los AINEs (que inhiben la acción protectora de la
mucosa gástrica que tienen las prostaglandinas). Sin embargo, aunque este fármaco es eficaz, son necesarias
3 ó 4 dosis al día, por lo que actualmente su uso es bastante escaso. Por ello, son mucho más utilizados los
inhibidores de la bomba de protones.

-----------------------------------------------------------o--------------------------------------------------------------

Info Pregunta: ed778f89-3e3d-406b-82ca-228b299ff7b7

117. Paciente ingresado por hematoma epidural secundario a traumatismo


craneoencefálico que presenta episodio de hemorragia digestiva alta, observándose en
la gastroscopia una úlcera única sangrante. Señale la afirmación CORRECTA respecto
a la complicación que Ud sospecha:

1. Se trata de una úlcera de Curling y está causada por la hipovolemia.


2. No es una úlcera verdadera sino una erosión de estrés.
3. El factor patogénico principal es la hipersecreción de ácido.
4. El hallazgo de una úlcera única descarta que se trate de una gastritis aguda de estrés.

Resp. Correcta: 3

Comentario: Las úlceras y erosiones agudas son provocadas por la hipoperfusión tisular, que ocurre en
enfermos en situación crítica, por sepsis, insuficiencia respiratoria o grandes quemados. En esta última
situación, se denominan úlceras de Curling. Cuando la causa es un traumatismo que afecta al sistema
nervioso central, más que por hipoperfusión lo que se produce es úlceras agudas por hiperacidez y reciben el
nombre de úlceras de Cushing. Las úlceras agudas o de estrés son múltiples, poco profundas y, por tanto,
causan poco dolor y se expresan clínicamente por sangrado múltiple. Suelen afectar a cuerpo y antro
gástrico.

-----------------------------------------------------------o--------------------------------------------------------------

Info Pregunta: 41f330ae-2253-409b-9cd0-22a02cd25ffc

118. Ante un paciente con un cuadro consistente en demencia, diarrea, oligoartritis


migratoria y miorritmia oculomasticatoria, ¿cuál de las siguientes exploraciones
realizaría en primer lugar?:

1. Biopsia cerebral.
2. Biopsia hepática.
3. Biopsia cutánea.
4. Biopsia del intestino delgado.

Resp. Correcta: 4

Comentario: La asociación clínica de diarrea, oligoartritis migratoria y alteraciones del sistema nervioso
central (demencia,miorritmia oculomasticatoria), deben hacernos sospechar como primera posibilidad
diagnóstica la enfermedad de Whipple. Por tanto, estaría indicada la biopsia de intestino delgado, porque
sería diagnóstica: se observarían macrófagos con inclusiones PAS+ en su interior, que corresponden a las
propias bacterias.

-----------------------------------------------------------o--------------------------------------------------------------

Info Pregunta: e7a8f323-0589-4b3d-9425-22b8499ade43

119. El estómago contribuye al control del peso corporal a través de:

1. Síntesis de grelina en el periodo postprandial


2. Síntesis de leptina en el periodo de ayuno
3. La síntesis de leptina en el periodo postprandial
4. La síntesis de grelina y leptina en el periodo de ayuno

Resp. Correcta: 3

Comentario: El estómago libera grelina durante el ayuno y leptina durante el periodo postprandial regulando
así la ingesta alimentaria y, por lo tanto, el peso corporal

-----------------------------------------------------------o--------------------------------------------------------------

Info Pregunta: 5d26508e-e8a2-477e-b83e-236068cc58ea

120. Mujer de 22 años con diagnóstico previo de epilepsia desde hacía tres meses y
amigdalectomizada, que acude nuevamente por cuadro de tos productiva desde hace 15
días. En la exploración física se observa una aparente disminución del murmullo
vesicular en base pulmonar derecha. Se realizó una Rx de tórax en la que se comprobó
la existencia de una neumonía basal derecha, instaurándose tratamiento antibiótico,
antipirético y mucolítico. La analítica solicitada mostró los siguientes resultados: Hb
13,2 g/dL, leucocitos 18.000/mm³ con aumento de neutrófilos y eosinófilos, plaquetas
380.000/mm³, glucosa 98 mg/dL, urea 50 mg/dL, creatinina 1,1 mg/dL, GOT 220 U/L,
GPT 352 U/L, GGT 538 U/L, bilirrubina total 1 mg/dL, fosfatasa alcalina 45 U/L, LDH
102 U/L, amilasa 60 U/mL, aumento de inmunoglobulina E total, proteinograma
normal. A raíz de estos resultados, se amplió el estudio. La serología viral y el estudio
de autoinmunidad fueron negativos. Señale, de entre los siguientes, cuál le parece el
que con MAYOR probabilidad podría ser responsable del cuadro de la paciente:

1. Paracetamol.
2. Fenitoína.
3. Eritromicina.
4. Codeína.

Resp. Correcta: 2

Comentario:

La fenitoína se puede asociar a cuadro de hepatitis inmunoalérgica (fíjate en que los eosinófilos y la IgE
están elevados en este cuadro de hepatitis - respuesta 2 correcta -). Veamos las demás opciones de respuesta:
1.- El paracetamol daría un cuadro de citólisis pura.
3.- La eritromicina daría un cuadro de colestasis.
4.- La Codeína muy raramente también daría un cuadro de colestasis.

-----------------------------------------------------------o--------------------------------------------------------------

Info Pregunta: fa775f16-ec74-47e8-a804-24a359c03842

121. La mayoría de las reacciones adversas de la sulfasalazina se deben a:

1. El ácido 4-aminosalicílico.
2. La sulfapiridina.
3. La mesalazina.
4. La olsalazina.

Resp. Correcta: 2

Comentario: La sulfasalazina pertenece al grupo de los salicilatos, que se emplean en los brotes leves-
moderados de la colitis ulcerosa y de la enfermedad de Crohn. Aunque se usan principalmente en el
tratamiento de mantenimiento de estas enfermedades. Las reacciones adversas provocadas por la
sulfasalazina se deben fundamentalmente a la molécula derivada de las sulfamidas, la sulfapiridina, que
puede desencadenar reacciones alérgicas importantes.

-----------------------------------------------------------o--------------------------------------------------------------

Info Pregunta: d3e3c832-b0e3-4ad8-aeac-2507efbe4b9b

122. Durante tu guardia en urgencias, atiendes a Carmen, una joven de 23 años que
acude por presentar un cuadro agudo de diarrea, con expulsión de moco y sangre,
además de dolor abdominal. La exploración física de la zona anal y perianal es
anodina. Le solicitas una analítica básica cuyos resultados son: leucocitosis con
desviación izquierda, anemia ferropénica, VSG y PCR elevadas e hipopotasemia. La
sospecha diagnóstica inicial es de una colitis ulcerosa. Respecto a esta entidad, señale la
respuesta falsa:
1. Las principales manifestaciones clínicas son la diarrea, la emisión de sangre, moco o pus por vía rectal
y el dolor abdominal.
2. La CU parece exhibir un perfil de citocinas tipo Th2, aunque existe un solapamiento de citocinas con
la enfermedad de Crohn
3. En muchos de los casos, el recto no está afecto.
4. La inflamación recurrente de la mucosa puede dar lugar a la aparición de cambios permanentes
característicos como fibrosis, desaparición de haustras y retracción longitudinal de colon, produciendo
un aspecto característico en cañería de plomo.

Resp. Correcta: 3

Comentario:

La colitis ulcerosa es una enfermedad que afecta fundamentalmente a la mucosa de colon, con participación
casi constante de la mucosa rectal y una extensión proximal más variable. Por esto, dado que en más de un
95% de las ocasiones el recto sí se ve afecto, la respuesta falsa entre las opciones es la opción 3.

-----------------------------------------------------------o--------------------------------------------------------------

Info Pregunta: c21bcd14-dc9e-4376-9906-2591c6026a83

123. Las úlceras gástricas que tienen predilección por la zona de transición del antro al
cuerpo con producción baja de ácido, se conocen como:

1. Tipo II de Johnson.
2. Tipo I de Johnson.
3. Tipo V de Johnson.
4. Tipo IV de Johnson.

Resp. Correcta: 2

Comentario:

De las úlceras gástricas, las que tienen predilección por la zona de transición del antro al cuerpo y que
presentan una producción de ácido baja, se denominan tipo I de Johnson (respuesta 2 correcta).
Distinguiéndose de otros tipos de úlceras gástricas que cursan con hipersecreción ácida, parecidas en su
fisiopatología a las duodenales:
- Si se acompañan de una úlcera duodenal se denominan tipo II.
- Si la úlcera gástrica es prepilórica se denominan tipo III.
Las úlceras tipo IV, con hiposecreción de ácido (como las de tipo I), se sitúan en la curvatura menor,
cercanas a la unión gastroesofágica.

-----------------------------------------------------------o--------------------------------------------------------------

Info Pregunta: 6250d3dc-0667-4c15-98f8-25a1dca7d3f3

124. Acude a tu consulta una mujer de 55 años por ardor retroesternal y sensación de
que la comida "se le vuelve a la boca", lleva aproximadamente 1 mes con los síntomas.
En la exploración no hay datos que llamen la atención, excepto obesidad.
Recomendarías:

1. Tratamiento con omeprazol 4 semanas


2. pHmetría de 24 horas.
3. Cirugía porque la clínica es muy sugerente de hernia de hiato.
4. Dieta hipocalórica.

Resp. Correcta: 1

Comentario: La presencia de pirosis y regurgitación ácida es altamente sugestiva de enfermedad por reflujo
gastroesofágico ácido y es suficiente como para instaurar directamente tratamiento con omeprazol u otro
inhibidor de la bomba de protones; si hubiera remisión de los síntomas quedaría además confirmado el
diagnóstico de esta enfermedad. La endoscopia se indicaría en caso de otros síntomas asociados o falta de
respuesta al tratamiento.

-----------------------------------------------------------o--------------------------------------------------------------

Info Pregunta: 3074161f-2877-4776-9c9a-25d4eeb5350b

125. ¿Cual de los siguientes fármacos utilizados en la enfermedad inflamatoria


intestinal crónica no se corresponde con un anti factor de necrosis tumor al alfa?.

1. Infliximab
2. Vedolizumab
3. Golimumab
4. Adalimumab

Resp. Correcta: 2

Comentario: El Vedolizumab es un anticuerpo anti-integrina alfa4-beta7. Los demás si son anti-TNF.

-----------------------------------------------------------o--------------------------------------------------------------

Info Pregunta: a3095cde-5bd0-4333-8e40-261a3427afaf

126. ¿Cuál de los siguientes enunciados es VERDADERO?

1. Maldigestión: déficit de las enzimas responsables de la hidrólisis de los principios activos.


2. Malabsorción: déficit de las enzimas responsables de la hidrólisis de los principios activos.
3. Maldigestión: daño del epitelio absortivo intestinal.
4. Malabsorción: exceso de grasa en las heces.

Resp. Correcta: 1

Comentario:

La maldigestión aparece cuando las enzimas responsables de la hidrólisis de los principios activos son
deficitarias, como en la insuficiencia pancreática. La malabsorción traduce un daño del epitelio absortivo
intestinal, es decir un problema a nivel de los enterocitos, como en la enfermedad de Whipple.

-----------------------------------------------------------o--------------------------------------------------------------

Info Pregunta: 55e5d48f-99f6-4de8-8fda-26974f4d3b65

127. Todas las siguientes afirmaciones corresponden a la cirrosis biliar primaria,


EXCEPTO:
1. La presencia de estigmas cirróticos típicos es más frecuente en este cuadro que en otras cirrosis.
2. La presencia de anticuerpos antimitocondriales es un criterio diagnóstico de cirrosis biliar primaria.
3. La detección de la enfermedad suele realizarse al estudiar el hallazgo de unas fosfatasas alcalinas
elevadas.
4. La presencia del anillo corneal de Kayser-Fleisher no nos descarta esta entidad.

Resp. Correcta: 1

Comentario: La cirrosis biliar primaria se diagnostica frecuentemente en su forma asintomática cuando se


observa solamente una elevación de la fosfatasa alcalina. Los anticuerpos antimitocondriales apoyan el
diagnóstico, ya que están elevados en más del 90% de los casos. Se puede encontrar, aunque es muy raro, un
anillo de Kayser- Fleisher. Los estigmas cirróticos no aparecen con mayor frecuencia en esta enfermedad.

-----------------------------------------------------------o--------------------------------------------------------------

Info Pregunta: 3b04153e-c76a-498d-a9b9-26bc5819dd09

128. ¿Qué no esperaría encontrar en una diarrea secretora?

1. volumen fecal <1.000ml/día


2. heces isotónicas
3. persistencia con el ayuno
4. acidosis hipopotasémica

Resp. Correcta: 1

Comentario: La diarrea secretora se produce por una alteración en el transporte de iones y agua a través del
epitelio intestinal, produciéndose un aumento de secreción de iones y con ello de la secreción de agua a la
luz. El volumen fecal diario es >1.000ml/día.

-----------------------------------------------------------o--------------------------------------------------------------

Info Pregunta: d5c91a67-141c-4fb7-879b-27271e08ea2c

129. Un varón de 40 años es estudiado por presentar en un análisis rutinario una


anemia ferropénica. Tras realizársele una colonoscopia se demuestra un
adenocarcinoma de colon a nivel del ángulo hepático, siendo el resto del colon normal.
Analizando los antecedentes familiares se encuentra que su abuelo paterno murió de
un cáncer de colon a los 50 años; su padre y dos hermanos de su padre también
murieron de cáncer de colon. Su mujer y 3 hijos de 15, 12 y 6 años están sanos. ¿Cuál
de las siguientes consideraciones es correcta?

1. Hay que investigar sangre oculta en heces anualmente en los tres hijos del paciente.
2. Se debe hacer colonoscopia anual a los dos hijos mayores y al pequeño a partir de los doce años.
3. Se debe hacer sigmoidoscopia cada cinco años a los hijos.
4. Se debe hacer colonoscopia cada 3 años a los hijos a partir de los 25.

Resp. Correcta: 4

Comentario:

En los pacientes que cumplen 2 de los criterios de Amsterdam del Síndrome de cáncer colorrectal
hereditario no asociado a poliposis (CCHNP): 3 ó más familiares afectos de cáncer ce colon (al menos uno
que sea familiar de primer grado de los otros 2), uno o más casos en la familia de cáncer colorrectal antes de
los 50 años y afectar al menos a dos generaciones consecutivas, deberá realizarse en los familiares una
colonoscopia periódica (cada 2- 3 años) a partir de los 25 años, o 5 años antes del familiar afecto más joven,
para el diagnóstico precoz de un eventual cáncer de colon.

-----------------------------------------------------------o--------------------------------------------------------------

Info Pregunta: 7ac52e7f-d1f8-4751-89df-278429136230

130. Paciente de 45 años cirrótico por hepatitis C con ascitis a tensión. Cuál es el
tratamiento de elección?

1. Espironolactona
2. Espironolactona + furosemida
3. Furosemida
4. Paracentesis evacuadora

Resp. Correcta: 4

Comentario: El tratamiento de elección de una ascitis a tensión es la paracentesis evacuada con reposición
de albúmina ya que no se puede esperar tanto al efecto de los diuréticos en esta situación.

-----------------------------------------------------------o--------------------------------------------------------------

Info Pregunta: 7bc17535-379e-4550-88f9-279556d18e42

131. En el paciente con úlcera péptica podemos emplear las siguientes estrategias,
EXCEPTO:

1. Utilizar omeprazol o pantoprazol que inhiben la bomba K/H.


2. Utilizar cimetidina o ranitidina que son anti-H2.
3. El hidróxido de aluminio tiene acción antisecretora.
4. El misoprostol tiene acción PG-like.

Resp. Correcta: 3

Comentario: El hidróxido de aluminio (al igual que el hidróxido de magnesio), no tiene actividad
antisecretora. Es un fármacos con efecto antiácido, que sólo puede emplearse como tratamiento sintomático
de la pirosis (generalmente en la enfermedad por reflujo esofágico). Tanto los inhibidores de la bomba de
protones, como los anti- H2 tienen acción antisecretora y, por tanto, se usan en el tratamiento de la úlcera
péptica. El misoprostol, al ser un análogo de prostaglandinas, se puede emplear en el tratamiento del ulcus
provocado por los AINEs (que inhiben la acción protectora de la mucosa gástrica que tienen las
prostaglandinas).

-----------------------------------------------------------o--------------------------------------------------------------

Info Pregunta: 8d60b358-600a-48c0-bce5-27dcaac8ab90

132. Varón de 36 años que está en seguimiento en la Unidad de Hepatología por


cirrosis alcohólica. En la actualidad y, a pesar de haber dejado el alcohol hace cinco
años, la enfermedad ha evolucionado, encontrándose en estos momentos en un estadio
funcional C de Child, con varices esofágicas sangrantes que han precisado esclerosis
endoscópica en cuatro ocasiones. Ante esta situación, ¿cuál es la MEJOR opción
terapéutica para este paciente?

1. Shunt porto-cava quirúrgico.


2. Propranolol.
3. Trasplante hepático.
4. Shunt intrahepático percutáneo.

Resp. Correcta: 3

Comentario:

El paciente presenta un importante deterioro de la función hepática (estadio C de Child) y ha tenido varios
sangrados procedentes de varices esofágicas, por lo que debemos asumir que padece una importante
hipertensión portal; por lo tanto, la mejor solución ante una situación tan grave sería el trasplante hepático
(respuesta 3 correcta). El hábito enólico activo podría ser un impedimento, sin embargo, nos dicen que el
paciente ha dejado de beber hace cinco años, por lo que no sería contraindicación.

-----------------------------------------------------------o--------------------------------------------------------------

Info Pregunta: 2f515bc3-d1f3-47d6-8a56-28d1ab4eac02

133. En un paciente con enfermedad inflamatoria intestinal (EII) orientará al


diagnóstico de enfermedad de Crohn frente al de colitis ulcerosa:

1. Afectación perianal.
2. Afectación colónica y de íleon terminal.
3. Afectación continua de la mucosa colónica.
4. Mucosa colónica granular con úlceras y hemorragias.

Resp. Correcta: 1

Comentario:

De entre las opciones de respuesta, lo más sugestivo de la enfermedad de Crohn es la afectación perianal
(respuesta 1 correcta), que suele ser en forma de fístulas. Las demás opciones de respuesta son sugestivas de
colitis ulcerosa. La opción de respuesta 2 puede plantear dudas, ya que la colitis ulcerosa habitualmente no
afecta a íleon terminal, pero en casos graves puede hacerlo; en cambio, la aparición de afectación perianal en
la colitis ulcerosa sería insólita.

-----------------------------------------------------------o--------------------------------------------------------------

Info Pregunta: 51ed46ca-abd8-461b-8593-29abf154bd23

134. Un paciente de 50 años, bebedor importante, acude a Urgencias por dolor


abdominal y vómitos. El dolor lo describe como "en cinturón", irradiado hacia la
espalda, empeora en decúbito supino y mejora un poco al inclinarse hacia delante. Los
datos analíticos iniciales son: GOT 80 U/L, GTP 55 U/L, leucocitos 16.000, triglicéridos
670, amilasa normal. No hay signos de irritación peritoneal. En relación con este caso,
señale la afirmación CORRECTA:

1. La normalidad de la amilasemia descarta el diagnóstico de pancreatitis, ya que este exige que al


menos triplique el límite superior de la normalidad.
2. Aunque no conocemos las cifras de lipasa, es improbable que esté elevada.
3. Debería iniciarse tratamiento con fluidoterapia y analgésicos.
4. La edad no es un factor con importancia pronóstica en la valoración inicial de este tipo de cuadros.

Resp. Correcta: 3

Comentario:

La clave de esta pregunta está en recordar que la clínica tiene mucho más valor que los datos analíticos. En
un paciente bebedor importante y con un cuadro clínico típico de pancreatitis aguda, una amilasemia normal
no descarta el diagnóstico (tiene hipertrigliceridemia significativa que se asocia típicamente a niveles de
amilasa bajos). Ante esto, se debe tratar como una pancreatitis con dieta absoluta, fluidoterapia y analgesia.
Este hecho de la amilasa normal puede solventarse diluyendo el suero y/o solicitando niveles de lipasa, que
son más específicos. La edad sí es un factor pronóstico y, de hecho, está incluida en los criterios de Ransom
al ingreso.

-----------------------------------------------------------o--------------------------------------------------------------

Info Pregunta: 8b022051-6d5e-4399-af97-2a92f8131711

135. Nos encontramos ante un varón de 60 años diagnosticado de cirrosis hepática con
serología positiva para el virus C y ascitis que no responde al tratamiento con 400 mg
de espironolactona con restricción de sal (menos de 800 mg de sodio) y reposo. ¿Cuál
debe ser nuestra actitud si queremos controlar la ascitis?

1. Estableceremos un cortocircuito portocava laterolateral.


2. Dispondremos un shunt peritoneovenoso, conscientes de las complicaciones infecciosas y de
coagulación que puede tener el enfermo.
3. Duplicaremos la dosis de espironolactona con especial atención al nivel de potasio en sangre.
4. Añadiremos al tratamiento furosemida de 20 a 80 mg al día.

Resp. Correcta: 4

Comentario:

En el tratamiento escalonado de la ascitis, se debe iniciar con restricción de sodio y reposo. A continuación,
se deben añadir diuréticos, aumentando progresivamente la dosis hasta llegar al máximo permitido. Aunque
la espironolactona es el diurético de elección, su inicio de acción es lento, y tiene potencia diurética baja;
por ello, se debe asociar furosemida, aumentando la dosis progresivamente hasta llegar al máximo tolerado o
hasta 160 mg al día.

-----------------------------------------------------------o--------------------------------------------------------------

Info Pregunta: 85bb4033-f503-441a-a4b8-2ab14931eaa4

136. Mujer de 37 años diagnosticada de un síndrome de Sjögren que acude a la


consulta de Reumatología para revisión. Aporta los resultados de una analítica
efectuada dos semanas antes de la consulta: Hb 12,6 g/dL, 4.800 leucocitos/mm3 con
fórmula normal, plaquetas 207.000/mm3, Cr 1, urea 58, GOT 45, GPT 51, Bilirrubina
0,9, GGT 28, FA 791. La paciente no refiere ninguna nueva sintomatología más allá de
la debida a su proceso de base. Ante la posibilidad de una colangitis biliar primaria, se
solicita una determinación de AMA (Ac antimitocondriales), que resulta negativa.
Señale la CIERTA:

1. Generalmente, es necesario la obtención de una biopsia hepática previa al tratamiento de una CBP.
2. La negatividad de los anticuerpos antimitocondriales descarta el diagnóstico de una colangitis biliar
primaria.
3. Los anticuerpos sp100 y gp210 son útiles en el diagnóstico de esta enfermedad.
4. El ácido ursodesoxicólico mejora la sintomatología, pero no ha demostrado un aumento en la
supervivencia de los pacientes con CBP.

Resp. Correcta: 3

Comentario:

Ante una elevación pronunciada de la fosfatasa alcalina en el contexto de una mujer joven diagnosticada
previamente de un trastorno autoinmune, debemos considerar la posibilidad de que estemos ante una CBP,
aún más si el trastorno autoinmune es un síndrome de Sjögren, que se asocia a la enfermedad que nos ocupa
en un 70 % de los casos aproximadamente. Más de la mitad de los diagnósticos se efectúan en personas
asintomáticas con una alteración analítica como las que nos presenta el caso. Los AMA son muy sensibles y
específicos y se encuentran en la mayor parte de los pacientes, pero su negatividad no descarta el
diagnóstico (respuesta 2 falsa). En estos casos, a veces aparecen otros también muy específicos: los sp100 y
los gp210 (respuesta 3 correcta). La presencia de alguno de estos anticuerpos junto con un patrón de
colestasis se considera diagnóstico, no siendo necesaria la biopsia salvo en caso de negatividad de los
mismos (respuesta 1 falsa). Por último, el ácido ursodesoxicólico SI prolonga la supervivencia.

-----------------------------------------------------------o--------------------------------------------------------------

Info Pregunta: e9c30d80-ffbb-49aa-aefa-2abfdb3f4ab6

137. En una radiografía de abdomen en supino se ve el ligamento falciforme. ¿Cuál es


el diagnóstico más probable?:

1. Perforación intestinal.
2. Peritonitis.
3. Oclusión intestinal.
4. Ileo paralítico.

Resp. Correcta: 1

Comentario:

El ligamento falciforme solamente puede ser visible si se contrasta por la presencia de aire en la cavidad
peritoneal, que cuando el paciente está en supino se dispone en la region anterior del abdomen.

Por tanto el neumoperitoneo necesario para que se haga patente el ligamento se produce por la perforación
de viscera hueca (respuesta 1).

-----------------------------------------------------------o--------------------------------------------------------------

Info Pregunta: fafeb4df-6371-4abc-aa77-2ad2e48b4a8d

138. Paciente de 52 años que es ingresada en el hospital por un episodio de pancreatitis


aguda litiásica. Después del episodio agudo, cuando ya está asintomática, se le realiza
una ecografía abdominal donde se objetiva una imagen sugestiva de pseudoquiste
pancreático. Se decide observarlo con controles ecográficos periódicos. A los tres meses
no ha desaparecido e incluso ha aumentado ligeramente de tamaño (actualmente, 7
cm). Señale la afirmación CORRECTA:

1. El pseudoquiste es la complicación más frecuente de la pancreatitis aguda.


2. Dado que no se ha resuelto espontáneamente, precisará una intervención de Whipple.
3. Al haberse resuelto la pancreatitis aguda, lo más probable es que las cifras de amilasa se hayan
normalizado.
4. La causa más frecuente de pseudoquiste es la pancreatitis aguda.

Resp. Correcta: 1

Comentario:

El pseudoquiste es la complicación más frecuente de la pancreatitis aguda, si bien su causa más frecuente es
la pancreatitis crónica. Antiguamente se consideraba que un pseudoquiste > 6 cm que no se hubiera resuelto
en 6 semanas debía intervenirse mediante drenaje o escisión quirúrgica dependiendo de la localización; sin
embargo, hoy día se considera más importante la repercusión clínica, de tal manera que, si no existen
síntomas, no existe indicación de tratamiento.

-----------------------------------------------------------o--------------------------------------------------------------

Info Pregunta: 8b557fba-b7f6-4000-8f0a-2ada7744ec5c

139. La hepatitis aguda grave, asociada en ocasiones a la enfermedad de Wilson, se


puede distinguir de otras hepatitis graves por:

1. Mayor intensidad de la encefalopatía.


2. Presencia de temblores asociados cuando se trata de pacientes pediátricos.
3. Descenso de la tasa de protrombina.
4. Presencia de hemólisis aguda.

Resp. Correcta: 4

Comentario:

En la enfermedad de Wilson la ausencia de ceruloplasmina conduce a un descenso del cobre total en sangre;
sin embargo, aumenta su cantidad en estado libre (no ligado a proteínas). Este aumento del cobre libre
produce toxicidad para las membranas de los hematíes y es por ello por lo que en la enfermedad de Wilson
es característica la asociación de hepatitis con hemólisis aguda (respuesta 4 correcta).

-----------------------------------------------------------o--------------------------------------------------------------

Info Pregunta: 3bed22b7-247c-4b31-9a72-2b5770ce6f8d

140. Juan Pedro acude a la consulta acompañado de sus dos hijos. Nos comenta que
Pau, de 12 años, ha sido diagnosticado recientemente de celiaquía. El motivo de la
consulta es que quiere saber si su otro hijo, Salvador, padece la enfermedad. Esto es
debido a que una vecina que también la padece le ha comentado que se suelen hacer
pruebas a los familiares cercanos. Salvador, de 10 años, no refiere ningún tipo de signo
o síntoma de tal enfermedad. Como antecedente de interés, el niño padece Síndrome de
Down, y fue operado de atresia duodenal a los pocos días de nacer. Ante dicho caso,
¿cuál sería el paso que cree usted MÁS ADECUADO?

1. Tranquilizar al padre explicándole que mientras su hijo no presente síntomas de la enfermedad, no es


necesario realizar ningún tipo de prueba diagnóstica.
2. Solicitar una analítica con anticuerpos antitransglutaminasa IgA junto con recuento total de IgA.
3. Solicitar bioquímica con perfil hepático y hemograma en busca de signos de la enfermedad, y actuar
en función de los resultados.
4. Solicitar tipaje HLA DQ2-DQ8.

Resp. Correcta: 4

Comentario:

Nos encontramos ante un caso clínico en el que se nos pide realizar un correcto screening familiar de la
enfermedad celíaca. En este caso, uno de los hijos presentan la enfermedad, mientras que el otro hermano se
encuentra aparentemente sano. Por el hecho de ser hermano biológico de un paciente con esta enfermedad,
ya se le debe hacer screening, más aún si presenta Síndrome de Down. Por ello, las respuestas 1 y 3 son
falsas (la utilidad del screening es justamente, diagnosticar la enfermedad en personas asintomáticas). La
duda lógica viene entre las respuestas 2 y 4. En esta ocasión la correcta es la RESPUESTA 4: lo primero en
realizarse es el tipaje del HLA, si este resultara positivo, entonces pediríamos los Anticuerpos. Si estos
también fueran positivos, y el paciente siguiera asintomático, pediríamos la biopsia como prueba final.

-----------------------------------------------------------o--------------------------------------------------------------

Info Pregunta: fc53a258-bd23-4914-94f7-2b8df73ed034

141. Un hombre de 60 años de edad, con colitis ulcerosa extensa de 15 años de


evolución y en remisión clínica los últimos 3 años, acude a nuestra consulta para
informarse sobre el riesgo de cáncer colorrectal y sobre la posibilidad de participar en
programas de prevención. Es correcto informarle que:

1. La colitis ulcerosa solo se asocia a un incremento en el riesgo de cáncer colorrectal en fumadores.


2. En su caso, dado que está en remisión de larga duración, se considera adecuado el cribado aconsejado
en la población general.
3. En casos como el suyo, se considera adecuado someterse a colonoscopias periódicas con toma de
múltiples biopsias escalonadas a lo largo de todo el colon.
4. En casos como el suyo, se considera adecuado el cribado mediante estudios periódicos de sangre
oculta en heces, pero con una frecuencia superior a la utilizada en el cribado de la población general.

Resp. Correcta: 3

Comentario: Existe un incremento del riesgo de neoplasia colorrectal en los pacientes con enfermedad
inflamatoria intestinal (EII) que afecta al colon (especialmente en la enfermedad extensa) Este incremento
del riesgo produce que no sirvan los programas de cribado poblacionales estándar en estos pacientes. El
cribado en la EII, se realiza mediante colonoscopia, tomando biopsias al azar tanto de segmentos sanos
como de zonas de mucosa anormales (3-4 biopsias cada 10cm desde el ciego al recto) en busca de displasia
o neoplasia.

-----------------------------------------------------------o--------------------------------------------------------------

Info Pregunta: c86d5921-b4b7-4a80-8098-2b9293b7c580


142. En la hemocromatosis idiopática primaria NO es característico encontrar:

1. Disminución de la ferritina sérica.


2. Elevación de la sideremia.
3. Hiperglucemia.
4. Elevación de los depósitos de hierro en el hígado.

Resp. Correcta: 1

Comentario:

Recuerda que un hallazgo característico de la hemocromatosis primaria es la elevación de la ferritina por


aumento de los depósitos de hierro. Todo lo contrario a la opción 1. De hecho, las opciones de respuesta 1 y
4 se contradicen, ya que la ferritina es un fiel reflejo de los depósitos hepáticos de hierro.

-----------------------------------------------------------o--------------------------------------------------------------

Info Pregunta: cd9d68f7-7916-402c-ada1-2bf2b50fe86f

143. Varón de 56 años que acude por crisis de diarrea intermitentes de hasta 12
deposiciones al día líquidas sin productos patológicos. No refería consumo de
productos de herbolario, tóxicos ni viajes recientes. Sensación distérmica pero cuando
se media nunca demostró la existencia de fiebre. La exploración física fue anodina. La
analítica de sangre mostró los siguientes resultados: Hb 11.1 g/dl, VCM 128 fl,
leucocitos 10100/dl, plaquetas 440.000, actividad de protrombina 90%, glucosa 105
mg/dl, urea 17, creatinina 0.7, GOT 30, GPT 38, bilirrubina 1.1 mg/dl, GGT 28,
fosfatasa alcalina 100 UI/ml,, amilasa 50, proteínas totales 8 g/dl, albúmina 4.2 mg/dl,
LDH 185 U/L, sodio 140, mEq/L potasio 4 mEq/L, proteína C reactiva 1.5 mg/dl. Se
realizó una sangre oculta en heces que resulto negativa. La determinación de grasa en
heces de 24 horas fue de 10 g. El test de D-xilosa fue ligeramente bajo en la orina (4
gramos a las 6 horas). ¿Qué exploración solicitaría a continuación en aras de llegar al
diagnóstico más probable?:

1. Test de Pancreatolauryl
2. Test de lactulosa-C*
3. Gastroscopia con biopsia de 2ª porción duodenal
4. Colonoscopia con biopsias seriadas de colon

Resp. Correcta: 2

Comentario: Se presenta el caso de un paciente con diarrea crónica y anemia macrocítica. El test de grasa en
heces fue patológico lo que apuntaría a descartar una posible malabsorción/maldigestión. El hecho de que el
test de D-xilosa sea patológico descartaría una insuficiencia pancreática exocrina. En esta situación lo más
adecuado a continuación sería descartar un sobrecrecimiento bacteriano con el test de Xilosa-C*/lactulosa-
C* en primer lugar. La macrocitosis apuntaría más a este diagnóstico que a una enfermedad celíaca. En
menos probable la existencia de una colitis microscópica que requeriría la toma de biopsias seriadas de
colon.

-----------------------------------------------------------o--------------------------------------------------------------

Info Pregunta: 05ab9eb8-8df8-4346-b769-2bff6dc98dcf


144. Paciente de 18 años, con retraso mental severo, que ingiere accidentalmente lejía.
Es traído a urgencias con grave deterioro del estado general, dolor intenso, disfagia
total y sialorrea, pero sin signos de perforación esofágica. Se le pone inmediatamente
tratamiento, que consiste en: dieta absoluta, sueros iv, analgesia y antibióticos. ¿Cuál
es lo siguiente que deberíamos hacer?:

1. Administrar eméticos.
2. Esofagectomía total + esofagostomía cervical + yeyunostomía.
3. Realizar endoscopia digestiva alta urgente.
4. Tránsito esofagogastroduodenal urgente.

Resp. Correcta: 3

Comentario:

La endoscopia es una prueba diagnóstica de interés en la esofagitis por cáusticos, pues aporta información
acerca de la extensión, severidad y localización de las lesiones. Debe realizarse precozmente, en las
primeras 24 horas si es posible, salvo que se sospeche neumomediastino, en cuyo caso está contraindicada.
Por ello, antes de realizar la endoscopia, es importante descartar perforación esofágica mediante una
radiografía de tórax.

El estudio baritado sólo es de interés en el estudio de las estenosis tardías y está contraindicado en el
momento agudo, sobre todo si hay sospecha de perforación. Está contraindicado el vómito inducido con
eméticos o el lavado gástrico.

-----------------------------------------------------------o--------------------------------------------------------------

Info Pregunta: 6978e445-d83d-46f5-b203-2ce8660d8ff2

145. Varón de 28 años que le es remitido para consejo en consulta por haber
presentado un cáncer de colon T1M0N0 en sigma a los 25 años con marcada presencia
de reacción Crohn-like en la pieza de sigmoidectomía. Asimismo se objetiva en la pieza
quirúrgica alta inestabilidad de microsatélites (IMS). ¿Cuál sería la siguiente actuación
que realizaría?:

1. Ampliar la resección con colectomía total


2. Sangre oculta en heces cada 2 años
3. Solicitar estudio de genes reparadores del DNA (MMR)
4. Colonoscopias anuales al paciente y familiares

Resp. Correcta: 3

Comentario:

Ante un paciente que cumple criterios de Bethesda (cáncer de colon en paciente menor de 50 años y con
histología característica de Síndrome de Lynch como es la reacción tipo Crohn-like). Estos pacientes si
presentan inestabilidad de microsatélites, se recomienda estudio en sangre de mutaciones de genes
reparadores de DNA con el objeto de posteriormente estudiar a todos los familiares en caso de no existir o
bien si hay mutaciones, estudiar genéticamente a los familiares con objeto de cribar específicamente a los
familiares afectos.

-----------------------------------------------------------o--------------------------------------------------------------
Info Pregunta: 36397506-8ae3-4c6f-a193-2cf25f96f2ce

146. Con respecto al cáncer colorrectal hereditario no asociado a poliposis es cierto


que:

1. Es más frecuente encontrar tumores sincrónicos que en el cáncer colorrectal esporádico.


2. Es más frecuente encontrar tumores metacrónicos que en el cáncer colorrectal esporádico.
3. Es más frecuente en el colon derecho.
4. Todas son ciertas.

Resp. Correcta: 4

Comentario: El síndrome de cancer colorrectal no asociado a poliposis (CCHNP), tradicionalmente llamado


Síndrome de Lynch se caracteriza por la presencia de tumores colónicos en pacientes menores de 40 años y
sobre todo en el colon derecho. Asimismo se asocia a la presencia tanto de tumores metacrónicos como
sincrónicos en otros lugares de la anatomía como endometriales, ováricos, tumores uroteliales, gástricos, etc.
Manual CTO.

-----------------------------------------------------------o--------------------------------------------------------------

Info Pregunta: b83805ec-df36-4ac6-a713-2d43a67dc560

147. Varón de 62 años sin antecedentes de interés que acude a urgencias por
deposiciones negras de 3 días de evolución y sensación de mareo. La analítica
practicada en urgencias demostró: Hb 6.3 g/dl, VCM 89 fl, 9500 leucocitos con 58% de
neutrófilos, plaquetas 275.000/mm3, INR 0.7, bilirrubina 0.8, AST 32, ALT 45, GGT
53, amilasa 34 UI/ml, glucemia 98, urea 123, creatinina 1.4, LDH 208, sodio 143,
potasio 3.8. Señale la causa menos probable del cuadro del paciente:

1. Desgarro esofágico por Mallory-Weiss.


2. Síndrome de Boerhaave.
3. Hemorragia por ulcera gástrica.
4. Hemorragia por varices esofágicas.

Resp. Correcta: 2

Comentario:

La causa más frecuente de hemorragia digestiva en población general es la ulcera péptica duodenal. De entre
las que se citan el Síndrome de Boerhaave, sería la causa menos frecuente.

-----------------------------------------------------------o--------------------------------------------------------------

Info Pregunta: 9641c331-1ad8-449d-9791-2d963ed990f0

148. ¿Cuál de las siguientes afirmaciones NO es correcta en relación a la metaplasia


columnar del esófago o metaplasia de Barrett?:

1. Su origen es adquirido como consecuencia del reflujo gastroesofágico crónico.


2. Se estima que el 0.5% de los pacientes con esófago de Barret desarrollan anualmente un
adenocarcinoma.
3. En la actualidad el mejor marcador de potencial malignidad sigue siendo la detección de displasia en
la biopsia.
4. La cirugía anti-reflujo logra controlar el reflujo gastroesofágico y hace desapaecer completamente la
metaplasia columnar.

Resp. Correcta: 4

Comentario:

Una pregunta sencilla en relación con el esófago de Barrett.

Conceptualmente, el esófago de Barrett es una metaplasia, es decir, la presencia de un tipo de epitelio en un


órgano donde, en principio, no correspondería encontrarlo. Como dice la respuesta 1, es consecuencia del
reflujo gastroesofágico crónico, ya que el epitelio esofágico normal, que es escamoso, no soporta bien la
exposición al medio ácido gástrico. Por este motivo, se produce una metaplasia columnar, que se adapta
mejor al mismo.

En ocasiones, la metaplasia de Barrett se complica, además, con displasia. Cuanto más severa es ésta, mayor
es la probabilidad de malignización, apareciendo sobre ella un adenocarcinoma de esófago.
Afortunadamente, esto ocurre en menos del 1% de los casos, pero debemos realizar endoscopias periódicas
para vigilar esta posible evolución, extirpando el esófago profilácticamente cuando encontramos displasia
severa.

-----------------------------------------------------------o--------------------------------------------------------------

Info Pregunta: e86b93df-f4fc-4a69-a71b-2dd3304e97ae

149. El síndrome de Zieve consiste en:

1. Anemia hemolítica, plaquetopenia inmune y cirrosis hepática.


2. Esteatosis hepática alcohólica, anemia hemolítica e hipertrigliceridemia.
3. Hiperesplenismo y esteatosis hepática.
4. Anemia microangiopática, esplenomegalia y hepatitis crónica.

Resp. Correcta: 2

Comentario:

El síndrome de Zieve es una complicación extremadamente infrecuente de la esteatosis hepática secundaria


al alcohol (hígado graso alcohólico). Se define por la asociación de hipertrigliceridemia, anemia
hemolítica e ictericia en un paciente con esteatosis hepática alcohólica. Los pacientes suelen tener
además dolor abdominal. La plaquetopenia, cirrosis, hiperesplenismo e hipergammaglobulinemia también
pueden manifestarse como parte de este síndrome, aunque no forman parte de la tríada que lo define.

-----------------------------------------------------------o--------------------------------------------------------------

Info Pregunta: 447a055d-6c73-4842-bd01-2e254a36ca90

150. Un paciente varón de 35 años, con antecedentes de dolor epigástrico, consulta por
un cuadro de debilidad y mareo, presentando al poco tiempo melena sin hematemesis.
¿Cuál es el origen más probable de la hemorragia?:

1. Hemorragia por varices esofágicas.


2. Fístula anal.
3. Ulcus duodenal.
4. Angiodisplasia de colon.

Resp. Correcta: 3

Comentario: La primera causa de hemorragia digestiva alta es la úlcera duodenal. El leiomioma de estómago
es raro, y también es raro que sangre. La hemorragia por varices esofágicas ocurre en pacientes con
hipertensión portal (generalmente cirróticos). La angiodisplasia de colon es típica en ancianos y suele
presentarse como sangrado recurrente. Y en la fístula anal generalmente lo que se observa es sangre roja con
las heces.

-----------------------------------------------------------o--------------------------------------------------------------

Info Pregunta: 8c859ca8-09c8-4b55-b061-2e7cb1bb05f1

151. Hombre de 60 años con cirrosis hepática etílica en abstinencia desde hace 8 años y
con función hepática grado A de CHILD. Se le diagnostica de carcinoma hepatocelular
de 10 cm de diámetro que invade arteria porta. El tratamiento indicado es:

1. Quimioembolización
2. Trasplante hepático
3. Sorafenib
4. Resección quirúrgica

Resp. Correcta: 3

Comentario: El paciente corresponde a un estadío C del BCLC estando indicado en tal caso el Sorafenib.

-----------------------------------------------------------o--------------------------------------------------------------

Info Pregunta: a5f61d6e-427e-462c-a0ce-2e7e7daa653f

152. ¿En cuál de las siguientes enfermedades autoinmunes podemos encontrar


típicamente anticuerpos antiendomisio?:

1. Lupus eritematoso sistémico.


2. Hepatitis.
3. Diabetes.
4. Enfermedad celíaca.

Resp. Correcta: 4

Comentario: Se trata de una pregunta directa. En la enfermedad celíaca se expresan una serie de
autoanticuerpos (antigliadina, antiendomisio, antitransglutaminasa tisular). Todos estos autoanticuerpos son
de tipo IgA únicamente; salvo los antigliadina, que son de tipo IgA e IgG. Los anticuerpos antiendomisio y
los antitransglutaminasa tisular (un subtipo de los antiendomisio), tienen una rentabilidad diagnóstica
cercana al 100 %. Y con una dieta correcta exenta de gluten, pueden llegar a negativizarse, por lo que son
muy útiles en el seguimiento.

-----------------------------------------------------------o--------------------------------------------------------------

Info Pregunta: 180b1b69-b7e7-4d8e-825c-2ea580139c66

153. Varón de 43 años, alcohólico, que consulta por desnutrición, edemas y diarrea de
un año de evolución. Un estudio de grasas fecales demostró 30 g/24 horas con nitrógeno
fecal de 4 g/24 h (normal hasta 2,5). La prueba de la D-xilosa fue normal. Entre las
siguientes pruebas diagnósticas, ¿cuál recomendaría a continuación?:

1. D-xilosa marcada con C14.


2. Alfa-1-antitripsina fecal.
3. Prueba de pancreolauryl.
4. Test de la permeabilidad intestinal.

Resp. Correcta: 3

Comentario:

Lo primero que debemos conocer es la utilidad de la prueba de la D- xilosa. Se emplea para detectar
situaciones en las que la integridad de la pared intestinal esté alterada. En condiciones normales, la D- xilosa
se absorbe en intestino delgado y se elimina a través de la orina. En cambio, en las patologías que alteren la
pared intestinal, la D- xilosa se excretará por las heces, porque no podría absorberse, con lo que no llegaría a
sangre ni a orina. Ante la normalidad de esta prueba, habría que dirigir el estudio hacia una posible
maldigestión por insuficiencia pancreática. Lógicamente, la prueba inicial debería ser no invasiva, por lo que
elegiríamos el test del pancreolauryl. Consiste en administrar vía oral una sustancia que, si hay una buena
secreción de las enzimas pancreáticas, se degrada a distintos productos que se podrán absorber y
detectaremos en orina posteriormente.

-----------------------------------------------------------o--------------------------------------------------------------

Info Pregunta: 483236e7-a52e-4f52-9c6e-2ef662b4427f

154. Cuál se los siguientes antivirales de acción directa tiene como mecanismo de
acción ser un inhibidor de la proteasa?

1. Grazoprevir
2. sofosbuvir
3. dasabuvir
4. Ombitasvir

Resp. Correcta: 1

Comentario: Los antivirales de acción directa acabamos en Asvir inhiben a la polimerasa A, los acabados en
Buvir a la B y los acabados en PRevir a la PRoteasa

-----------------------------------------------------------o--------------------------------------------------------------

Info Pregunta: 20d6e6a3-8c09-4c52-9382-2f69bd6dc3be

155. Un paciente le consulta para pedir segunda opinión porque en una endoscopia
realizada en otro centro le han diagnosticado de esofago de Barret con displasia leve.
¿cuando indicaría la siguiente endoscopia?

1. A los 6 meses.
2. A los 2 años.
3. A los 3 años.
4. A los 5 años.
Resp. Correcta: 1

Comentario: En el esófago de Barret con displasia leve se recomiendan las revisiones endoscópicas con
toma de biopsias cada 6 meses.

-----------------------------------------------------------o--------------------------------------------------------------

Info Pregunta: 0c3be698-eb08-446e-a430-2fd7096b8bda

156. Marta es una paciente de 36 años que desde los 8 años de edad padece vitíligo.
Desde hace unos meses se encuentra muy cansada y su médico de cabecera ha
descubierto en una analítica que padece una anemia con déficit de vitamina B12. Es
derivada al digestivo para estudiar el origen de esta anemia. Respecto a la gastritis que
sospecha en la paciente, ¿cuál de las siguientes afirmaciones es FALSA?

1. Es la forma más frecuente de gastritis crónica.


2. Es más frecuente en el fundus.
3. Aumenta el riesgo de cáncer gástrico.
4. Suele haber un estado de hipoclorhidria e hipergastrinemia.

Resp. Correcta: 1

Comentario:

Caso clínico que describe a una paciente con gastritis crónica tipo A, atrófica o autoinmune; no es la forma
más frecuente de gastritis crónica, siendo mucho más frecuente la gastritis B asociada a H. pylori (opción 1
falsa, por lo que la marcamos). La gastritis atrófica es más prevalente en mujeres y puede relacionarse con
otro tipo de enfermedades autoinmunes, como puede ser el vitíligo o la enfermedad de Addison. Es más
frecuente en el fundus, y se relaciona con la existencia de autoanticuerpos que destruyen las células
parietales del estómago (el 90% son anticélulas parietales y el 40% anti-Factor intrínseco). El déficit de
células parietales hace que disminuya la secreción de HCL (hay un estado de hipoclorhidria el que no
necesita tratamiento) y, a su vez, este estado provoca un aumento de gastrina compensadora que aumenta el
riesgo de cáncer gástrico, en concreto de gastrinomas.

-----------------------------------------------------------o--------------------------------------------------------------

Info Pregunta: cab2db44-50e3-4af2-900f-2ffe3fd328d6

157. ¿Cuál de los siguientes datos clínicos es MÁS frecuente en la úlcera gástrica que
en la duodenal?

1. Dolor 1,5 horas después de las comidas.


2. Dolor que se alivia con la comida.
3. Vómitos.
4. Dolor que despierta al paciente por la noche.

Resp. Correcta: 3

Comentario:

Los síntomas de las úlceras duodenales resultan ser mucho más típicos que los de las gástricas en la mayor
parte de los casos. Cuando hablamos de estos síntomas típicos, nos referimos a los que aparecen en todas las
opciones de respuesta, excepto en la 3, ya que la presencia de vómitos es más frecuente en las gástricas
porque asocian un componente de gastritis con mayor frecuencia y severidad (respuesta 3 correcta).

-----------------------------------------------------------o--------------------------------------------------------------

Info Pregunta: 233f29f4-4557-412a-9bdf-30dbb45850f7

158. ¿Quién desconjuga los ácido biliares para que puedan ser reabsorbidos en el íleon
terminal (cirulación enterohepática)?

1. lipasa intestinal
2. enterocinasa
3. tripsina
4. la flora bacteriana

Resp. Correcta: 4

Comentario: Por acción de la flora bacteriana, en el intestino, los ácidos biliares se desconjugan, lo que
permite su reabsorción a nivel del íleon terminal. A través de la circulación portal vuelven al hígado donde
se reconjugan y se secretan de nuevo a la bilis (circulación enterohepática)

-----------------------------------------------------------o--------------------------------------------------------------

Info Pregunta: ba8ec1cf-4e2e-4f45-99d5-313542de409c

159. Nos encontramos ante un varón de 50 años que presenta, sobre todo cuando se
inclina hacia delante, una sensación de quemazón retroesternal que se irradia hacia
ambos lados del tórax y mandíbula, la sensación disminuye de intensidad al tragar
saliva. ¿En qué diagnóstico pensaría usted?:

1. Esófago de Barrett.
2. Enfermedad por reflujo gastroesofágico.
3. Espasmo esofágico difuso.
4. Acalasia esofágica primaria.

Resp. Correcta: 2

Comentario: La clínica fundamental de la acalasia es la disfagia, tanto a líquidos como sólidos y en muy
menor medida pirosis, pues es excepcional la presencia de reflujo, ya que tienen contraído el esfínter
esofágico inferior. En caso de presentar pirosis sería por esofagitis química,inducida por el contacto
continuo con el alimento. El espasmo esofágico causa dolor y, en ocasiones, disfagia con la deglución. Y la
pirosis que se modifica con los cambios posturales sugiere enfermedad por reflujo ácido. La posibilidad de
un esófago de Barret no se podría excluir, pero obviamente estaría en el contexto de una enfermedad por
reflujo subyacente.

-----------------------------------------------------------o--------------------------------------------------------------

Info Pregunta: a62e5efb-be28-45f5-ac6f-3142d2be190e

160. Una mujer de 26 años es diagnosticada de colitis ulcerosa, con afectación de los 30
cm distales del colon. Tras tratamiento durante 15 días con enemas de mesalazina (5-
ASA), la paciente se recupera, por lo que los suspende totalmente. Al cabo de 2
semanas comienza de nuevo con aumento del número de deposiciones, con sangre
escasa y dolor abdominal leve. Está afebril y tolera la alimentación sólida. ¿Qué
medida adoptaría en esta paciente?:

1. Comenzar con prednisona oral.


2. Comenzar con inmunosupresores.
3. Comenzar con prednisona por vía parenteral.
4. Restituir el tratamiento con enemas de mesalazina y seguir una disminución paulatina, si resulta
efectivo.

Resp. Correcta: 4

Comentario: Estamos ante un brote leve de colitis ulcerosa, con predominio de afectación distal (proctitis).
Por tanto, es de elección el uso de 5'ASA tópicos (enemas). La mejoría y resolución del cuadro es clara y
completa en el primer episodio, por lo que ante la aparición de síntomas similares dos semanas después
estaría indicado exactamente el mismo tratamiento.

-----------------------------------------------------------o--------------------------------------------------------------

Info Pregunta: 428ce66e-6b21-48ea-b9ea-3162e7acf470

161. Respecto a las complicaciones de las úlceras gastroduodenales, señale la


afirmación CORRECTA:

1. La hemorragia es la más frecuente.


2. La hemorragia es la que tiene la morbimortalidad más elevada.
3. La perforación tiene escasa morbimortalidad si se aborda por laparoscopia.
4. La estenosis pilórica es más frecuente que la perforación.

Resp. Correcta: 1

Comentario:

Respecto a las complicaciones de las úlceras gastroduodenales debemos saber que la más frecuente es la
hemorragia, aunque la mayoría de las veces se consigue controlar conservadoramente mediante endoscopia.
La perforación de una úlcera gastroduodenal es la segunda complicación en orden de frecuencia y se
presenta en un 80% de los casos como consecuencia de una úlcera duodenal y en un 20% de una úlcera
gástrica, fundamentalmente prepilórica. La mortalidad es elevada, pudiendo llegar hasta un 10% sea cual sea
el abordaje quirúrgico que se emplee en su reparación. Por último la estenosis es la complicación menos
frecuente.

-----------------------------------------------------------o--------------------------------------------------------------

Info Pregunta: e54a8e11-57ac-4d61-b08d-31cdbbf5e1d1

162. ¿Cuál de las siguientes pautas es más probable que sea efectiva en la erradicación
de Helicobacter Pylori después del fracaso de un primer tratamiento con, metronidazol
(500mg cada 12 horas vía oral), Omeprazol (dosis doble/12h), Amoxicilina(1g/12h),
Claritromicina (500 mg/12h) durante diez días?:

1. Repetir nuevamente la pauta inicial pero mantenida durante 14 días.


2. Repetir la pauta inicial pero cambiando Amoxicilina por Tetraciclina (500 mg/6h).
3. Repetir la pauta inicial pero cambiando Claritromicina por Metronidazol (500 mg/8h).
4. Administrar bismuto, Omeprazol (20 mg/12h), Amoxicilina (1 g/12h) y Levofloxacino (500 mg/12h)
Resp. Correcta: 4

Comentario:

Como pautas de segunda línea aceptadas a día de hoy se encuentras la cuaduple terapia y la “BOLA”

-----------------------------------------------------------o--------------------------------------------------------------

Info Pregunta: aa484cc8-e00d-4f36-b23a-31f9874c8733

163. Señale, de entre los siguientes microorganismos, cuál NO se asocia a diarrea


enteroinvasiva:

1. E. coli enterohemorrágico.
2. Bacillus cereus.
3. Amebiasis.
4. Shigella.

Resp. Correcta: 2

Comentario:

El Bacillus cereus es un germen que produce en su forma entérica una diarrea acuosa mediada por
producción de toxinas sin invadir la pared intestinal.

-----------------------------------------------------------o--------------------------------------------------------------

Info Pregunta: 61840aa8-df7d-48ee-8054-3248b8e0ccbd

164. La reposición urgente de la volemia en un paciente politraumatizado con pérdidas


importantes de sangre, en un primer momento se logra MEJOR a través de:

1. Catéter de Swan-Ganz vía subclavia.


2. Disección de la vena safena.
3. Catéter central vía yugular interna.
4. Dos cánulas venosas cortas percutáneas.

Resp. Correcta: 4

Comentario:

La clave de esta pregunta está en el propio enunciado: “URGENTE”, “PÉRDIDAS IMPORTANTES”, “EN
UN PRIMER MOMENTO”, por lo que habrá que elegir la técnica más rápida, la que permita infundir
líquidos de la forma más inmediata posible. Por ello, la opción de respuesta correcta es la 4. Si disponemos
de una vía central y un catéter de Swan-Ganz, incluso podríamos monitorizar presión pulmonar, presión de
enclavamiento, etc., pero si se trata de un politraumatizado en el momento del accidente, lo prioritario es la
reposición urgente de volumen, y colocar tales dispositivos es laborioso y consume más tiempo.

-----------------------------------------------------------o--------------------------------------------------------------

Info Pregunta: 6cf56db9-faad-401f-8a8d-327649fbd646

165. Álvaro es un paciente de 34 años de edad que desde hace 4 meses refiere molestias
gastrointestinales tras la comida. No tiene antecedentes personales de interés ni
familiares, no refiere pirosis ni otros síntomas gastrointestinales, solo dispepsias. El
médico le comenta que puede tomar un procinético e ir siguiendo la evolución en unos
meses, sin embargo el paciente no mejora y exige una solución a su problema temiendo
que tenga una bacteria en el estómago como tuvo su madre y que esta le esté
produciendo algo grave. ¿Cuál de estas actitudes sería la correcta?

1. Comenzar con tratamiento para erradicar de H.pylori con metronidazol, omeprazol claritromicina y
amoxicilina durante 10 días.
2. Dar de alta al paciente explicándole que es un problema funcional que se resolverá de manera
progresiva.
3. Comenzar tratamiento para erradicar H.pylori con metronidazol, omeprazol, levofloxacino y
amoxicilina.
4. Realizar la prueba del Test del aliento con urea.

Resp. Correcta: 4

Comentario:

Nos encontramos con un paciente con dispepsia sin más síntomas al cual no encontramos motivos
funcionales para estos síntomas y queremos descartar la infección por H.pylori. Actualmente el 50% de la
población de los países desarrollados están infectados por esta bacteria que coloniza el estómago y la
mayoría son asintomáticos sin necesitar tratamiento alguno. Sin embargo, síntomas como padecer dispepsias
es una causa por las cuales está indicada la erradicación siendo SIEMPRE antes de comenzar el
tratamiento el diagnóstico de la infección cuya prueba más utilizada es el Test del aliento con urea.
(Respuesta correcta 4)

Si esta prueba resultara positiva, el tratamiento de primera línea que deberíamos pautar sería el de la
respuesta 1.

-----------------------------------------------------------o--------------------------------------------------------------

Info Pregunta: b0104e20-2147-4956-a305-32f2e65c692e

166. Si es un paciente con déficit de vitamina B12 realizamos un test de Schilling y tras
administrar vitamina B12 + enzimas pancreáticos se corrige el déficit de B12 la causa
es:

1. anemia perniciosa
2. insuficiencia pancreática
3. sobrecrecimiento bacteriano
4. lesión ileal

Resp. Correcta: 2

Comentario: Si al reponer la tripsina se corrige la deficiencia de vitamina B12 la causa es un déficit de


producción de enzimas a nivel pancreático

-----------------------------------------------------------o--------------------------------------------------------------

Info Pregunta: 1247bd74-69d5-4cd2-bb72-332b509504b7


167. Señale cual de los siguientes afirmaciones no se corresponde con los criterios de
Bethesda revisados del síndrome de Lynch:

1. Paciente con cáncer colorrectal a los 40 años.


2. Paciente con cáncer colorrectal a los 48 años con histología con reacción “Crohn-like” en la pieza.
3. Paciente de 52 años con cáncer colorrectal y antecedente en su madre de cáncer vesical a los 58 años.
4. Paciente con cáncer colorrectal a los 50 años y de endometrio a los 56 años.

Resp. Correcta: 3

Comentario:

Los criterios de Bethesda revisados son los siguientes: paciente con cáncer colorrectal diagnosticado antes
de los 50 años, que este sea sincrónico o metacrónico a otro tumor asociado al síndrome de Lynch, paciente
con cáncer colorrectal diagnosticado antes de los 60 años, histología característica de síndrome de Lynch
como la reacción Crohn-like, paciente con cáncer colorrectal y un familiar en primer grado con un tumor
asociado al síndrome de Lynch siendo al menos uno de ellos diagnosticado antes de los 50 o paciente con
cáncer rectal y dos son más familiares de primero segundo grado con un tumor asociada al síndrome de
Lynch.

-----------------------------------------------------------o--------------------------------------------------------------

Info Pregunta: 1a97adde-d7b0-43e2-9ac2-332e8c9df4fd

168. En relación con el hepatocarcinoma, señale la afirmación que considere


CORRECTA:

1. Una alfafetoproteína normal descarta el diagnóstico.


2. El tratamiento con sorafenib consigue aumentar ampliamente la supervivencia en pacientes
subsidiarios de trasplante.
3. La alcoholización es el tratamiento más adecuado para un hepatocarcinoma de 9 cm.
4. En la localización subcapsular hepática debe evitarse la radiofrecuencia.

Resp. Correcta: 4

Comentario:

Con respecto al hepatocarcinoma debes saber que, aunque es un tumor que típicamente produce
alfafetoproteína, no es una constante. El diagnóstico puede hacerse por el comportamiento en una técnica de
imagen en las que se observe una lesión > 1 cm con realce en fase arterial y lavado precoz en fase venosa.
La localización subcapsular o en contacto con la porta hacen que deba evitarse la radiofrecuencia (respuesta
4 correcta), ya que, por mal control de la zona tratada, se asocia a rotura de estas estructuras. La
alcoholización es más eficaz en tumores pequeños.

-----------------------------------------------------------o--------------------------------------------------------------

Info Pregunta: b39a401b-0c4a-46aa-8e39-33a24cc4d79e

169. Señale la afirmación que considere correcta en relación con la fisiología de la


digestión y absorción a nivel intestinal:

1. El hierro se absorbe en forma ferrosa fundamentalmente en el yeyuno distal.


2. La secretina activa fundamentalmente la secreción de enzimas pancreáticos y la colecistoquinina de
bicarbonato.
3. El calcio se absorbe fundamentalmente en el íleon por un mecanismo de transporte activo dependiente
de vitamina D.
4. La digestión de los hidratos de carbono comienza desde la misma cavidad oral.

Resp. Correcta: 4

Comentario: La digestión de los hidratos de carbono comienza desde la misma cavidad oral por la acción de
la amilasa salivar. El hierro se absorbe en forma ferrosa fundamentalmente en el duodeno, al igual que el
calcio por un mecanismo de transporte activo dependiente de vitamina D. La secretina activa
fundamentalmente la secreción de bicarbonato y la colecistoquinina de enzimas pancreáticos.

-----------------------------------------------------------o--------------------------------------------------------------

Info Pregunta: dd1ebd2d-1e4f-435d-b964-347505e36b18

170. ¿Cuál de los siguientes hallazgos no esperaría encontrar en una enfermedad de


Whipple?

1. Alfa-1-antitripsina fecal normal


2. Esteatorrea
3. Biopsia con macrófagos con inclusiones PAS+
4. Linfadenopatías

Resp. Correcta: 1

Comentario: La enfermedad de Whipple presenta enteropatía pierde-proteínas asociada, la cual se


diagnóstica por un aumento de alfa-1-antitripsina fecal aumentada.

-----------------------------------------------------------o--------------------------------------------------------------

Info Pregunta: 826f9fe6-1788-4b84-a25e-3492eb3bd606

171. ¿Cual de las siguientes es el objeto principal de erradicar la infección por H.


Pylori en la ulcera gástrica?

1. Evitar la recurrencia ulcerosa.


2. Acelerar la cicatrización de la lesión.
3. Evitar la progresión a cáncer.
4. Evitar la necesidad de utilizar antisecretores en su tratamiento.

Resp. Correcta: 1

Comentario: Con la erradicación de H. Pylori en la ulcera gástrica y duodenal lo que se busca


fundamentalmente es disminuir de forma muy significativa la recurrencia ulcerosa.

-----------------------------------------------------------o--------------------------------------------------------------

Info Pregunta: 9d7e3a1d-d2eb-4ebb-ab78-34ddc89415d8

172. La infección crónica por virus de la hepatitis B continua siendo un problema de


salud mundial de enorme relevancia. Respecto a su tratamiento con interferón
pegilado. ¿Cuál sería el efecto secundario que más probabilidad encontrará al
administrarlo?:

1. Alopecia
2. Síndrome pseudogripal
3. Irritabilidad
4. Cuadro depresivo

Resp. Correcta: 2

Comentario: El efecto secundario más frecuente del tratamiento con Interferón es el síndrome pseudogripal.

-----------------------------------------------------------o--------------------------------------------------------------

Info Pregunta: d12beb92-40e2-4afc-857b-364d08255988

173. Varón de 30 años, con una colitis ulcerosa conocida, que viene con signos de colitis
grave, fiebre elevada, taquicardia, depleción de volumen con desequilibrio electrolítico
y dolor abdominal. La placa simple de abdomen muestra una dilatación del colon de 7
cm de diámetro. Señalar la FALSA:

1. La localización más frecuente de la dilatación es el colon descendente-sigma.


2. Si se perfora, la mortalidad es elevada
3. Si con el tratamiento médico no hay clara mejoría, debe hacerse colectomía de urgencia.
4. Hay que administrar corticoides intravenosas y antibióticos de amplio espectro.

Resp. Correcta: 1

Comentario:

El caso clínico es típico de una complicación muy grave de la colitis ulcerosa: el MEGACOLON TÓXICO.
Por definición, se diagnostica cuando, ante una clínica compatible, se observa en la RX simple de abdomen
una dilatación en colon transverso mayor de 6 centímetros. Inicialmente, debe tratarse con medidas
conservadoras (sueroterapia, corticoides sistémicos, antibióticos de amplio espectro). Y si en pocos días no
hay mejoría, colectomía urgente, dado el riesgo de perforación.

Es muy importante que recuerdes que la zona dilatada es el colon transverso. Ha sido preguntado en varias
convocatorias.

-----------------------------------------------------------o--------------------------------------------------------------

Info Pregunta: dc3d2f9f-c264-4610-b39d-3690731c014e

174. Un hombre de 25 años consulta por ictericia. Practica el culturismo y se ha


inyectado esteroides anabolizantes sustitutos del 17 alfa-alquil, tres semanas antes. Por
una amigdalitis había tomado amoxicilina-clavulánico que retiró hace 15 días.
Analítica: AST 1.200 UI/L (límite superior normal, lsn 40), ALT 1300 UI/L (lsn 40),
GGT 150 UT/1, (1.511 50), fosfatasa alcalina 180 UIJL (lsn 105), bilirrubina total 4,8
mg/dL con predominio de bilirrubina directa. Serología de virus Anti-HBs y Anti-HBc
positivos. La ecografía sugiere esteatosis grado I. ¿Cuál es el diagnóstico MÁS
probable?

1. Hepatitis por amoxicilina-clavulánico.


2. Hepatitis por esteroides anabólicos.
3. Hepatitis aguda por virus B.
4. Esteatohepatitis no alcohólica.

Resp. Correcta: 1

Comentario:

El paciente presenta una hepatitis, dado que se observa una hipertransaminasemia de alto rango con patrón
colestásico (fosfatasa alcalina, GGT e hiperbilirrubinemia a expensas de la fracción directa). Esta última se
debe al infiltrado inflamatorio parenquimatoso que dificulta el flujo biliar. La serología corresponde a una
hepatitis B pasada. Aunque tiene una esteatosis hepática, cabe destacar que ecográficamente leve; si
presentara datos de hepatitis, suele cursar con hipertransaminasemia de bajo rango. Por todo ello, y con los
antecedentes referidos en el enunciado, las opciones de respuesta entre las que sería más razonable dudar
serían la 1 y la 2. La hepatitis por amoxicilina-clavulánico cursa típicamente con este patrón de hepatitis
(hasta en el 75% de los casos). Además, se describe un período de latencia compatible desde el cese del
tratamiento con amoxicilina-clavulánico hasta el comienzo de los síntomas (normalmente oscila entre 13
días y 6 semanas). Los anabolizantes tipo testosterona, aunque suelen asociarse más a aparición de tumores
y lesiones vasculares hepáticas, pueden provocar hepatitis colestásica. El hecho de su menor frecuencia
descrito en la literatura por esta causa y que suela cursar con hipertransaminasemia de bajo rango durante el
tratamiento con anabolizantes, lo hacen menos probable como etiología del cuadro que presenta el paciente.

-----------------------------------------------------------o--------------------------------------------------------------

Info Pregunta: 1b8c67f7-6819-47eb-ac67-36a76b0dee09

175. Mujer de 62 años, que ingresa por ictericia franca. Se le realiza una
colangiorresonancia magnética nuclear que muestra una masa en la cabeza del
páncreas y estenosis de colédoco adyacente a la masa, pero también una estenosis de
conducto hepático común. Una punción-aspiración con aguja fina de la masa no
muestra células malignas. Debido a una crisis de broncoespasmo no relacionada con el
problema pancreático, se administran corticoides. Curiosamente la ictericia cede. Se le
realiza una TC abdominal y la masa ha disminuido de tamaño, apreciándose una
menor dilatación de la vía biliar. ¿Qué prueba sería MÁS útil para confirmar la causa
MÁS probable del cuadro?

1. Determinación serológica de IgG 4.


2. PET-TAC.
3. Gammagrafía con octreótido marcado.
4. Estudio serológico de Virus de la Inmunodeficiencia Humana (VIH).

Resp. Correcta: 1

Comentario:

La pancreatitis autoinmune (PAI) es una enfermedad inflamatoria crónica, infrecuente y poco conocida,
expresión pancreática de una entidad recientemente descrita y conocida como enfermedad esclerosante
relacionada con IgG 4, que afecta a múltiples órganos. Se caracteriza por la aparición de masas pancreáticas
de origen inflamatorio (pero que por imagen pueden ser indistinguibles de neoplasias, por lo que la
diferenciación histológica es necesaria), que pueden ir asociadas a masas inflamatorias en la vía biliar. La
PAI es de curso benigno, sin embargo, en muchos casos, indistinguible del cáncer de páncreas por técnicas
de imagen en las que ambas entidades aparecen como lesiones ocupantes de espacio pancreáticas o biliares,
por lo que su diagnóstico diferencial es importante, ya que puede evitar un número de resecciones
pancreáticas innecesarias con la morbimortalidad que estas conllevan. El estándar de oro para el diagnóstico
diferencial de ambas entidades es el examen histológico de la lesión. El tratamiento con esteroides produce
mejoría clínica y la desaparición de las masas inflamatorias por imagen.

-----------------------------------------------------------o--------------------------------------------------------------

Info Pregunta: a4fb772f-d017-497f-b3e4-36e113da3612

176. Varón de 42 años, sin antecedentes personales de interés, que presenta cuadro de
ocho meses de evolución, progresivo, consistente, según refiere la familia del paciente,
en apatía, indiferencia a lo que sucede a su alrededor, reducción de la iniciativa,
inhibición psicomotriz y disminución marcada del flujo del lenguaje. El cuadro clínico
es interpretado como una depresión mayor, iniciando tratamiento con fluoxetina.
Pasan los meses y, lejos de mejorar, la clínica del paciente sigue progresando. Ha
comenzado a tener problemas mnésicos, ha sufrido un marcado descenso de su
rendimiento laboral, que ha conducido a su despido laboral, y lo más angustioso para
la familia es que ha comenzado a precisar ayuda para casi todas las actividades de la
vida diaria. Los familiares han advertido que está continuamente frotándose las
manos. En la exploración física: temblor de manos, tanto en reposo como postural,
hipertonía e hiperreflexia de las cuatro extremidades, con respuesta de Babinski.
Presenta disartria, con hipofonía, y hiponimia facial. Asimismo, se aprecia a nivel
ocular anillo corneal de Kayser-Fleischer. Se le realiza un TAC cerebral en el que se
pueden apreciar lesiones hipodensas a nivel de los ganglios basales, confirmadas
posteriormente en el estudio mediante RMN. Ante la sospecha de una enfermedad de
Wilson, se solicitan estudios para valorar el metabolismo del cobre: ceruloplasmina
sérica 119 mg/dL (disminuida), cupremia 183 g/dL (aumentada), cupruria 125 g/dL
(aumentada). Con relación a lo que sabemos de la enfermedad de Wilson, esta puede
manifestarse inicialmente de todas las formas expuestas en las siguientes opciones,
EXCEPTO:

1. Hepatitis crónica activa.


2. Hepatitis fulminante.
3. Coma hiperosmolar.
4. Anemia hemolítica.

Resp. Correcta: 3

Comentario:

El coma hiperosmolar es una complicación metabólica aguda que aparece en la diabetes mellitus tipo 2
como consecuencia de una intensa hiperglucemia que se acompaña de poliuria osmótica, con la consiguiente
deshidratación. En la enfermedad de Wilson, al ser una enfermedad hepática, lo que cabría esperar es todo lo
contrario: tendencia a la hipoglucemia, por disminución de la gluconeogénesis hepática, por lo que la opción
de respuesta 3 (coma hiperosmolar) sería la correcta, ya que la hipoglucemia es diametralmente opuesta a lo
que cabría esperar en esta entidad.

-----------------------------------------------------------o--------------------------------------------------------------

Info Pregunta: d8ce245c-2e90-4d80-b7c8-375e12d6c53d


177. Varón de 62 años, hipertenso, diabético tipo 2 y retinopatía diabética, acude a
urgencias por melenas de 3 días de evolución con empeoramiento del estado general y
mareo. A su llegada presentaba TA 80/40 mmHg y 135 lpm. El tacto rectal fue positivo
para melenas. La analítica practicada en urgencias mostraba los siguientes resultados:
Hb 6.4 g/dl, VCM 78 fl, leucocitos 12500/mm3, plaquetas 175.000/mm3, glucosa 280
mg/dl, urea 184 mg/dl, creatinina 1.4 mg/dl, GOT 40 U/L, GPT 30 U/L, GGT 28 U/L,
bilirrubina total 0.8 mg/dl, LDH 250 U/L, amilasa 10 U/ml. Tras estabilización se
practicó una gastroscopia que mostraba la existencia de una ulcera gástrica de 2 cm
con fondo de fibrina y una lesión elevada de coloración granate (vaso visible) y bordes
intensamente hiperémicos. ¿Cuál seria su actitud a continuación?:

1. Esclerosis con adrenalina 1/10000 y colocación de hemoclip.


2. Alta hospitalaria con IBP oral.
3. Vagotomía supraselectiva con piloroplastia
4. Fulguración de bordes con argón

Resp. Correcta: 1

Comentario: Se presenta el caso d aun paciente con una ulcera Forrest IIa (al tener un vaso visible), por lo
que la actitud más adecuada sería el doble tratamiento endoscópico acompañado de perfusión i.v. de IBPs.

-----------------------------------------------------------o--------------------------------------------------------------

Info Pregunta: a99ba748-48ec-4ec8-9b0d-37978caccddd

178. Una de las siguientes afirmaciones acerca de los virus de la hepatitis NO es


correcta. Indíquela:

1. En contraste cn el VHB y VHC, la transmisión por sangre del VHA es poco común, porque la viremia
es breve y de bajo título.
2. El VHE es un virus defectivo que usa el antígeno de superficie del VHB como su proteína de
envoltura.
3. La polimerasa codificada por el VHB actúa como una transcriptasa inversa, usando el RNAm vírico
como molde para la síntesis de los genomas de DNA progenie.
4. La detección del HBsAg durante más de seis meses indica un estado del portador crónico.

Resp. Correcta: 2

Comentario: Pregunta básica sobre hepatitis vitales Opción 1 (correcta): a pesar de que la transmisión del
virus de la hepatitis A es fecal oral presenta un corto periodo de viremia que hace factible (aunque
infrecuente) una transmisión parenteral Opción 2 (incorrecta): el virus defectivo que utiliza el antígeno de
superficie el virus B como su proteína de envoltura es el virus delta (no el virus E) Opción 3 (correcta) La
polemizaras es la base para la síntesis de los genomas de la progenie Opción 4 (correcta): el marcador de
presencia de viral es el AgHBs, su presencia durante más de 6 meses es el criterio que empleamos para
definir la cronicidad de una infección por VHB.

-----------------------------------------------------------o--------------------------------------------------------------

Info Pregunta: 2645cedd-d825-4db3-bae7-3880a8283328

179. Un chico de 17 años, deportista y sin antecedentes de interés, acude al médico por
presentar ictericia conjuntival ocasionalmente. Niega ingesta de fármacos y dolor
abdominal. En los análisis realizados se observan cifras de ALT, AST, GGT, y FA
normales, con bilirrubina total de 3,2 mg/dl y bilirrubina directa de 0,4 mg/dl. No tiene
anemia y el hígado es ecográficamente normal. ¿Cuál es su diagnóstico?

1. Síndrome de Rotor.
2. Síndrome de Dubin-Johnson.
3. Hepatitis aguda.
4. Síndrome de Gilbert.

Resp. Correcta: 4

Comentario:

El síndrome de Gilbert es un déficit de la conjugación hepática por un deficiente funcionamiento de la


enzima glucuronil-transferasa. Esto genera un incremento de la bilirrubina indirecta en situaciones de estrés
hepático (ayuno, consumo de fármacos, infecciones, grandes esfuerzos). El resto del perfil bioquímico
hepático es normal y el hígado ecográficamente también presenta un aspecto normal. Para el diagnóstico es
necesario solicitar las fracciones de bilirrubina y realizar el test del ayuno.

No requiere tratamiento.

-----------------------------------------------------------o--------------------------------------------------------------

Info Pregunta: e8c924b6-916f-4b59-85eb-38ca0a9a076f

180. ¿En qué capa de la pared esofágica se encuentra el plexo mientérico de Auerbach?

1. mucosa
2. submucosa
3. muscular
4. adventicia

Resp. Correcta: 3

Comentario:

En la pared del esófago se encuentran dos plexos nerviosos:

- El de Meissner en la capa submucosa

- El mientérico de Auerbach en la capa muscular (opción 3 correcta)

-----------------------------------------------------------o--------------------------------------------------------------

Info Pregunta: 3bf8b232-e9c9-4cdc-9f5a-3900b6c246a9

181. ¿Cuál de los siguientes pólipos tiene mayor potencial de malignización?:

1. Pólipo hiperplásico.
2. Adenoma tubular.
3. Adenoma velloso.
4. Pólipo inflamatorio.

Resp. Correcta: 3

Comentario: El pólipo con mayor potencial de malignización es el adenomatoso que presentan cierto grado
de displasia, dentro de los cuales se incluyen los tubulares, los vellosos y los mixtos. Los tubulares son los
más frecuentes y generalmente son pequeños y pediculados. Los vellosos en cambio, son grandes y sésiles,
siendo los de mayor potencial de malignización. Manual CTO.

-----------------------------------------------------------o--------------------------------------------------------------

Info Pregunta: 96437534-19ff-40b1-a9c2-39edac757740

182. Paciente de 46 años que consulta por presentar dificultad progresiva para ingerir
alimentos, tanto a sólidos como a líquidos, de aproximadamente un año de evolución,
junto con una notoria pérdida de peso. El paciente también nos cuenta que en
ocasiones presenta un fuerte dolor retroesternal durante la ingesta, que cede
posteriormente. No tiene antecentes médicos de interés. ¿Ante el cuadro clínico que se
presenta que prueba diagnóstica indicaría en primer lugar?

1. Manometría esofágica.
2. Tránsito baritado esófago-gástrico.
3. Endoscopia digestiva alta.
4. TAC torácico.

Resp. Correcta: 2

Comentario: La acalasia es el trastorno motor esofágico más frecuente. En cuanto a su etiología esta puede
ser primaria (más rara, de etiología desconocida, en la que se produce una pérdida sobre todo de neuronas
inhibitorias a nivel de los plexos mientéricos) o secundaria (recordar que la causa más frecuente de acalasia
secundaria es una neoplasia a nivel del fundus gástrico). En esta pregunta se refleja la clínica típica de la
acalasia: disfagia progresiva tanto para líquidos como para sólidos, dolor retroesternal con la ingesta
(explicado por la dificultad que tiene la comida para pasar hacia el estómago por un esfínter esofágico
inferior que no se relaja con la deglución), pérdida progresiva de peso y regurgitación no ácida (si bien, en
este caso, este último síntoma no viene reflejado es muy importante recordar que la acalasia no asocia
reflujo, ni por tanto la clínica típica de ERGE). Es una enfermedad que afecta más frecuentemente al adulto
joven (entre los 30-50 años), sin existir grandes diferencias de indicidencia entre sexos. Ante un caso de
disfagia, la secuencia diagnóstica es la siguiente: estudio baritado à endoscopia digestiva alta à manometría
esofágica. El estudio baritado es la primera prueba a realizar, la endoscopia digestiva alta nos permite
estudiar causas orgánicas de disfagia, y por último, la manometría permite confirmar o descartar el
diagnóstico de acalasia.

-----------------------------------------------------------o--------------------------------------------------------------

Info Pregunta: c93fe963-4f49-4586-afff-3a00a26d8ee0

183. Un paciente ingresa con el diagnóstico de hemorragia digestiva alta en situación


hemodinámica estable. Se le practica una gastroscopia que informa: lesión ulcerada en
cara posterior del bulbo duodenal con hemorragia activa no pulsátil. Forrest lb. Se
practica esclerosis endoscópica con adrenalina, consiguiéndose hemostasia. Señale la
afirmación CORRECTA:
1. La lesión descrita tiene un riesgo bajo de recidiva hemorrágica.
2. En caso de recidiva hemorrágica, es imprescindible la intervención quirúrgica.
3. Por la localización de la lesión, puede estar afectada la arteria gastroduodenal.
4. La descripción y localización de la úlcera sugieren una lesión de Dieulafoy.

Resp. Correcta: 3

Comentario:

Se trata de una pregunta directa sobre hemorragia digestiva por úlcera duodenal de cara posterior de bulbo,
por donde anatómicamente, discurre la arteria gastroduodenal y, por consiguiente, podría estar afectada. La
lesión con sangrado activo corresponde en la clasificación de Forrest a una úlcera Forrest Ib (I: Sangrado
activo, II: Signos de sangrado reciente, III: Ya no hay signos de sangrado reciente). Ésta, presenta en caso de
cese espontáneo de sangrado, unas tasas de recidiva muy altas, en torno al 50% de los pacientes o más. Por
ello, requiere tratamiento endoscópico como así se refleja en el enunciado. En el caso de recidivar la
hemorragia, puede realizarse un nuevo tratamiento endoscópico. La indicación de tratamiento quirúrgico se
utilizaría en caso de fracaso de esta segunda terapéutica endoscópica o hemorragia exanguinante. Según la
gravedad de la misma, tamaño y factores propios del paciente, pueden realizarse una vagotomía troncular,
un Billroth o la sutura-ligadura de la úlcera (técnica cada vez más utilizada.

-----------------------------------------------------------o--------------------------------------------------------------

Info Pregunta: 9cf42c21-f411-4d9e-98c7-3a3bba2ce2e8

184. Varón de 40 años, con antecedente de alcoholismo, que ingresa por vómitos en
posos de café. En la endoscopia se aprecian múltiples erosiones gástricas, algunas de
ellas sangrantes, y varices esofágicas de grado II sin signos de sangrado. El tratamiento
de elección para este paciente sería:

1. Fotocoagulación con láser.


2. Omeprazol i.v.
3. Propanolol i.v.
4. Embolización arterial.

Resp. Correcta: 2

Comentario: Se trata de un paciente con hemorragia digestiva alta. El origen del sangrado no se debe a las
varices esofágicas, ya que son de grado II (baja probabilidad de sangrado) y no presentan signos de sangrado
activo. El origen que se observa al estudio endoscópico es debido a erosiones gástricas: el tratamiento de
elección para esta enfermedad es el omeprazol i.v.

-----------------------------------------------------------o--------------------------------------------------------------

Info Pregunta: 60cf1318-3b9d-42c7-9adb-3a410e202f3c

185. Mujer de 23 años sin antecedentes de interés, acude por deshidratación y sincope.
En el interrogatorio afirmaba la existencia de una diarrea de hasta 4-6 deposiciones al
día sin sangre con moco ocasional. La exploración física demostraba ligera palidez de
piel y mucosas, constitución muy delgada y ligera palidez. La analítica mostraba los
siguientes resultados: Hb 10 g/dl, leucocitos 8500/mm3, plaquetas 450.000/mm3,
glucosa 68 mg/dl, urea 62 mg/dl, creatinina 1.7 mg/dl, GOT 10 U/L, GPT 20 U/L,
bilirrubina total 1.6 mg/dl, fosfatasa alcalina 20 U/L, LDH 140 U/L, amilasa 40 U/ml,
sodio 148 mEq/L, potasio 3.4 mEq/L, VSG 15 mm, PCR 1 mg/dl y calprotectina 35
mg/g. El estudio de heces mostraba ausencia de gérmenes y parásitos y un gap fecal de
258 mmol/l. ¿Con la información aportada, cual sería el tipo predominante de diarrea
que tiene el paciente?:

1. Diarrea inflamatoria.
2. Diarrea secretora.
3. Diarrea facticia.
4. Diarrea motora.

Resp. Correcta: 3

Comentario: La diarrea facticia es una diarrea inducida por consumo de laxantes y por tanto que cumple
criterios de diarrea osmótica. Dado que el gap fecal es alto, lo más probable es que se trate de una diarrea
osmótica. Dado que no aparece entre las opciones y la delgadez marcada de la paciente, apuntan a una
diarrea facticia.

-----------------------------------------------------------o--------------------------------------------------------------

Info Pregunta: a0623535-b0cb-433c-a8b0-3ada52078b45

186. En cuanto al tratamiento de la enfermedad de Crohn, indique la afirmación que le


parece INCORRECTA:

1. Las fístulas y complicaciones perianales pueden tratarse con antibióticos.


2. La obstrucción intestinal puede ser tratada con esteroides.
3. El tratamiento quirúrgico está reservado a las complicaciones y debe ser lo más conservador posible.
4. Los corticoides deben ser empleados como tratamiento de mantenimiento.

Resp. Correcta: 4

Comentario: En el tratamiento de la enfermedad de Crohn, durante los brotes está indicado el uso de
esteroides. Una vez controlado el brote, es necesario un tratamiento de mantenimiento. Para esto, no se
emplearían corticoides, ya que sería un tratamiento a largo plazo, y los importantes efectos secundarios de
los corticoides contraindicarían su uso prolongado. Usaríamos como tratamiento de mantenimiento los
5'ASA. En la enfermedad de Crohn se tiende a evitar al máximo posible la cirugía, ya que más de dos tercios
de los pacientes requerirán a lo largo de su vida una intervención quirúrgica. El metronidazol queda
reservado para los casos con enfermedad fistulosa perianal.

-----------------------------------------------------------o--------------------------------------------------------------

Info Pregunta: cd025078-00c2-4141-bba0-3afbbe2897f7

187. Varón de 53 años que acude a urgencias por malestar general progresivo con
aumento de perímetro abdominal en las ultimas semanas. A su llegada presenta un
abdomen muy globuloso con semiología de ascitis a tensión. La analítica practicada
mostró los siguientes resultados: Hb 12 g/dl, leucocitos 7400/dl, plaquetas 65.000/mm3,
actividad de protrombina 40%, glucosa 108 mg/dl, urea 55 mg/dl, creatinina 1.4 mg/dl,
GOT 70 U/L, GPT 88 U/L, bilirrubina 2 mg/dl, GGT 128 U/L, amilasa 50 U/ml,
proteínas totales 3.2 g/dl, albúmina 1.4 g/dl, LDH 80 U/L, sodio 142 mEq/L, potasio 4
mEq/L, proteína C reactiva 0.2 mg/dl. ¿Cuál sería la actitud más adecuada a
continuación?:

1. Restricción hídrica.
2. Paracentesis evacuadora con reposición de albúmina.
3. Iniciar Espironolactona 100 mg/día.
4. Colocación de TIPS.

Resp. Correcta: 2

Comentario: Comentario realizado en formato vídeo

-----------------------------------------------------------o--------------------------------------------------------------

Info Pregunta: a457622d-897b-4dc5-9841-3b32a21776d8

188. Ante una sospecha de colitis ulcerosa, ¿qué prueba escogería entre las siguientes
para confirmar el diagnóstico?:

1. Gastroscopia.
2. Enema opaco.
3. Sigmoidoscopia.
4. Tránsito intestinal.

Resp. Correcta: 3

Comentario: En la colitis ulcerosa, es muy frecuente y característica la afectación del recto- sigma
(proctitis), por lo que la realización de una sigmoidoscopia sería muy rentable. La gastroscopia debería ser
normal, ya que en la colitis ulcerosa sólo se suele afectar el colon. El resto de pruebas, aunque pueden
sugerir el diagnóstico, presentan una desventaja clar frente a la sigmoidoscoipia. Y es que esta última
permite la obtención de biopsia colónica para confirmar el diagnóstico.

-----------------------------------------------------------o--------------------------------------------------------------

Info Pregunta: ee1dd04d-4294-473d-a551-3ba275a801ee

189. Sobre el mecanismo de transmisión, para evitar la infección por el VHB, es útil lo
siguiente EXCEPTO:

1. Es necesario aislar los pacientes HBsAg positivo.


2. La principal vía de transmisión es la sangre contaminada.
3. La ebullición a 100 ºC diez minutos consigue la esterilización.
4. Es importante como mecanismo de transmisión el mecanismo sexual.

Resp. Correcta: 1

Comentario:

Teniendo en cuenta la vía de transmisión del VHB (hematógena y sexual), los pacientes portadores del virus
(HBsAg positivos) no precisan aislamiento, por lo que la respuesta correcta es la 1. Los que sí lo precisan
son los pacientes con hepatitis A, ya que la transmisión del VHA es orofecal.

-----------------------------------------------------------o--------------------------------------------------------------
Info Pregunta: ab434404-79ad-4260-8a03-3bde3a1ea998

190. Paciente de 28 años, sometido hace 3 años a una panproctocolectomía


restauradora con reservorio en J ileal debido a una colitis ulcerosa refractaria al
tratamiento médico, que acude a consulta de revisión. Refiere presentar una diarrea
líquida con moco acompañada de tenesmo, incontinencia y dolor pélvico de dos
semanas de evolución. En la analítica solo destaca un valor de PCR de 12 mg/L. Señale
el diagnóstico de sospecha y su tratamiento:

1. Reservoritis. Ciprofloxacino + metronidazol junto con enemas de budesonida y probióticos.


2. Desgarro esfinteriano. Esfinteroplastia si el defecto es < 180 grados.
3. Absceso perianal. Amoxicilina-clavulánico + drenaje.
4. Reservoritis. Ciprofloxacino + metronidazol junto con probióticos y azatioprina.

Resp. Correcta: 1

Comentario:

Caso clínico sobre un paciente diagnosticado de colitis ulcerosa que ha sido tratado con una
panproctocolectomía total con reservorio ileal en J. En este contexto, la reproducción de la clínica de la CU,
en ocasiones junto con incontinencia, nos debe hacer sospechar la presencia de una reservoritis aguda, en la
cual la mucosa del reservorio sufre un proceso de metaplasia colónica y puede volver a expresar un proceso
inflamatorio similar a la enfermedad original. El tratamiento de la misma sería ciprofloxacino +
metronidazol junto con enemas de budesonida y probióticos (respuesta 1 correcta). La azatioprina se reserva
para un tercer escalón en el manejo de la reservoritis crónica en caso de que los tratamientos anteriores
hayan fracasado.

-----------------------------------------------------------o--------------------------------------------------------------

Info Pregunta: 92180df9-1e1f-4118-a4b2-3be1ca69d853

191. Un paciente de 35 años acude a su consulta por presentar dolor epigástrico intenso
de varios meses de evolución, que a veces le despierta por la noche y que cede con la
ingesta, excepto con determinados alimentos (salsa de tomate, cebolla, alcohol y
especias). Con solo estos datos clínicos, señale cuál de los siguientes es su diagnóstico de
sospecha:

1. Cáncer de estómago.
2. Úlcera duodenal penetrada en páncreas.
3. Úlcera duodenal.
4. Úlcera gástrica.

Resp. Correcta: 3

Comentario:

El cuadro que nos presentan es bastante sugestivo de una úlcera duodenal. En esta enfermedad, el síntoma
más frecuente es dolor epigástrico, que aparece entre 1,5 y 3 horas después de las comidas, y que se alivia
con los alimentos o antiácidos. El síntoma más discriminante, pero lejos todavía de ser perfecto, es el dolor
que despierta al paciente por la noche entre las 00:00 h y las 3:00 a.m. El dolor es inducido por el ácido. Un
cambio en las características del dolor debe hacer pensar en una complicación. Ten cuidado y no lo
confundas con una úlcera gástrica. Estas manifestaciones también podrían deberse a este tipo de úlcera, pero
es menos frecuente y, desde el punto de vista clínico, no se comporta de una forma tan típica. Un dato
habitual en las úlceras gástricas es la mayor frecuencia de vómitos (por ejemplo, en este paciente no
aparecen).

-----------------------------------------------------------o--------------------------------------------------------------

Info Pregunta: 332eda11-3ae0-4bf6-8295-3ca3675b214c

192. Acude un niño de 4 años a Urgencias pediátricas tras la ingesta accidental de lejía.
De entre las siguientes pruebas diagnósticas, ¿cuál NO aporta beneficio alguno en el
momento agudo?

1. Exploración física de la cavidad orofaríngea.


2. Radiografía simple de tórax.
3. Endoscopia alta.
4. Tránsito digestivo baritado.

Resp. Correcta: 4

Comentario:

Ante una causticación esofágica, es obvio que conocer el tóxico ingerido es fundamental para distinguir si es
de naturaleza ácida o alcalina. La exploración de la cavidad orofaríngea permite ver si hay úlceras por el
cáustico o sospechar lesión de la vía aérea. La Rx de tórax es precisa para evaluar la posible perforación, que
se manifestaría como neumomediastino. En cuanto a la endoscopia, debe realizarse de forma urgente
durante las primeras 24 horas para valorar la extensión, localización y severidad de las lesiones (habiendo
descartado previamente perforación esofágica mediante la Rx de tórax). En algunos casos el tránsito
baritado puede ser de utilidad (por ejemplo, si hay dudas en la radiografía), pero en este caso el paciente está
perforado y el bario puede provocar una mediastinitis química. Si decidimos hacer un contraste, habría que
hacerlo con un contraste de tipo hidrosoluble (marcamos la opción de respuesta 4).

-----------------------------------------------------------o--------------------------------------------------------------

Info Pregunta: 72dc9cdc-f2d0-4a52-8013-3d93a7ce771f

193. Varón de 34 años que acude por aumento en el número de deposiciones, con heces
de coloración amarillenta, sin sangre ni moco, ha perdido 4-5 kg en las últimas
semanas y presenta discreta palidez cutánea. No presenta otros antecedentes salvo
catarros frecuentes. Se le realiza Van de Kamer con 16 g de grasa en heces/24 horas, y
el test de D-xilosa resulta normal. Usted sospecharía:

1. Sobrecrecimiento bacteriano.
2. Enfermedad celíaca.
3. Insuficiencia pancreática exocrina.
4. Colon irritable.

Resp. Correcta: 3

Comentario:

En esta pregunta no sería lógico pensar en un colon irritable, donde no habría pérdida de peso ni esteatorrea
(recuerda que se considera por encima de 7 g de grasa en heces). Las opciones de respuesta 1 y 2 suelen
alterar el test de la D- Xilosa, por lo que solo nos queda la insuficiencia pancreática exocrina (respuesta 3
correcta). El test de la D-xilosa consiste en administrar este azúcar en ayunas, y luego se mide su presencia
en orina, cuando pasan unas horas. La D-xilosa se absorbe en el yeyuno por difusión pasiva, sin necesidad
de que intervengan las enzimas pancreáticas o las sales biliares; por ello, está alterada en procesos que
afectan a la pared del tubo digestivo, sin verse influida por alteraciones pancreáticas.

-----------------------------------------------------------o--------------------------------------------------------------

Info Pregunta: 789c9858-9e18-4029-aad0-3ecbbccfc963

194. Varón de 60 años que una analítica de control se observa hipertransaminasemia


de bajo rango. La exploración física y la ecografía fueron normales. En el estudio de
hepatopatía, se observaron los siguientes marcadores serológicos: anti-VHA IgM -,
anti-VHA IgG +, AgHBs -, antiHBs +, antiHBc +, AgHBe -, antiHBe +, DNA-VHB
indetectable, Ag-VHD -, Ac-VHC , RNA-VHC 120.000 UI/ml, genotipo 2. Ante estos
hallazgos, cual sería el tratamiento más adecuado:

1. Entecavir oral.
2. Sofosbuvir + Ledipasvir.
3. Pribentasvir + Glecaprevir.
4. Grazoprevir + Elbasvir.

Resp. Correcta: 3

Comentario:

Se presenta el caso de un paciente con serología de hepatitis A y B pasadas. El marcador de virus C es


positivo, confirmándose una infección crónica al observarse positividad para el RNA. Se trata de un
genotipo dos y por tanto solo podrían utilizarse las pautas pan genotípicas, como la de la opción número 3
(Pribentasvir + Glecaprevir o Sofosbuvir + Velpatasvir).

-----------------------------------------------------------o--------------------------------------------------------------

Info Pregunta: 3f84098f-659d-4fec-a957-3fb3dee75d4e

195. Señale la opción que considere INCORRECTA sobre el hepatocarcinoma


fibrolamelar:

1. Es un tumor típico de pacientes jóvenes.


2. Tiene mal pronóstico.
3. La elevación de alfafetoproteína es excepcional.
4. La mayoría son resecables al diagnóstico.

Resp. Correcta: 2

Comentario:

Se trata de un tumor típico de la juventud (segunda, tercera década de la vida) con buen pronóstico, ya que
al diagnóstico la mayoría son resecables y es excepcional las metástasis al inicio. Solo en el 10% hay
aumento de alfafetoproteína.

-----------------------------------------------------------o--------------------------------------------------------------

Info Pregunta: 0e548450-adef-44cc-947a-401b59f41687


196. Mujer de 61 años exfumadora, exbebedora y con antecedente de ictus de la
cerebral media hace 8 años que acude a revisión. La exploración física demostró la
presencia de hepatomegalia a 5 trasveses de dedo. La analítica mostraba los siguientes
datos: Hb 11.5 g/dl, leucocitos 6000/mm3 con 70% neutrófilos, plaquetas 110.000/mm3,
glucosa 157 mg/dl, urea 29 mg/dl, creatinina 1.3 mg/dl, AST 200 U/L, ALT 128 U/L,
GGT 310 U/L, bilirrubina total 1.1 mg/dl, fosfatasa alcalina 120 U/L, LDH 100 U/L,
amilasa 35 U/ml. La ecografía demostró la existencia de hepatomegalia con hígado
heterogéneo y contorno irregular y un nódulo de 7 cm. La gastroscopia fue normal., El
TC comprobaba la existencia de un nódulo en segmento VI de 72 mm con realce en
fase arterial y lavado precoz en fase venosa a la inyección de contraste. ¿Cuál sería la
opción de tratamiento más adecuada?:

1. Segmentectomía hepática
2. Radiofrecuencia percutánea de la lesión
3. Quimioembolización de la lesión
4. Vigilancia estrecha

Resp. Correcta: 1

Comentario:

Se nos presenta un paciente cirrótico con una lesión ocupante de espacio sólida cuyo comportamiento con
contraste en las pruebas de imagen es típico de hepatocarcinoma (marcado realce en fase arterial y
aclaramiento rápido en fase venosa). Dado que es una lesión única y que no presenta ictericia ni datos
sugerentes de hipertensión portal clínicamente significativa, podría realizarse cirugía resectiva.

-----------------------------------------------------------o--------------------------------------------------------------

Info Pregunta: c1059acd-452c-4be1-a940-40502331b664

197. ¿Cuál de las siguientes complicaciones no es frecuente en los pacientes que han
recibido un trasplante hepático alogénico?

1. Trombosis de la vena porta.


2. Rechazo hiperagudo.
3. Infecciones por Citomegalovirus.
4. Enfermedad linfoproliferativa.

Resp. Correcta: 2

Comentario:

El trasplante hepático es un tema poco preguntado. Debes centrarte en las indicaciones y contraindicaciones.

Las complicaciones y, por lo tanto, las causas de fracaso del trasplante varían a lo largo del tiempo. En los
tres primeros meses se suelen deber a complicaciones quirúrgicas de tipo técnico, o infecciones
postoperatorias. En este periodo de tiempo se suele desarrollar la trombosis de la vena porta o de la arteria
hepática.

Posteriormente, los fracasos se relacionan más con infecciones por la inmunosupresión, rechazo o recidiva
de la enfermedad primaria.
En el primer mes suelen ser infecciones bacterianas y, a partir del segundo, secundarias a la
inmunosupresión, apareciendo infecciones oportunistas como CMV, herpes, hongos… La inmunosupresión
también facilita el desarrollo de enfermedades linfoproliferativas. Y en cuanto al rechazo, el más frecuente
es el rechazo agudo celular, que es reversible con corticoides. Por el contrario, el rechazo hiperagudo es
poco frecuente. Se debe a incompatibilidad ABO, y el hecho de seleccionar a pacientes compatibles antes de
realizar el trasplante hace que su frecuencia sea escasa. Se debe a la existencia de Ac contra las células
endoteliales y no tiene tratamiento, la única solución es el retrasplante.

-----------------------------------------------------------o--------------------------------------------------------------

Info Pregunta: 296b79b4-d2d8-460f-a841-405a34c9ea9c

198. Mujer de 35 años que acude a consulta por epigastralgia urgente de predominio
postpandrial y deposiciones oscuras desde hace 8 días a raiz de la toma de
Dexketoprofeno por esguince en tobillo izquierdo. La exploración física fue normal. La
analítica mostraba: Hb 10,5 g/dl, VCM 60 fl, 8500 leucocitos, plaquetas 250.000/mm3.
Se solicitó una gastroscopia que mostraba la presencia de una ulcera de 1.5 cm en cara
anterior de bulbo duodenal con bordes ligeramente hiperémicos y elevados y fondo con
fibrina y puntos de hematina. ¿Cuál sería su actitud a continuación?:

1. Ingreso para perfusión i.v. de inhibidores de la bomba de protones.


2. Inhibidores de la bomba de protones orales
3. Esclerosis con adrenalina 1/10000, colocación de hemoclip y perfusión i.v. de inhibidores de la bomba
de protones.
4. Ligadura endoscópica con bandas y perfusión i.v. de inhibidores de la bomba de protones.

Resp. Correcta: 2

Comentario: Se trata de una ulcera Forrest Iic (Fibrina y puntos de hematina). Por tanto solo precisa
tratamiento con IBPs sin ser necesario el tratamiento endoscópico.

-----------------------------------------------------------o--------------------------------------------------------------

Info Pregunta: 9058b4ee-b61e-4943-999b-409aa0d05b76

199. Varón de 56 años, bebedor crónico, fumador, dislipémico y con antecedentes


personales de colelitiasis, acude a Urgencias por presentar, desde hace unas horas,
dolor epigástrico agudo asociado a cuadro vegetativo. Al realizar la exploración física
se aprecia una coloración azulada en flancos. Con respecto a la sospecha diagnóstica,
¿cuál de los siguientes parámetros NO es considerado útil para predecir la evolución
clínica del paciente cuando este ingresa en Urgencias?

1. GOT/AST.
2. Niveles de amilasa.
3. Leucocitosis.
4. LDH.

Resp. Correcta: 2

Comentario:

Ante una pancreatitis aguda, que es lo que muestra el paciente de la pregunta, se deben evaluar los criterios
de Ranson (5 al ingreso y 6 a las 48 horas). Estos criterios evalúan el riesgo que tiene el paciente de que su
pancreatitis derive a una pancreatitis grave. Los criterios de Ranson al ingreso del paciente son:
- LDH > 350 IU/L.
- Leucocitosis > 16.000 células/mm³.
- Edad > 55 años.
- Glucosa > 200 mg/dL.
- AST/GOT > 250 IU/L.
Los niveles de amilasa son útiles para el diagnóstico de la pancreatitis, pero su elevación no predice la
gravedad de la enfermedad.

-----------------------------------------------------------o--------------------------------------------------------------

Info Pregunta: a990b421-b7e1-49f6-aefb-41281e34759b

200. Valeria, una joven de 25 años, acude a tu consulta porque, desde hace meses, sufre
de molestias abdominales que describe como retortijones que alivian cuando va al
baño. Además, también te cuenta que unos días sufre de diarrea, y otros de
estreñimiento (ambos con evacuación de moco), sin relacionar estos cambios de hábito
deposicional con nada. Ante la sospecha clínica, ¿cuál de entre las siguientes opciones
es correcta?

1. La mayoría de las veces se encuentra una causa clara y tratable


2. Los síntomas abdominales se localizan siempre a nivel epigástrico.
3. El diagnóstico de esta entidad es puramente clínico.
4. Se debe tratar con urgencia, ya que el 90% deriva en patología maligna.

Resp. Correcta: 3

Comentario:

Valeria sufre del síndrome de intestino irritable. Se trata de un síndrome cuya etiología es desconocida, y
por ello, el tratamiento de la misma no está claro.

La localización de los síntomas es variable y, aunque generalmente se manifiestan en zona inferior


izquierda, pueden localizarse en cualquier otro punto (opción 2 falsa). El pronóstico de este síndrome es
benigno, ya que no predispone a ningún otro tipo de enfermedad.

-----------------------------------------------------------o--------------------------------------------------------------

Info Pregunta: a31f3aa8-2c2d-4d75-8bdb-41660e6d0d0c

201. Para diferenciar con certeza si una úlcera es benigna o maligna, basta con:

1. Aspecto radiológico del nicho ulceroso y su regresión con tratamiento antiulceroso.


2. Gastroscopia y multibiopsia.
3. Nivel sanguíneo de gastrina.
4. Estudio de la secreción gástrica (BAO y MAO).

Resp. Correcta: 2

Comentario: La metodología que garantiza el diagnóstico o la exclusión de un tumor es siempre el estudio


anatomopatológico. En el caso de las úlceras gástricas, casi un 5 % de ellas "esconden" un carcinoma. Éste
es el motivo por el que en el diagnóstico de cualquier úlcera gástrica es obligado el estudio endoscópico con
toma de biopsias de los bordes de la úlcera.

-----------------------------------------------------------o--------------------------------------------------------------

Info Pregunta: b75fdaf7-a7a4-417e-bde0-416d8c5b47fe

202. En un paciente trasplantado de hígado que está recibiendo inmunosupresión con


tacrolimus, ¿cuál de los siguientes antibióticos incrementa de forma notable los niveles
del inmunosupresor y, por tanto, debe evitarse?

1. Amoxicilina-clavulánico.
2. Ciprofloxacino.
3. Eritromicina.
4. Cefuroxima.

Resp. Correcta: 3

Comentario:

Eltacrolimus, se metaboliza vía citocromo hepático P-450 (CYP) 3A4. Los fármacos que inhiben esta
isoenzima pueden reducir el metabolismo, aumentando los niveles en sangre deltacrolimus, y, por lo tanto,
el riesgo de toxicidad. Algunos de estos agentes son: amiodarona, inhibidores de la proteasa antirretroviral,
bromocriptina, cimetidina, claritromicina, dalfopristina, quinupristina, danazol, delavirdina, fluoxetina,
fluvoxamina, metronidazol, metilprednisolona, mifepristona, RU-486, quinidina, quinina, y
troleandomicina. Los fármacos como la carbamazepina, fosfenitoína, nevirapina, oxcarbazepina,
fenobarbital, fenitoína, rifabutina, rifampina, rifapentina y troglitazona, que inducen el citocromo P-450
3A4, pueden aumentar los niveles en sangre deltacrolimus. Se recomienda la monitorización de los niveles
en sangre del tacrolimus, si alguno de estos agentes es utilizado al mismo tiempo. La administración
concomitante de eritromicina ytacrolimus, puede aumentar los niveles de este último, aumentando el riesgo
de nefrotoxicidad. Se debe evitar el uso de este antibiótico y, si fuera indispensable, se deben monitorizar los
niveles detacrolimus. Se han comunicado fallo renal, delirio y altas concentraciones detacrolimus, en
pacientes tratados simultáneamente contacrolimus, y nefazodona. Algunos antagonistas del calcio (como por
ejemplo diltiazem, nicardipina, nifedipina, y verapamil) y eltacrolimus, pueden mostrar interacciones, ya
que son todos ellos metabolizados por el citocromo P-450 3A4. El uso concomitante de agentes nefrotóxicos
debe ser considerado con precaución para evitar efectos aditivos. Algunos agentes que pueden causar
nefrotoxicidad son los antibióticos aminoglucósidos, la anfotericina B, el cisplatino, foscarnet, ganciclovir,
la bacitracina o la polimixina B, y la vancomicina. El paracetamol, la aspirina y los AINEs, deben ser
utilizados con precaución, dado que pueden enmascarar la fiebre, dolor e inflamación y otros síntomas de
infección. La administración concomitante detacrolimus, y ciclosporina, aumenta el riesgo de nefrotoxicidad
por efectos aditivos o sinérgicos; se recomienda no utilizar ambos fármacos simultáneamente. Cuando se
pasa de la ciclosporina al tacrolimus, se recomienda esperar al menos 24 horas después de la última dosis de
ciclosporina antes de comenzar el tratamiento contacrolimus.

-----------------------------------------------------------o--------------------------------------------------------------

Info Pregunta: ea5a2e80-e843-4a2a-b397-4231d96d13ae

203. ¿Cuál de las siguientes técnicas de endoscopia digestiva está asociada a mayor
riesgo de complicaciones mayores?;

1. Endoscopia digestiva alta diagnóstica.


2. Esclerosis de varices esofágicas.
3. Colonoscopia diagnóstica en paciente con dolicocolon.
4. Polipectomía de pólipo en colon descendente de morfología sésil.

Resp. Correcta: 2

Comentario: Pregunta anulable. De los cuatro procedimientos de la pregunta, y tomando como ejemplo de
complicación mayor, un evento hemorrágico, dos se consideran de bajo riesgo (ecoendoscopia alta
diagnóstica sin punción y colonoscopia diagnóstica) y dos de alto riesgo (tratamiento de varices y
polipectomía). En cuanto al riesgo de complicaciones, tras polipectomía de pólipo sésil, habrá que tener en
cuenta no sólo la localización sino también el tamaño del mismo. Se selecciona la respuesta 2, por el mayor
riesgo de bacteriemia transitoria en la esclerosis de varices (que justifica la realización de profilaxis
antibiótica en individuos de alto riesgo (portadores de válvula protésica, cardiopatías congénitas,
antecedente de endocarditis…) frente a la polipectomía.

-----------------------------------------------------------o--------------------------------------------------------------

Info Pregunta: 2ef7e0ba-9a00-4c4f-ae5f-427557d28fe2

204. Varón de 47 años, consumidor de 2 litros de cerveza al día, vino en las comidas y
como mínimo 1 bebida de alta graduación diaria, acude a urgencias por aumento de
perímetro abdominal e ictericia. Una ecografía abdominal demostró la existencia de un
hígado heterogéneo e irregular y liquido libre peritoneal en cuantía leve. La analítica
urgente mostraba los siguientes datos:: Hb 10.2, VCM 118 fl, ferritina 1700 ng/ml,
leucocitos 12500/mm3, neutrófilos 80%, plaquetas 145.000, actividad protrombina
30%, urea 62 mg/dl, creatinina 1.5 mg/dl, GOT 342 UI/L, GPT 208 UI/L, GGT 1190
UI/L, bilirrubina total 21.6 mg/dl, proteínas totales 4.5 g/dl, albúmina 2.2 g/dl,
fosfatasa alcalina 62 U/L, LDH 198 UI/L, triglicéridos 486 mg/dl, sodio 144 mEq/L,
potasio 3.8 mEq/L. ¿Ante la patología que sospecha, que tratamiento considera
indicado?:

1. Esteroides i.v.
2. Enoxaparina s.c.
3. Sorafenib oral.
4. N-acetilcisteina oral.

Resp. Correcta: 1

Comentario:

Se trata de un paciente con posible hepatitis aguda alcohólica puesto que presenta un cuadro de hepatitis
ictérica en el contexto de consumo excesivo de bebidas alcohólicas. Por los niveles de coagulopatía y la
bilirrubina, seguramente el índice de Madrey sea superior a 32 y, por consiguiente, se trata de una hepatitis
aguda etílica. En estos casos el uso de esteroides contribuye a mejorar el pronóstico de estos pacientes.

-----------------------------------------------------------o--------------------------------------------------------------

Info Pregunta: e213b73f-9985-4335-aa48-429b1345e5fc

205. Mujer de 62 años con cirrosis VHC que en ecografía de control se observó un
hígado heterogéneo e irregular con un nódulo de 4 cm en segmento VIII, sin otras
alteraciones relevantes. La analítica mostraba: Hb 13 g/dl, leucocitos 6000/mm3,
plaquetas 145.000/mm3, actividad de protrombina 60%, glucosa 89 mg/dl, urea 50
mg/dl, creatinina 1 mg/dl, GOT 90 U/L, GPT 60 U/L, GGT 100 U/L, bilirrubina total 1
mg/dl, fosfatasa alcalina 16 U/L, LDH 40 U/L, amilasa 28 U/ml, sodio 140 mg/dl,
potasio 4 mg/dl. La RMN hepática confirmo dicho nódulo que a la inyección de
gadolinio endovenoso tenía realce arterial y lavado rápido portal. ¿Ante la sospecha
más probable, cuál sería el estadio y tratamiento más correcto?:

1. Estadio A BCLC – Resección quirúrgica


2. Estadio D BCLC – Ablación percutánea
3. Estadio C BCLC - Sorafenib oral
4. Estadio B BCLC – Quimioembolización

Resp. Correcta: 1

Comentario: Se nos presenta un paciente cirrótico con una lesión ocupante de espacio sólida cuyo
comportamiento con contraste en las pruebas de imagen es típico de hepatocarcinoma (marcado realce en
fase arterial y aclaramiento rápido en fase venosa). Dado que es una lesión única y que no presenta ictericia
ni datos sugerentes de hipertensión portal, podría realizarse cirugía resectiva.

-----------------------------------------------------------o--------------------------------------------------------------

Info Pregunta: a4c25b84-9694-4ea1-bc9b-42df0713d31f

206. Una paciente de 40 años es estudiada por un cuadro de diarrea crónica,


diagnosticándose de probable enfermedad celíaca. Tras varios meses de tratamiento no
se consigue una mejoría. ¿Cuál es la primera prueba que le haría?

1. Investigar la existencia de un linfoma.


2. Colonoscopia para descartar colitis colágena.
3. Añadir azatioprina.
4. Revisar la dieta.

Resp. Correcta: 4

Comentario:

El tratamiento de la enfermedad celíaca consiste en aportar los nutrientes que falten y en realizar una dieta
libre de gluten. Lo más frecuente es que, en cuestión de semanas, haya una clara mejoría, que inicialmente
es clínica y después histológica.

Cuando el paciente no responde al tratamiento, existen muchas posibilidades, pero la más frecuente es que
no esté cumpliendo la dieta adecuadamente. También puede deberse a un diagnóstico incorrecto, la
existencia de otra causa concurrente (déficit de lactasa, esprue colágeno, desarrollo de un linfoma…). No
obstante, el mal cumplimiento es mucho más habitual (respuesta 4 correcta).

-----------------------------------------------------------o--------------------------------------------------------------

Info Pregunta: e47f7c23-73a2-4f4b-a4c7-431306b96dc3

207. El procedimiento INICIAL de elección para investigar una posible obstrucción de


las vías biliares, es:

1. Ecografía hepatobiliar.
2. Colangiopancreatografía retrógrada endoscópica (CPRE).
3. Colangiorresonancia Magnética.
4. Tomografía computarizada. TC abdominal.

Resp. Correcta: 1

Comentario:

La primera exploración a realizar ante un paciente con colestasis es la ecografía abdominal. A continuación,
se expone el algoritmo de manejo ante la sospecha de patología biliar:

-----------------------------------------------------------o--------------------------------------------------------------

Info Pregunta: 82f11d4d-68b9-4763-9bd9-4313c9f1fb24

208. En un paciente con cirrosis hepática, ¿cuál de los siguientes es el procedimiento


MÁS útil para medir la respuesta de la ascitis al tratamiento diurético?

1. Medir el perímetro abdominal diariamente.


2. Determinar cada semana el gradiente de albúmina entre el suero y la ascitis.
3. Registrar el peso cada día.
4. Evaluar la natriuresis cada 48 horas.

Resp. Correcta: 3

Comentario:

El control de la respuesta al tratamiento diurético en los pacientes cirróticos con ascitis se realiza con la
medición del peso diario y/o la natriuresis cada 24 horas.

-----------------------------------------------------------o--------------------------------------------------------------

Info Pregunta: d95e6454-0b8c-450a-8161-431730ddc161

209. Paciente de 15 años que acude por tercera vez este año a urgencias por presentar
episodios de impactación esofágica tras la ingesta de carne que han precisado de su
extracción por endoscopia oral. Como antecedentes cabe reseñar que presenta asma
bronquial. ¿En cual de las siguientes entidades pensaría como la más probable?

1. Acalasia
2. Esofagitis eosinofílica
3. Enfermedad por reflujo grastroesofágico
4. Esofagitis caústica

Resp. Correcta: 2

Comentario: En un paciente joven con impactación y antecedentes de asma la primera sospecha diagnóstica
es de esofagitis eosinofílica

-----------------------------------------------------------o--------------------------------------------------------------

Info Pregunta: c7aa1f3a-4c4f-4ab6-9427-431a983fc901


210. Un paciente de 34 años acude a su consulta preocupado por pérdida de la líbido.
Entre sus antecedentes destaca un episodio autolimitado de sinovitis en rodilla derecha,
necesidad de insulina y extirpación de quiste sebáceo de la ceja izquierda hace cuatro
meses. Usted le explora y percibe cierta coloración cutánea que le llama la atención, un
pezón supernumerario, y una hepatomegalia. Dentro de la analítica no existen
trastornos de la función hepática. ¿Hacia qué prueba analítica dirigiría su atención
principalmente?:

1. Serología de virus B y C.
2. Determinación de ácido úrico.
3. Cultivo de exudado uretral.
4. Metabolismo del hierro.

Resp. Correcta: 4

Comentario: Este paciente presenta la asociación de varios síntomas que podrían orientar hacia una
hemocromatosis: artropatía, disminución de la libido (hipogonadismo hipogonadotropo), diabetes mellitus y
hepatomegalia. En estadios iniciales de esta enfermedad la bioquímica hepática puede ser normal. Como
primera prueba diagnóstica se debe determinar la ferritina y la saturación de transferrina, que se confirmarán
con estudio genético y eventualmente biopsia hepática si están aumentadas.

-----------------------------------------------------------o--------------------------------------------------------------

Info Pregunta: dc0dcc59-cd29-482d-a4ac-43584b1a0201

211. Antonio es un paciente de 57 años, cirrótico por VHC, con 7 puntos en la


clasificación de CHILD. Es traído a Urgencias por su hijo, que le cuenta que cuando
fue a visitarle lo encontró “raro”. Usted percibe que Antonio se encuentra aletargado,
no responde con claridad a las preguntas que le formula y presenta una clara
incoordinación en los movimientos. A la exploración presenta ascitis y dolor a la
palpación abdominal. La temperatura es de 38,1 ºC. ¿Cuál es la causa MÁS probable
del cuadro clínico de Antonio?

1. Alteraciones iónicas.
2. Hemorragia digestiva alta.
3. Peritonitis bacteriana espontánea.
4. Estreñimiento.

Resp. Correcta: 3

Comentario:

Un cuadro clínico caracterizado por cambios en el estado mental y alteraciones neuromusculares en un


paciente cirrótico debe hacernos pensar en encefalopatía hepática. Todas las opciones de respuesta son
causas reconocidas de esta patología, aunque la presencia de ascitis, exploración abdominal dolorosa y
fiebre, apuntan a la peritonitis bacteriana espontánea como factor precipitante más probable de la
encefalopatía (respuesta 3 correcta).

-----------------------------------------------------------o--------------------------------------------------------------

Info Pregunta: 9ab8e33f-5cfe-4be5-a583-4374997fed74


212. Señale de entre las siguientes cuál no es una indicación de erradicación por H.
pylori:

1. Deficit de vitamina B12 idiopático.


2. Familiares en primer grado d pacientes con adenocarcinoma gástrico.
3. Esófago de Barrett.
4. Gastritis atrófica con metaplasia intestinal.

Resp. Correcta: 3

Comentario: El reflujo esofágico y, por ende, el esófago de Barret no son indicaciones de erradicación; Es
mas, la erradicación podría empeorar la clínica de reflujo.

-----------------------------------------------------------o--------------------------------------------------------------

Info Pregunta: c34d7780-b6e9-413b-a1dd-4375c6567b31

213. Señale de entre las siguientes afirmaciones en relación con la infección por H.
Pylori, cuál de ellas consideraría correcta:

1. La biopsia antral es un método directo con una sensibilidad de en torno al 90%, especialmente si se
toma de zonas metaplásicas.
2. La serología tiene su utilidad fundamentalmente en la comprobación de la erradicación.
3. El test de la ureasa es un test directo y rápido pero costoso.
4. El test de aliento es un método indirecto con una especificidad del 96%, aunque menor que la del
cultivo.

Resp. Correcta: 4

Comentario:

El test de aliento es un método indirecto con una especificidad del 96%, aunque menor que la del cultivo
que es del 97-100%. La serología tiene escasa utilidad tanto en diagnóstico como en comprobar la
erradicación. La biopsia antral es especialmente útil si se toma de zonas no atróficas. El test de ureasa es un
método indirecto, rápido y barato.

-----------------------------------------------------------o--------------------------------------------------------------

Info Pregunta: 82379463-47cc-4d12-9dfb-43a1ff373195

214. ¿Cuál de los siguientes regímenes terapéuticos le parece más útil frente a
Helicobacter pylori?:

1. Eritromicina más nitrofurantoína.


2. Metronidazol más ranitidina más ofloxacina.
3. Metronidazol más bismuto más tetraciclinas más omeprazol.
4. Omeprazol más norfloxacina.

Resp. Correcta: 3

Comentario: La pauta terapéutica de elección para erradicar H.pylori es la combinación de omeprazol,


amoxicilina y claritromicina. No obstante, se considera una pauta de similar eficacia, pero utilizada como
alternativa, la conocida como cuádruple, en la que se incluye bismuto coloidal, omeprazol, metronidazol y
tetraciclinas. Bismuto coloidal es el fármaco que, en forma aislada, tiene un mayor potencial frente a
H.pylori como bactericida.

-----------------------------------------------------------o--------------------------------------------------------------

Info Pregunta: 30c53d6d-2411-4e84-aee2-43d1a1f6a660

215. En un paciente con colitis ulcerosa de 12 años de evolución, las biopsias seriadas
demostraron la presencia de displasia grave en las muestras del área sigmoidea, ¿Qué
actitud recomendaría?:

1. Colectomía total.
2. Nueva toma de biopsias en 3 meses y resección de la lesión en caso de confirmarse.
3. Ecoendoscopia para evaluar la invasión de la pared intestinal.
4. Quimioterapia con 5-fluorouracilo y reevaluación en 6 meses.

Resp. Correcta: 1

Comentario: En la colitis ulcerosa, los factores de riesgo más importantes para el desarrollo de un cáncer de
colon son la existencia de una pancolitis y la duración de la enfermedad más de 8 años. La presencia de
displasia de alto grado o grave, así como la de un cáncer, son indicaciones de Proctocolectomía total.

-----------------------------------------------------------o--------------------------------------------------------------

Info Pregunta: 586498b8-6619-4538-aec9-43fe00c77925

216. En relación con los pólipos del colon, indique la afirmación que le parece cierta:

1. Hasta el 25% de los pólipos malignizan.


2. La mayoría de los cánceres colorrectales derivan de un pólipo adenomatoso.
3. El síndrome de Gardner asocia los pólipos hamartomatosos del colon a neoplasias del sistema
nervioso central.
4. La polipectomía con colonoscopia no es tratamiento suficiente de un pólipo con carcinoma in situ (no
sobrepasa la membrana basal glandular).

Resp. Correcta: 2

Comentario: Recuerda que la mayoría de los cánceres colorrectales (90% de los casos) provienen de un
pólipo previo (respuesta correcta). Sin embargo, ante un pólipo, la probabilidad de malignización es menor
del 1%, siendo los que más riesgo tienen los adenomatosos con componente velloso. En caso de carcinoma
in situ, como el tumor no sobrepasa la membrana basal, la polipectomía endoscópica es más que suficiente.
El síndrome de Gardner son pólipos adenomatosos

-----------------------------------------------------------o--------------------------------------------------------------

Info Pregunta: fbf9d27e-ceca-422f-a6bb-459428d38519

217. En un paciente con úlcera duodenal, si al finalizar el tratamiento fuese necesario


comprobar la erradicación de H. pylori, ¿cuál sería la prueba de mayor rentabilidad?:

1. Test de la ureasa.
2. Test de la urea en el aliento.
3. Título de anticuerpos.
4. Test de Lundh.

Resp. Correcta: 2

Comentario: En las úlceras duodenales no complicadas, no es obligatorio comprobar la erradicación del


Helicobacter pylori una vez completado el tratamiento. Sin embargo, si se realizase esta comprobación, la
prueba de elección sería el test del aliento, ya que de este modo nos evitaríamos la endoscopia. En cambio,
en las úlceras gástricas, al ser necesaria la endoscopia para la comprobación de la cicatrización de la úlcera,
las pruebas que se emplearían para comprobar erradicación serían el test de la ureasa y el cultivo de biopsia
antral, que juntas tienen la misma rentabilidad diagnóstica (95 %) que el test del aliento.

-----------------------------------------------------------o--------------------------------------------------------------

Info Pregunta: 3a826245-65c4-4071-a478-45a72088e67f

218. Varón de 52 años, fumador y bebedor moderado, que acude a urgencias por
melenas de 14 horas de evolución y cuadro presincopal en su domicilio. A su llegada
presenta TA 80/35 mmHg y 140 lpm. El tacto rectal comprobó la existencia de melenas.
La analítica urgente mostraba los siguientes resultados: Hb 9.2 g/dl, leucocitos
12000/mm3, plaquetas 350.000/mm3, glucosa 118 mg/dl, urea 106 mg/dl, creatinina 1.3
mg/dl, GOT 10 U/L, GPT 20 U/L, GGT 18 U/L, bilirrubina total 1.9 mg/dl, fosfatasa
alcalina 72 U/L, LDH 202 U/L, amilasa 40 U/ml. ¿Cuál sería la actitud más adecuada a
continuación?:

1. Administración de lactulosa en enemas.


2. Gastroscopia urgente.
3. Estabilización con cristaloides.
4. Transfundir 3 concentrados de hematíes.

Resp. Correcta: 3

Comentario:

Se trata de un paciente con hemorragia digestiva alta que acude a urgencias con hipotensión y taquicardia,
por lo que la primera actitud a tomar sería la estabilización hemodinámica con sueros. La transfusión de
hematíes estaría indicada por debajo de 7 de hemoglobina y por lo tanto en esta pregunta sería menos
correcta. La gastroscopia urgente se realizaría una vez estabilizado el paciente.

-----------------------------------------------------------o--------------------------------------------------------------

Info Pregunta: 809d3c5a-3f6e-49c3-9bdd-45ce4fda705c

219. Varón de 56 años con cirrosis alcohólica en seguimiento por digestivo,


asintomático e independiente para las actividades de la vida diaria, en el que en una
ecografía de control presenta un nódulo hipoecogénico de 7 cm en segmento VII
hepático y liquido libre abdominal. La analítica mostraba: Hb 12.2 g/dl, leucocitos
5000/mm3, plaquetas 80.000/mm3, glucosa 90 mg/dl, urea 35 mg/dl, creatinina 0.9
mg/dl, GOT 65 U/L, GPT 48 U/L, GGT 66 U/L, bilirrubina total 1.5 mg/dl, fosfatasa
alcalina 30 U/L, LDH 150 U/L, amilasa 30 U/ml, sodio 140 mg/dl, potasio 4 mg/dl. El
TC con contraste confirmó dicho nódulo con realce en fase arterial y lavado precoz en
fase venosa. ¿Cuál sería el tratamiento más adecuado?:
1. Segmentectomía hepática
2. Trasplante hepático
3. Quimioembolización
4. Sorafenib oral

Resp. Correcta: 3

Comentario: Se describe un paciente cirrótico alcohólico con vida normal y una lesión ocupante de espacio
sólida cuyo comportamiento con contraste en las pruebas de imagen es típico de hepatocarcinoma (marcado
realce en fase arterial y aclaramiento rápido en fase venosa). Dado que presenta un tamaño mayor de 5 cm y
datos sugerentes de hipertensión portal (ascitis), se trataría de un estadío B de la BCLC y requeriría
quimioembolización.

-----------------------------------------------------------o--------------------------------------------------------------

Info Pregunta: 0ba683a1-3dda-4581-805f-46a26e3d09cd

220. En el tratamiento de mantenimiento a largo plazo en pacientes con enfermedad


inflamatoria intestinal, ¿cuál de los siguientes fármacos es el MENOS recomendable?

1. Azatioprina.
2. Mesalazina.
3. Prednisona.
4. 6-Mercaptopurina.

Resp. Correcta: 3

Comentario:

Una pregunta muy clásica sobre el tratamiento médico de la enfermedad inflamatoria intestinal. Nunca se
debe utilizar tratamiento con corticosteroides (prednisona) como mantenimiento por el riesgo de efectos
secundarios asociados a los mismos. Estos solo se deben utilizar en el manejo del brote.

-----------------------------------------------------------o--------------------------------------------------------------

Info Pregunta: 7ad29305-cc2a-4942-8a6f-46dc38844a11

221. ¿Quién realiza la activación de tripsinógeno a tripsina en duodeno?

1. La colecistoquinina
2. La secretina
3. La enterocinasa
4. la lipasa

Resp. Correcta: 3

Comentario: La enterocinasa duodenal realiza la activación de tripsinógeno a tripsina siendo esta última la
que activa al resto de enzimas pancreáticos

-----------------------------------------------------------o--------------------------------------------------------------

Info Pregunta: 83f74cf3-c3ee-4c0b-8c4f-471e024bdf0f


222. En un programa de cribado de cáncer de colon, a un paciente de 52 años se le
realiza una colonoscopia. Todo el colon es de aspecto normal, salvo el hallazgo de un
pólipo pediculado de 2 cm en sigma que se extirpa con asa de diatermia. El resultado
anatomopatológico nos indica que existe un carcinoma “in situ” limitado a la cabeza
del pólipo. Tiene una TC toracoabdominal informada como normal. Indique cuál sería
la conducta correcta a seguir:

1. Resección segmentaria del colon afecto.


2. Vigilancia periódica endoscópica.
3. Resección local de la base del pólipo.
4. Sigmoidectomía más linfadenectomía.

Resp. Correcta: 2

Comentario: Un pólipo con un carcinoma in situ implica que la lesión maligna está limitada a la capa
mucosa. Por ello, la resección completa del pólipo implicaría su curación. Lo más importante es tener la
certeza en el informe anatomopatológico de que los márgenes están libres y por tanto no nos hemos dejado
ningún resto tumoral. En el caso, nos informan de que es un pólipo pediculado que se reseca de forma
completa, y que el carcinoma in situ se encuentra sólo en la cabeza del pólipo, por lo que los márgenes están
respetados. Encima nos dan el dato de que en el TC no se ven otras alteraciones, como adenopatías que
podría orientar a que el tumor se hubiera extendido. Por ello, la indicación es la vigilancia endoscópica
(opción 2 correcta).

-----------------------------------------------------------o--------------------------------------------------------------

Info Pregunta: 413681df-e4ac-471f-b29b-482d59133639

223. A un paciente de 24 años con antecedente de sinusitis de repetición se le ha


realizado una biopsia intestinal por síndrome de malabsorción. Se ha demostrado
atrofia subtotal de las vellosidades con un infiltrado linfocitario en lámina propia sin
evidencia de células plasmáticas. Cinco meses después de la retirada del gluten,
continúa con síntomas. Lo más probable es que:

1. Incumpla la dieta.
2. Presente un linfoma.
3. Padezca una hipogammaglobulinemia.
4. Haya desarrollado un adenocarcinoma.

Resp. Correcta: 3

Comentario: La hipogammaglobulinemia es una de las pocas entidades en que la biopsia intestinal es


patognomónica, y aporta el dignóstico. En el estudio anatomopatológico, se observa ausencia de células
plasmáticas. Otras enfermedades en las que la biopsia intestinal es diagnóstica son: linfangiectasia intestinal
(dilataciones tortuosas de los vasos linfáticos), enfermedad de Whipple (macrófagos con inclusiones PAS+),
abetalipoproteinemia (enterocitos cargados de gotas lipídicas). Por lo que es evidente que no se trata de una
enfermedad celíaca y que, por tanto, no haya mejoría tras una dieta sin gluten.

-----------------------------------------------------------o--------------------------------------------------------------

Info Pregunta: c9401fac-456c-448f-8b16-48584efec168

224. Un joven de 30 años está diagnosticado de colitis ulcerosa desde hace 11. En los
dos últimos análisis de sangre se ha determinado una elevación de la fosfatasa alcalina
en sangre. En base a la enfermedad que se debe sospechar, ¿cuál de las siguientes
afirmaciones le parece INCORRECTA?

1. El diagnóstico se realiza con colangioRM o CPRE.


2. El paciente puede presentar prurito, ictericia y episodios de fiebre con dolor en hipocondrio derecho.
3. El pronóstico de la enfermedad es bueno.
4. La imagen radiológica típica consiste en un arrosariamiento de la vía biliar.

Resp. Correcta: 3

Comentario:

La presencia de colestasis en un paciente varón en la edad media de la vida y con colitis ulcerosa nos debe
hacer sospechar una colangitis esclerosante primaria (CEP). El diagnóstico de elección en la CEP es la
colangioRM (previamente era la CPRE, aunque es invasiva con mayor tasa de complicaciones, reservándose
para casos dudosos), que muestra dilataciones y ensanchamientos múltiples de la vía biliar extra e
intrahepática. Esta enfermedad puede presentarse con episodios similares a una colangitis bacteriana, con
síntomas similares. El pronóstico de esta enfermedad no es bueno, ya que la supervivencia es de unos 9 a 12
años tras el diagnóstico; además, tienen una mayor incidencia de colangiocarcinoma.

-----------------------------------------------------------o--------------------------------------------------------------

Info Pregunta: c7f399ca-b915-41d4-bfeb-489ec63db17f

225. Varón de 12 años con al menos 6 episodios de pancreatitis aguda de carácter leve
de origen idiopático desde los 8 años. Acude nuevamente consulta por un nuevo
episodio de pancreatitis aguda. El estudio con Colangio-RMN, ecoendoscopia y
microscópico de bilis fueron normales. En relación con una de las entidades que
debería sospechar, señale la que considera incorrecta:

1. Es una posible causa de pancreatitis crónica.


2. Tienen un riesgo muy elevado de cáncer de páncreas.
3. Se debe normalmente a la mutación del gen inhibidor de la tripsina.
4. Se produce por aumento de la activación del tripsinógeno en tripsina activa.

Resp. Correcta: 3

Comentario:

En un paciente muy joven con pancreatitis de repetición y sin otras etiologías debe considerarse la
posibilidad de una pancreatitis hereditaria. El gen inhibidor de la tripsina (SPINK1) aunque esta implicado,
no es el más frecuente. La mutación más frecuente es la del gen que codifica para el tripsinógeno catiónico
(PRSS1).

-----------------------------------------------------------o--------------------------------------------------------------

Info Pregunta: da792785-e3b9-4cad-a207-48aa00c0936e

226. ¿Cuál de las siguientes características NO le parece que corresponda a los


síntomas de una úlcera duodenal?:
1. Dolor agudo quemante en el epigástrio.
2. El dolor aparece entre hora y media y tres horas después de las comidas.
3. No suele ser nocturno.
4. El dolor puede localizarse en hipocondrio derecho.

Resp. Correcta: 3

Comentario:

Aunque no hay ningún dato patognomónico en la clínica para diferenciar úlcera gástrica de duodenal, sí
puede haber síntomas y signos característicos de una u otra localización.

En la úlcera duodenal, el dolor aparece entre una hora y media a tres horas depués de las comidas, y que se
alivia con los alimentos o con antiácidos/antisecretores, típicamente despierta por la noche al paciente entre
las 00h y las 3 a.m. El dolor es inducido por el ácido. Mientras que la úlcera gástrica, tiene un patrón menos
típico y más irregular para el dolor, aunque es más frecuente la presencia de vómitos.

-----------------------------------------------------------o--------------------------------------------------------------

Info Pregunta: a2f5246f-c18f-4cbc-ae13-48ca6a22d115

227. Paciente de 23 años que refiere malestar postprandial con dolor leve de tipo cólico,
digestión pesada y sensación de distensión abdominal. Acerca del cuadro que presenta
el paciente, señale la afirmación CORRECTA:

1. Se realizará gastroscopia para descartar organicidad.


2. La asociación a H. pylori es excepcional, por lo que no es necesario investigar ni erradicar en caso de
ser positivo.
3. Si presenta estreñimiento asociado, el diagnóstico más probable hace que deba descartarse una
obstrucción intestinal con colonoscopia.
4. No tiene datos de alarma, lo que permite el tratamiento directo del paciente sin realizar más pruebas
complementarias.

Resp. Correcta: 4

Comentario:

La dispepsia funcional es un proceso frecuentemente asociado al Síndrome de intestino irritable que puede
cursar con sensación de ardor o de digestiones pesadas. El estreñimiento que nos relatan puede formar parte
de ese intestino irritable y no obligatoriamente de una estenosis que condicione una obstrucción. En
pacientes jóvenes sin signos de alarma puede tratarse según los síntomas sin necesidad de realizar más
exploraciones. El estudio de H. pylori en estos casos está recogido en los últimos consensos y algunos
pacientes se benefician de la erradicación. El tratamiento suele realizarse con IBPs o anti-H2, procinéticos y
fármacos para flatulencia tipo simeticona. El tratamiento con antiácidos y protectores de mucosa mejora solo
a algunos pacientes con dispepsia de tipo ulceroso y de forma sintomática no mantenida.

-----------------------------------------------------------o--------------------------------------------------------------

Info Pregunta: f6c05906-c4f6-45f7-a027-48dfa6301ea5

228. ¿Cuál de los siguientes tratamientos NO se considera como potencialmente


curativo del carcinoma hepatocelular?:
1. Cirugía resectiva
2. Radiofrecuencia
3. Sorafenib
4. Trasplante hepático

Resp. Correcta: 3

Comentario: El Sorafenib es un quimioterapia oral que aumenta la supervivencia de pacientes con


hepatocarcinoma avanzado pero no curativo. El resto de opciones si lo son.

-----------------------------------------------------------o--------------------------------------------------------------

Info Pregunta: 354f699f-763f-4e8b-80c4-4944128255b0

229. Varón de 62 años, fumador, diabético y bebedor diario, acude traído por su
familia por astenia e ictericia marcada en la ultima semana. La analítica mostraba los
siguientes resultados: Hb 16.2 g/dl, leucocitos 12500/mm3, PMN 75%, plaquetas
180.000/mm3, actividad de protrombina 35%, glucosa 160 mg/dl, urea 36 mg/dl,
creatinina 1.2 mg/dl, GOT 237 U/L, GPT 138 U/L, GGT 1520 U/L, bilirrubina total 9.6
mg/dl, fosfatasa alcalina 170 U/L, LDH 390 U/L, amilasa 40 U/ml, sodio 140 mEq/L,
potasio 4 mEq/L. El estudio especifico de hepatopatía fue: anti-VHA IgM negativo,
anti-VHA IgG positivo, AgHBs -, antiHBs +, antiHBc +, AgHBe -, antiHBe -, Ag-VHD
-, Ac-VHC -, ANA -, Ac anti- musculo liso -, anti-LKM -, pANCAs -, Ac anti-citosol
hepático -, AMA -, inmunoglobulina G 800 mg/dl (Valores normales 700-1700 mg/dl).
La ecografía fue normal salvo mínima cantidad de liquido perihepático. En base a su
diagnóstico de presunción, ¿cuál sería el tratamiento más adecuado?:

1. Silibinina oral.
2. Esteroides orales.
3. N-acetilcisteina oral.
4. Trasplante hepático urgente.

Resp. Correcta: 2

Comentario:

Se presenta el caso de un paciente bebedor y que presenta un cuadro compatible con una hepatitis aguda
alcohólica, puesto que presenta leucocitosis leve hipertransaminasemia moderada y marcada ictericia y
deterioro de la coagulación. Si se realizará el índice de Maddrey, dado el marcado deterioro de la
coagulación y la elevada hiperbilirrubinemia, se encontrará seguramente con un valor superior a 32. Ello le
confiere que sea una hepatitis grave y por tanto, se beneficia del uso de esteroides orales.

-----------------------------------------------------------o--------------------------------------------------------------

Info Pregunta: dff30150-cf84-4d9e-b3d4-49f4240c9db7

230. ¿Quién degrada el pepsinógeno a pepsina en la cavidad gástrica?

1. H pylori
2. El ácido clorhídrico
3. la gastrina
4. la histamina
Resp. Correcta: 2

Comentario: El ácido clorhídrico es el responsable de la degradación del pepsinógeno a pepsina (actividad


proteolítica)

-----------------------------------------------------------o--------------------------------------------------------------

Info Pregunta: 53493b86-01ee-463e-aa06-4a02fb9daf0b

231. Mujer de 40 años que presenta diarrea de hasta 10 deposiciones al día de dos
meses de evolución. La grasa en heces de 24 horas es de 9 g. Se realizó un test de D-
xilosa con niveles bajos en orina a las cinco horas. La alfa-1-antitripsina fecal es alta.
¿Cuál, de entre los siguientes, es el diagnóstico MÁS probable?

1. Enfermedad de Whipple.
2. Sobrecrecimiento bacteriano.
3. Insuficiencia pancreática.
4. Esclerodermia.

Resp. Correcta: 1

Comentario:

Caso clínico sobre una diarrea crónica en el que el test de grasa en heces y el test de D-xilosa son
patológicos. Ello hace que ya pueda descartarse una maldigestión y, por tanto, la insuficiencia pancreática.
La alfa-1-antitripsina fecal elevada indica que hay una enteropatía pierde-proteínas, por lo que la causa de
esteatorrea es una malabsorción por enteropatía que, entre las que se citan, aparece en la enfermedad de
Whipple (respuesta 1 correcta), que produce una enteropatía (afectación de la pared intestinal) cuya biopsia
es patognomónica (macrófagos con inclusiones PAS+). El sobrecrecimiento bacteriano no altera la pared
intestinal.

-----------------------------------------------------------o--------------------------------------------------------------

Info Pregunta: 9c72b78b-d6db-42ac-84ed-4ab45d9fc3b8

232. Paciente diagnosticado de fibrosis retroperitoneal que presenta astenia, prurito e


ictericia. En la analítica se observa elevación de la fosfatasa alcalina, GGT y
bilirrubina, con ALT normal. En la CPRE se observan estrechamientos y dilataciones
multifocales de los conductos biliares intra y extrahepáticos, y en la biopsia se objetiva
disminución de los conductos biliares normales e imágenes en piel de cebolla. El
diagnóstico de este paciente es:

1. Colangitis esclerosante primaria.


2. Cirrosis biliar primaria.
3. Síndrome de Dubin-Johnson.
4. Enfermedad de Wilson.

Resp. Correcta: 1

Comentario:

Ante la presencia de un cuadro de colestasis y una CPRE con semejantes hallazgos, el diagnóstico más
probable es el de colangitis esclerosante primaria. Esta enfermedad se asocia a colitis ulcerosa en un altísimo
porcentaje de casos, pero también puede relacionarse con otras patologías con un componente autoinmune
(observa que, en este caso, nos hablan de fibrosis retroperitoneal, con la que también se asocia). Por otra
parte, en la biopsia nos mencionan imágenes en piel de cebolla en los conductos biliares. Esto nos confirma
el diagnóstico, puesto que este hallazgo es patognomónico de la colangitis esclerosante primaria. De todas
formas, recuerda que, en general, la prueba más útil es la CPRE (aunque actualmente se prefiere la
colangioRM al evitar las complicaciones de la CPRE que es una prueba invasiva, ya que aporta información
anatómica muy precisa de la vía biliar), porque las imágenes en piel de cebolla solo se aprecian en un escaso
porcentaje de casos. Por ello, la biopsia no es práctica habitual en esta enfermedad.

-----------------------------------------------------------o--------------------------------------------------------------

Info Pregunta: 42e6d389-2e94-47af-ba5f-4b10ada5fedd

233. Un paciente de 18 años acude a urgencias por haber ingerido lejía dos horas antes.
No hay evidencia de perforación. En la esofagoscopia se observa que la mucosa
esofágica está edematosa y eritematosa. ¿Cuál de las siguientes actitudes es más
correcta?

1. Tratamiento esteroideo.
2. Exploración quirúrgica del mediastino.
3. Iniciar alimentación oral a las 48 horas.
4. Administrar alimentación parenteral durante 10 días.

Resp. Correcta: 3

Comentario:

La esofagitis por cáusticos puede aparecer tras la ingestión de ácidos o bases fuertes. La sintomatología
depende de la severidad de la esofagitis. Oscila desde el paciente asintomático hasta una intensa odinofagia,
con disfagia, hemorragia o perforación. No existe una buena correlación entre los síntomas y los hallazgos
de la exploración física con la severidad de las lesiones en esófago o estómago.

En cuanto al manejo diagnóstico, está indicado hacer una laringoscopia directa, una radiografía de tórax para
descartar perforación y, en las primeras 24 horas, una endoscopia. Respecto al manejo terapéutico, las bases
fuertes no deben ni neutralizarse ni diluirse; los ácidos fuertes tampoco deben neutralizarse, pero sí que
pueden diluirse. La mayoría de los autores recomiendan profilaxis antibiótica, pues la sobreinfección
incrementa el riesgo de estenosis. Los esteroides, en cambio, no han demostrado utilidad (ni tampoco la
ACTH, con el mismo propósito). Se recomienda iniciar la alimentación oral cuanto antes. Sólo se recurre a
la alimentación parenteral en caso de perforación esofágica o estenosis tardías severas.

En el caso de estenosis como complicación tardía, el tratamiento es la dilatación, aunque en bastantes casos
hay que llegar a la esofaguectomía. Los pacientes con esofagitis cáustica tienen un aumento de riesgo de
carcinoma epidermoide de esófago hasta 40 años después del episodio.

-----------------------------------------------------------o--------------------------------------------------------------

Info Pregunta: 29863caa-9a05-4245-8d5d-4bdf571a7057

234. Varón de 40 años diagnosticado de enfermedad de Crohn ileocólica hacía 20 años


y en tratamiento con Azatioprina de mantenimiento con buena evolución. Acude por
astenia y aumento de numero de deposiciones a mínimo 15 al día, algunas nocturnas.
Había perdido unos 6 kg. Ante esta situación el paciente inició tratamiento con
Prednisona con escasa respuesta tras 3 semanas de tratamiento. La exploración física
era anodina y la analítica mostraba: Hb 9.2 g/dl, leucocitos 18500/mm3, plaquetas
550.000/mm3, glucosa 90 mg/dl, urea 40 mg/dl, creatinina 1.2 mg/dl, GOT 10 U/L, GPT
15 U/L, GGT 16 U/L, bilirrubina total 1.2 mg/dl, fosfatasa alcalina 50 U/L, LDH 100
U/L, anti-VHA IgM -, anti-VHA IgG +, AgHBs +, antiHBs -, antiHBc +, AgHBe -,
antiHBe +, DNA-VHB indetectable, Ag-VHD -, Ac-VHC -, RNA-VHC indetectable, Ac
antiVEB IgG positivos, Ac anti VEB IgM negativos, Ac anti-CMV IgG -, Ac anti-CMV
IgM -. Los estudios microbiológicos en heces fueron negativos con disbacteriosis. En
este momento señale cual sería la actitud que tomaría en primer lugar:

1. Iniciar Rifaximina oral.


2. Iniciar Tenofovir oral.
3. Iniciar Infliximab i.v.
4. Iniciar Aciclovir oral.

Resp. Correcta: 2

Comentario:

Se trata de un paciente con una enfermedad de Crohn que presenta un nuevo brote con criterios de
corticorrefractariedad. Por ello es previsible que precise iniciar biológico. Sin embargo presenta una
hepatitis crónica B no replicativa (estadío portador). Antes de ello debe iniciarse tratamiento antiviral para
evitar una reactivación.

-----------------------------------------------------------o--------------------------------------------------------------

Info Pregunta: 5c5cd41c-6262-42af-8290-4c45b68965e1

235. Una mujer de 35 años, con ingesta de alcohol importante desde hace años, acude a
Urgencias por ictericia y malestar general. En la analítica efectuada se observa lo
siguiente: GPT 3.358 mU/ml; GOT 2.855 mU/ml, bilirrubina total 8 mg/dl. ¿Cuál de
estas afirmaciones es CORRECTA?:

1. El pronóstico de la hepatopatía es grave.


2. Probablemente sufre una hepatitis alcohólica.
3. Probablemente se trata de una ictericia obstructiva.
4. La analítica es sugestiva de necrosis hepática extensa.

Resp. Correcta: 4

Comentario:

Se trata de una paciente con elevación importante de las transaminasas, lo que traduce una intensa citolisis
(necrosis) hepática. Además, presenta ictericia, que es un hallazgo que puede asociarse en las hepatitis
agudas. Con una elevación tan intensa de las transaminasas es poco probable que se trate de una ictericia
obstructiva, además una obstrucción biliar las enzimas que suele elevar son las de colestasis (GGT y FA). La
hepatitis alcohólica suele cursar con elevación moderada de las transaminasas (menos de 300 mU/ml,
generalmente).

Recuerda que ni la cifra de transaminasas ni la cifra de bilirrubina son indicadores pronósticos en la hepatitis
aguda y lo que define la gravedad es la presencia de coagulopatía y encefalopatía (hepatitis aguda
fulminante).
-----------------------------------------------------------o--------------------------------------------------------------

Info Pregunta: 8b05517d-16a9-4c8d-9966-4c5c84b28e95

236. El síndrome portopulmonar que aparece en pacientes cirróticos NO se caracteriza


por una de las siguientes afirmaciones. Señálela:

1. Se trata de una hipertensión pulmonar como consecuencia de la hipertensión portal.


2. Presenta ortodeoxia.
3. Es una complicación grave que en ocasiones contraindica el trasplante hepático.
4. Generalmente se presenta en cirrosis avanzada.

Resp. Correcta: 2

Comentario:

El síndrome portopulmonar es una complicación generalmente en pacientes con cirrosis avanzada y que se
caracteriza por una hipertensión pulmonar por vasoconstricción y posterior fibrosis a nivel pulmonar
precapilar. En casos graves contraindica un trasplante hepático y debe diferenciarse del síndrome
hepatopulmonar, que se caracteriza por ortodeoxia, consecuencia de una redistribución vascular del riego
pulmonar.

-----------------------------------------------------------o--------------------------------------------------------------

Info Pregunta: 0c3a1108-90cc-4d88-a84e-4d6a6eca5d3f

237. Varón de 45 años que consulta por dolor abdominal epigástrico. Una endoscopia
demuestra una úlcera gástrica a nivel del antro. Las biopsias descartan malignidad. Se
inicia tratamiento con 20 mg/día de omeprazol. A las cuatro semanas se le hace un
control endoscópico que demuestra que el lecho ulceroso persiste, aunque se ha
reducido en un 75%. Se mide la gastrina sérica, que está claramente elevada. ¿Qué
haría con este paciente?

1. Aumentar omeprazol a 40 mg/día.


2. Cirugía.
3. Le haría un TAC pancreático.
4. Seguiría con el mismo tratamiento.

Resp. Correcta: 4

Comentario:

Algunos tumores gástricos pueden debutar como úlceras gástricas, por lo que es obligatorio, al hacer un
diagnóstico por imagen de un ulcus gástrico, tomar biopsias para descartar malignidad. Además, las úlceras
gástricas, en general, suelen ser más grandes que las duodenales, y por ello su tiempo necesario para la
cicatrización es mayor. Los fármacos que se emplean son los antisecretores, fundamentalmente inhibidores
de la bomba de protones, como el omeprazol. El tiempo estimado de tratamiento con omeprazol necesario
para la cicatrización de una úlcera gástrica es de unas 4-8 semanas. En este caso, parece que el tratamiento
está siendo correcto, aunque aún habría que mantenerlo 4 semanas más (respuesta 4 correcta). En cuanto a la
gastrina, es normal que se eleve mientras estamos en tratamiento con un IBP ya que, al inhibir la bomba de
protones, aumenta el pH gástrico por disminuir la secreción ácida en el estómago (hipoclorhidria) y estimula
las células G productoras de gastrina para activar a las células parietales; este es el mecanismo fisiológico de
regulación de la secreción ácida y no hay que pensar en la presencia de un gastrinoma.
-----------------------------------------------------------o--------------------------------------------------------------

Info Pregunta: 0366d5a5-d0b5-4399-988f-4d9fe1785680

238. Varón de 45 años, cirrótico VHC y fumador de 10 cigarrillos/día. Entre sus


antecedentes, destaca episodio de encefalopatía hace cuatro meses. Ingresa en
Urgencias por melenas, donde se observa una hemorragia por varices tratada con
somatostatina y esclerosis con oleato de etanolamina. A los cinco días no presenta
recidiva hemorrágica. ¿Cuál es la actitud MÁS eficaz para mejorar la evolución a
continuación?

1. Iniciar tratamiento espironolactona 100 mg/día.


2. Iniciar propranolol y nitratos.
3. Ligadura endoscópica con bandas.
4. Propranolol, nitratos y ligadura endoscópica con bandas.

Resp. Correcta: 4

Comentario:

Se trata de un paciente cirrótico VHC en el que se nos pregunta sobre la profilaxis segunda de una HDA por
varices esofágicas. En los últimos estudios se ha demostrado que lo más eficaz es la combinación de
betabloqueantes no cardioselectivos, nitratos y ligadura endoscópica con bandas hasta conseguir la
erradicación de las varices.

-----------------------------------------------------------o--------------------------------------------------------------

Info Pregunta: 14e777fa-ed02-4c1d-ad8a-4dc078f7ca17

239. La panproctocolectomía con reservorio ileal es el tratamiento de elección en:

1. Poliposis adenomatosa familiar.


2. Poliposis colónica familiar atenuada.
3. Cáncer de colon localizado en colon ascendente.
4. Enfermedad de Crohn colónica refractaria al tratamiento.

Resp. Correcta: 1

Comentario:

La panproctocolectomía consiste en la extirpación de todo el colon y el recto, por lo que debe reservarse a
pacientes que presenten patología a dicho nivel, fundamentalmente poliposis familiares y enfermedades
inflamatorias. El problema de la resección del recto se presenta en la frecuencia defecatoria, ya que actúa
como mecanismo de continencia. Para mejorar dicho síntoma se les puede ofrecer a estos pacientes, sobre
todo si son jóvenes, la creación de un reservorio, que consiste en que un asa de intestino delgado, plegada,
generalmente en J, adquiera la función de contener temporalmente las heces. Así consiguen frecuencias
defecatorias aceptables (4- 5 diurnas y 1- 2 nocturnas), mejorando su calidad de vida. Dicha opción no debe
ofrecerse en pacientes con enfermedad de Crohn, ya que implica alto riesgo de reservoritis. La poliposis
atenuada presenta pólipos en un número menor y fundamentalmente en el colon derecho, con lo que podría
ser susceptible de colectomía, preservando el recto si este presenta un número de pólipos razonablemente
controlables mediante rectoscopia.

-----------------------------------------------------------o--------------------------------------------------------------
Info Pregunta: 6a5a7246-7374-4fe6-9601-4e4ea2cc3547

240. Niño de 2 años de edad que acude al Servicio de Pediatría del Hospital por retraso
ponderoestatural, heces pastosas y malolientes, alteraciones del equilibrio y dificultad
para hablar. En un frotis de sangre se observan eritrocitos espinosos. Señale el
diagnóstico más probable de entre los siguientes:

1. Enfermedad celíaca.
2. Abetalipoproteinemia.
3. Agammaglobulinemia.
4. Infección por Tropheryma Whippelii.

Resp. Correcta: 2

Comentario:

La abetalipoproteinemia o enfermedad de Bassen y Kornzweig es un trastorno autosómico recesivo que


afecta con más frecuencia a varones, aunque también afecta a las niñas, causado por un defecto en el gen de
la proteína de transferencia de triglicéridos microsómicos (MTP). Este defecto hace que el organismo
sea incapaz de fabricar lipoproteínas (VLDL y quilomicrones) impidiendo la digestión y absorción
apropiada de grasas y vitaminas esenciales. De ello se deriva el retraso ponderoestatural (por el déficit de
nutrientes y grasas) y las heces estatorreicas. En estos niños también son frecuentes las alteraciones
neurológicas, tanto por el déficit vitamínico, como por el déficit de grasas que impide la correcta
mielinización de los axones neuronales. Un dato característico en la enfermedad, aunque no exclusivo de
ella, es la presencia de eritrocitos espinosos o acantocitos, que son eritrocitos con prolongaciones
citoplasmáticas formados como consecuencia del depósito de ácidos grasos libres en la membrana de los
mismos (ver imagen en sección inferior). Por eso la RESPUESTA 2 CORRECTA. En la
agammaglobulinemia nos hablarían de una ausencia de células plasmáticas en una biopsia intestinal
[RESPUESTA 3 INCORRECTA]. En la enfermedad de Whipple de un síndrome de malabsorción junto con
artritis no deformante (que puede preceder en años a las manifestaciones intestinales), fiebre, linfadenopatía
periférica, trastornos neurológicos, enteropatía pierde-proteínas, manifestaciones cardíacas y melanosis
cutánea [RESPUESTA 4 INCORRECTA]. La enfermedad celíaca podría dar manifestaciones similares a las
de este cuadro clínico pero no nos hablarían de eritrocitos espinosos y nos faltarían los datos de los
anticuerpos antiendomisio y antitransglutaminasa tisular [RESPUESTA 1 INCORRECTA].

-----------------------------------------------------------o--------------------------------------------------------------

Info Pregunta: 7653f2dd-5de2-4630-b7d3-4ecc403ad491

241. Una mujer de 35 años, con ingesta de alcohol importante desde hace años, acude a
Urgencias por ictericia y malestar general. En la analítica efectuada se observa lo
siguiente: GPT 3.358 mU/ml; GOT 2.855 mU/ml, bilirrubina total 8 mg/dl. ¿Cuál de
estas afirmaciones es correcta?

1. El pronóstico de la hepatopatía es grave.


2. Probablemente sufre una hepatitis alcohólica.
3. Probablemente se trata de una ictericia obstructiva.
4. La analítica es sugestiva de necrosis hepática extensa.

Resp. Correcta: 4

Comentario:

Se trata de una paciente con elevación importante de las transaminasas, lo que traduce una intensa citolisis
hepática. Además, presenta ictericia, que es un hallazgo que puede asociarse en las hepatitis agudas. La
hepatitis aguda es un cuadro auto limitado que se resuelve por lo que el pronóstico es bueno. La ecografía
abdominal es necesaria para descartar patología biliar, si bien con una elevación tan intensa de las
transaminasas es poco probable que se trate de ictericia obstructiva. La hepatitis alcohólica suele cursar con
elevación moderada de las transaminasas (menos de 300 mU/ml, generalmente).

-----------------------------------------------------------o--------------------------------------------------------------

Info Pregunta: 3ab603c0-6936-47c2-bd81-4f2bffe86f8e

242. Mujer de 42 años, trabajadora social y con diagnostico previo de una pancolitis
ulcerosa corticodependiente por lo que esta en tratamiento con mesalazina y
Azatioprina. Acude por empeoramiento del estado general, diarrea de incontables
deposiciones con sangre y fiebre de 39ºC. La exploración física mostraba una ligera
palidez de piel y mucosas y una abdomen doloroso a la palpación, especialmente en
flanco izquierdo e hipogastrio. ADVP acude por malestar general, astenia y ligera
ictericia. En este caso se instauró tratamiento con metilprednisolona i.v. pero con
escasa respuesta a los 15 días. Se plantea iniciar biológico y, por ello, se solicita
mantoux, Rx tórax, serologías de VHB, VHC y HIV. En caso de resultar alguno
positivo, señale el diagnóstico que menor se vería afectado si iniciáramos directamente
Infliximab:

1. Tuberculosis latente.
2. Hepatitis crónica B.
3. Hepatitis crónica C.
4. HIV +.

Resp. Correcta: 3

Comentario:

Aunque antes de iniciar biológico se requiere estudio de las infecciones señaladas en el caso clínico, la
hepatitis C no se ha demostrado que empeore al iniciar biológico. El HIV si tendría que estar controlado. La
hepatitis crónica B requiere obligatoriamente iniciar antivirales para evitar reactivación y la tuberculosis
latente iniciar quimioprofilaxis para evitar tuberculosis diseminada.

-----------------------------------------------------------o--------------------------------------------------------------

Info Pregunta: c2f2889c-1448-4d62-84e2-4f42b78027f2

243. Una mujer de 55 años ingresa en el hospital con el diagnóstico de pancreatitis


aguda. ¿Cuál de las siguientes pruebas o determinaciones analíticas NO es útil para
predecir la gravedad de esta enfermedad?

1. Tomografía axial computarizada (TAC) del abdomen.


2. Niveles de amilasa y lipasa en sangre.
3. Hematocrito.
4. Nitrógeno ureico en sangre (BUN).

Resp. Correcta: 2

Comentario:

Pueden usarse diversos índices de gravedad, si bien los más utilizados son los clínico-analíticos de Ransom
y APACHE II o el radiológico de Baltazar. Los valores de BUN y hematocrito forman parte de los criterios
de Ransom. El TC abdominal nos permite saber si existe necrosis asociada o no, lo que es un factor
predictivo independiente de mortalidad. Los niveles de amilasa y lipasa no son predictivos de gravedad.

-----------------------------------------------------------o--------------------------------------------------------------

Info Pregunta: 97755b9d-f229-46d7-942d-4f51f9b5d6cb

244. Mujer de 40 años consulta por diarrea crónica acuosa fluctuante, sin esteatorrea
ni sangre. La colonoscopia es macroscópicamente normal y en la biopsia se objetiva
infiltrado por linfocitos y células plasmáticas en la lámina propia e intraepitelial.
Respecto a la enfermedad que probablemente presenta esta paciente, señale la opción
FALSA:

1. En la evolución de la enfermedad de esta paciente podría observarse engrosamiento de la banda


subepitelial de colágeno.
2. Tiene un curso crónico recidivante y se han descrito casos de remisión espontánea.
3. Debe instaurarse tratamiento con dieta sin gluten.
4. El pronóstico, en general, es bueno y no hay evidencia de predisposición a cáncer.

Resp. Correcta: 3

Comentario:

Esta pregunta nos presenta un caso de colitis linfocítica/colágena. Vamos a resumir sus características
principales:
- Principal síntoma: Diarrea acuosa crónica.
- Otros: Pérdida de peso fluctuante que luego se estabiliza.
- Es más frecuente en mujeres.
- Asociación con enfermedades autoinmunes.
- Colonoscopia normal.
- Histología característica: Aumento de fibras colágenas o infiltrado linfocitario (colitis linfocitaria, en este
caso, que por lo demás son iguales).
- Tratamiento: Sintomático (antidiarreico en principio). Solo en ocasiones se utilizan AINEs y corticoides.
- No predispone a cáncer.
Lógicamente no precisa dieta sin gluten, porque no se trata de una enfermedad celíaca.

-----------------------------------------------------------o--------------------------------------------------------------

Info Pregunta: 84d20104-8942-4d03-bcce-4f53f8d393d8

245. Mujer de 22 años sin antecedentes personales de interés que consulta por ictericia
y astenia leve. La exploración física no mostraba alteraciones relevantes salvo la citada
coloración amarillenta de piel y mucosas. En la analítica se observaban los siguientes
datos: Hb 14,2 g/dl, VCM 80 fl, leucocitos 8000/mm3, plaquetas 250.000/mm3,
glucemia 98 mg/dl, urea 34 mg/dl, creatinina 1.2 mg/dl, bilirrubina 8 mg/dl (conjugada
6.3), GOT 30 U/L, GPT 42 U/L, GGT 48 U/L, fosfatasa alcalina 50 U/L, sodio 142
mEq/ml, potasio 4.1 mEq/ml. Relación con el proceso que debería sospechar, señale la
opción que considere correcta:

1. Cursa con patrón de herencia autosómico dominante.


2. Se produce por una alteración en el almacenamiento vacuolar intracitoplasmático de bilirrubina.
3. La curva de eliminación de bromosulftaleína mostrará un segundo pico a los 90 minutos
aproximadamente.
4. Los niveles de coproporfirinas séricos estarán incrementados.

Resp. Correcta: 4

Comentario:

Un paciente con hiperbilirrubinemia a expensas de fracción directa y que presenta un segundo pico a los 90
minutos en la curva de eliminación de bromosulftaleína, debe hacernos sospechar un síndrome de Dubin-
Johnson. Es un proceso que tiene una herencia autosómica recesivo se debe a una alteración del transporte
de bilirrubina de los hepatocitos al canalículo biliar. Los niveles de coproporfirinas séricos estarán normales
pero a expensas de las de tipo I (normalmente predominan las de tipo 3).

-----------------------------------------------------------o--------------------------------------------------------------

Info Pregunta: 021080a9-ce92-4d94-87a0-4f82fb7ea922

246. Varón de 49 años, bebedor de 50 g/día de alcohol y fumador de 20 cigarrillos/día,


que acude a su consulta por dolor abdominal y diarrea de tres meses de evolución. La
exploración física demostró la existencia de dolor abdominal en hemiabdomen superior
a la palpación, sin peritonismo. La radiografía simple de abdomen mostraba un patrón
gaseoso inespecífico con pequeñas calcificaciones en región central de abdomen
superior. La analítica realizada fue la siguiente: Hb 12,2 g/dL, leucocitos 7.900/mm³,
plaquetas 159.000/mm³, glucosa 103 mg/dL, urea 55 mg/dL, creatinina 0,9 mg/dL,
albúmina 5,1 g/dL, GOT 97 U/L, GPT 40 U/L, GGT 65 U/L, bilirrubina total 0,7
mg/dL, fosfatasa alcalina 180 U/L, LDH 100 U/L, amilasa 8 U/ml, sodio 140, potasio 4.
¿Cuál sería el tratamiento MÁS adecuado para la diarrea?

1. Pancreatina.
2. Loperamida.
3. Mesalazina.
4. Metronidazol.

Resp. Correcta: 1

Comentario:

Se trata de un paciente bebedor y fumador que tiene clínica y radiología simple diagnósticas de pancreatitis
crónica, por lo que la diarrea se produce por probable esteatorrea y, por tanto, el tratamiento más eficaz
consistirá en la administración de enzimas pancreáticos.

-----------------------------------------------------------o--------------------------------------------------------------
Info Pregunta: d89e9cf0-4abb-49ac-a2be-50b2a0d404d2

247. Los hepatocitos en vidrio esmerilado son CARACTERÍSTICOS de la:

1. Hepatitis crónica por VHB.


2. Hepatitis crónica por VHC.
3. Hepatitis crónica por VH delta.
4. Hepatitis por alcohol.

Resp. Correcta: 1

Comentario:

Histológicamente, la hepatitis C se caracteriza por:


- Folículos linfoides en los espacios porta.
- Daño de ductos biliares.
- Presencia de cuerpos apoptóticos.
- Esteatosis macrovesicular.
- Linfocitos en los sinusoides.

Histológicamente, la hepatitis B se caracteriza por:


- Hepatocitos en vidrio esmerilado.
- Inmunotinción positiva para el antígeno de superficie B.

Histológicamente, el daño por alcohol se caracteriza por:


- Daño celular (degeneración balonante con o sin evidencia de necrosis).
- Infiltrado inflamatorio (predominio de neutrófilos).
- Fibrosis (imagen en malla de alambre).
- Hialina de Mallory.

Las hepatitis delta produce hepatitis fibronecrótica sin datos histológicos característicos.

-----------------------------------------------------------o--------------------------------------------------------------

Info Pregunta: 89d260fc-3554-481d-9285-50dd54076e80

248. Sobre la anatomía patológica de la colitis ulcerosa, señale la FALSA:

1. Existe correlación entre el grado de actividad de la enfermedad y el de inflamación.


2. La afectación colónica se limita a la mucosa, con un aspecto muy friable.
3. La inflamación y necrosis frecuentemente profundiza y forma fístulas.
4. Con el tiempo pueden aparecer áreas de displasia que pueden evolucionar a adenocarcinoma.

Resp. Correcta: 3

Comentario: En la colitis ulcerosa, la afectación suele ser continua de las capas más superficiales (mucosa y,
en casos graves, también submucosa). Esto le confiere una gran frabilidad a la pared colónica, con
congestión vascular y riesgo de hemorragias superficiales. En la mucosa afectada, con el tiempo, pueden
aparecer áreas de displasia y evolucionar a adenocarcinoma. Sin embargo, no hay afectación transmural ni
presencia de fíatulas (característico esto último de la enfermedad de Crohn).
-----------------------------------------------------------o--------------------------------------------------------------

Info Pregunta: 4e30ba19-91ba-436a-91c9-514d1145f6ba

249. Varón de 70 años, que consulta por disfagia progresiva que se acompaña, tres
meses después de su inicio, de regurgitación alimenticia postingesta. Refiere pérdida de
10 kg de peso. Con respecto a estos datos, señale cuál de las siguientes afirmaciones NO
es correcta:

1. Ha de indicarse una endoscopia oral para descartar el cáncer de esófago.


2. Si en la endoscopia se encuentra un esófago dilatado y sin peristaltismo, está excluido el cáncer,
porque se trata de una acalasia.
3. Si en la endoscopia no se ha encontrado tumor, debe indicarse una manometría esofágica, porque
podría tratarse de un trastorno motor primario.
4. La manometría normal excluye el diagnóstico de acalasia.

Resp. Correcta: 2

Comentario:

La acalasia puede deberse a una alteración primaria del esfínter esofágico inferior (acalasia primaria), cuya
capacidad de relajación disminuye… Pero también puede obedecer a una infiltración tumoral maligna,
habitualmente por un adenocarcinoma gástrico. El estudio manométrico daría exactamente los mismos datos
y, desde el punto de vista endoscópico, en ambos casos encontraríamos un esófago dilatado y sin
peristaltismo, por lo que la respuesta falsa es la 2. Es importante revisar bien el estómago para descartar
cáncer a este nivel que simule una acalasia.

-----------------------------------------------------------o--------------------------------------------------------------

Info Pregunta: a766dee3-3051-4b17-ae04-51bc9c0486a0

250. Acude a tu consulta un paciente varón, de 31 años, remitido desde atención


primaria por presentar distensión y dolor abdominal, además de flatulencia, desde
hace tiempo. La analítica realizada al inicio del estudio muestra una anemia
ferropénica y elevación leve de las transaminasas hepáticas. Teniendo en cuenta el
diagnóstico de sospecha, ¿qué prueba diagnóstica realizaría en primer lugar?

1. Serología Ac antitransglutaminasa IgA + IgA total.


2. Estudio genético
3. Biopsia de duodeno proximal
4. Serología Ac antitransglutaminasa IgG

Resp. Correcta: 1

Comentario: Ante un paciente con alta probabilidad diagnóstica de enfermedad celíaca, como es el caso de
nuestro paciente, primero se debe realizar una serología en busca de los anticuerpos antitransglutaminasa de
tipo IgA, además de solicitar la IgA total por si hubiera un déficit de dicha inmunoglobulina que falseara los
resultados.

-----------------------------------------------------------o--------------------------------------------------------------

Info Pregunta: b9588722-de26-4f26-86cf-5237f8969513


251. Señale lo FALSO respecto al megacolon tóxico que aparece en la colitis ulcerosa:

1. Su perforación puede cursar con signos mínimos locales.


2. El tratamiento es siempre médico.
3. La hipopotasemia puede contribuir a la aparición de esta complicación.
4. Puede ser la forma de inicio de la enfermedad.

Resp. Correcta: 2

Comentario: El megacolon tóxico se trata de una complicación muy grave de la colitis ulcerosa. A veces,
hay dolor abdominal pero con escasos signos de peritonitis, lo que puede confundir al clínico. Su
diagnóstico se basa en una clínica compatible, junto con una radiografía simple de abdomen en la que se
observe una dilatación mayor de 6 cm en colon transverso. La hipopotasemia, al favorecer la hipomotilidad
intestinal, puede contribuir a su aparición. Incluso se han descrito casos de debut de colitis ulcerosa con un
megacolon tóxico. Lo que es incuestionable es que el tratamiento se basa en medidas de soporte vital, y
cirugía con intención curativa (colectomía total, según diversas técnicas quirúrgicas).

-----------------------------------------------------------o--------------------------------------------------------------

Info Pregunta: ee003283-fe7c-4189-bd52-52576f00b9ef

252. En relación al Helicobacter pylori, ¿cuál de las siguientes afirmaciones es FALSA?

1. La transmisión es por vía oral-oral o fecal-oral.


2. Es un microorganismo microaerófilo gramnegativo.
3. Está diseñado para vivir en el medio ácido del estómago.
4. El test de ureasa para el diagnóstico del H. pylori presenta una sensibilidad y especificidad inferior al
50%.

Resp. Correcta: 4

Comentario:

H. pylories un bacilo Gram negativo microaerófilo curvado que puede sobrevivir en medio ácido gracias a
que posee un enzima llamada ureasa. La ureasa fracciona la urea en dióxido de carbono y amoniaco. Este
último es un álcali que permite sobrevivir alH. pylorien medio ácido, pero es extremadamente irritante para
la mucosa gástrica y la inflama produciendo gastritis. No existe una clara evidencia acerca de cuál es la ruta
predominante de transmisión del H. pylori. El hecho que es ampliamente aceptado es que la bacteria debe
llegar al estómago humano por la boca como vía de entrada, por lo que lo más aceptado es la transmisión
oral-oral o fecal-oral. Entre los test diagnósticos se encuentra el test rápido de la ureasa. Es un test rápido y
sencillo. Se basa en la capacidad del H. pylori de producir ureasa. Se realiza con una biopsia del antro
gástrico, tomada durante la endoscopia, que se coloca en un tubo con urea y un indicador. Si la muestra
contiene ureasa, aumenta el pH y cambia el color de la solución. Pueden producirse resultados falsos
negativos si la cantidad de bacterias en el estómago es pequeña, y en casos de hemorragia digestiva. Su
sensibilidad es del 90% al 95% y la especificidad del 95% al 100%.

-----------------------------------------------------------o--------------------------------------------------------------

Info Pregunta: 452a98bd-fa1d-4301-8037-526efd3fc078

253. Varón de 55 años, alcohólico, acude traído por el SAMUR pordesorientación y


lenguaje incoherente. En la exploración física se observaba moderada ictericia y
abdomen globuloso y con signos de ascitis. Flapping positivo. La analítica de urgencias
fue: Hb 10.1 g/dl, leucocitos 3100/dl, plaquetas 52.000, actividad de protrombina 44%,
glucosa 190 mg/dl, urea 59 mg/dl, creatinina 1.1 mg/dl, GOT 160 UI/L, GPT 62 UI/L,
bilirrubina 2.5 mg/dl, GGT 1380 UI/L, amilasa 40 U/ml, proteínas totales 4.8 g/dl,
albúmina 2 g/dl, LDH 38 U/L, sodio 139 mEq/L, potasio 4.3 mEq/L. La paracentesis
mostraba un líquido con los siguientes resultados: leucocitos 1900/mm3, neutrófilos
70%, glucosa 140 mg/dl, LDH 80 U/L, albumina 0.6. El estudio toxicológico fue
negativo salvo etanol en sangre elevado. ¿Cuál sería su actitud a continuación?:

1. Realizar TC abdominal.
2. Aumentar la dosis de diurético.
3. Paracentesis evacuadora con reposición de albumina.
4. Ceftriaxona i.v.

Resp. Correcta: 4

Comentario:

Se describe un varón alcohólico que acude por encefalopatía hepática. Ellíquido presenta unos valores de
más de 250 neutrófilos/mm3, lo que debe hacernos sospechar una peritonitis bacteriana primaria. Por
consiguiente lo más adecuado sería tratar con antibiótico que cubra gram negativos con una cefalosporina de
3ª generación.

-----------------------------------------------------------o--------------------------------------------------------------

Info Pregunta: 3c426177-c535-4cf2-ad2a-528755f9ccdb

254. Varón de 54 años, consumidor habitual de alcohol, que acude a revisión de su


hepatopatía de base. Una ecografía abdominal demostró la existencia de un hígado
heterogéneo e irregular con una porta de 18 mm con flujo hepatópeto y líquido
perihepático y periesplénico. La analítica demostró los siguientes parámetros: Hb 11,2,
VCM 102 fL, leucocitos 5.500/mm³, plaquetas 85.000, actividad protrombina 40%,
urea 34 mg/dL, creatinina 1,4 mg/dL, GOT 300 UI/L, GPT 200 UI/L, GGT 890 UI/L,
bilirrubina total 2,4 mg/dL, proteínas totales 4 g/dL, albúmina 2,2 g/dL, fosfatasa
alcalina 50 U/L, LDH 100 UI/L, sodio 140 mEq/L, potasio 4,1 mEq/L. ¿Qué estadio de
Child tiene este paciente?

1. Grado A: 6 puntos.
2. Grado B: 9 puntos.
3. Grado C: 10 puntos.
4. Grado C: 15 puntos.

Resp. Correcta: 3

Comentario:

El paciente del caso clínico tiene una probable cirrosis hepática alcohólica. No relatan encefalopatía hepática
(1 punto), tiene ascitis leve (2 puntos), tiene una actividad protrombina de entre 30 y 50% (2 puntos),
bilirrubina entre 2 y 3 mg/dL (2 puntos) y albúmina < 2,8 g/dL (3 puntos), por lo que la opción de respuesta
más correcta es la 3.

-----------------------------------------------------------o--------------------------------------------------------------
Info Pregunta: 711ff3bf-2332-4a42-831c-52c89529d3fc

255. Varón de 60 años de edad sin antecedentes previos de interés, acude por prurito e
ictericia, dolor abdominal epigástrico leve. En la exploración física destacaba
únicamente la citada ictericia de piel y mucosas. La analítica presentaba Hb 13 mg/d,
plaquetas 240.000/mm3, leucocitos 11.000/mm3, glucosa 180 mg/dl, urea 40 mg/dl,
creatinina 1.2 mg/dl, bilirrubina total 8,2 mg/dl, bilirrubina directa 6.1 mg/dl, GOT 70
U/L GPT 92 U/l, GGT 60 U/l, Fosfatasa alcalina 488 U/l, amilasa 780 U/l, LDH 342 U/l,
PCR 40 mg/l, serologías de VHB y VHC negativos. ANA -, anti-LKM -, ANCA -. La
ecografía abdominal demostró dilatación de vía biliar intra y extrahepática y
engrosamiento de cabeza de páncreas. El TC abdominal objetivó la presencia de una
masa en cabeza de 2 cm y dilatación de la vía biliar intra y extrahepática. La PAAF de
la lesión demostró la existencia de un intenso infiltrado linfoplasmocitario. En relación
con la entidad que debería sospechar, cual sería el dato que menos cuadraría con su
sospecha?:

1. Hipergammaglobulinemia IgG de 2500 mg/dl.


2. IgG4 de 400 mg/dl.
3. Ca 19.9 de 7 UI/ml.
4. CEA 150 UI/ml.

Resp. Correcta: 4

Comentario: Se trata de un paciente con una masa de cabeza de páncreas, lo que podría hacer pensar nos
encontramos ante un cáncer de páncreas. Sin embargo, se trata de una pancreatitis autoinmune. Lo que nos
da la pista es una persona con una lesión nodular (pancreatitis focal) con intenso infiltrado inflamatorio. Se
caracteriza normalmente por aumento de inmunoglobulinas (especialmente la IgG4) y biomarcadores
negativos. Los niveles de CEA elevados no se relacionan con esta entidad.

-----------------------------------------------------------o--------------------------------------------------------------

Info Pregunta: b84fd7e3-cb01-42cb-971f-533fee3b0585

256. Indique la situación clínica que, en relación con la infección por virus de la
hepatitis B, presenta un paciente de 5 años procedente de Nigeria, con exploración
física normal y con la siguiente serología frente a hepatitis B: HBsAg + / ANTI-HBs - /
HBeAg - / ANTI-HBe + / ANTI-HBc IgM - / ANTI-HBc IgG + / DNA VHB +:

1. Infección aguda.
2. Infección crónica.
3. Paciente vacunado.
4. Portador asintomático.

Resp. Correcta: 2

Comentario: Pregunta directa sobre serología del VHB. La serología que nos describen es la de una hepatitis
crónica (AntiHbc IgG+ y AgHbs+) replicativa (DNA +) mutante precore (AgHbe negativo pero DNA
positivo). Recuerda que la 1 es falsa porque el AntiHbc IgM es negativo. La 3 es falsa porque el paciente no
tiene AntiHbs y además tiene una infección crónica. La 4 es falsa porque el paciente tiene DNA positivo.

-----------------------------------------------------------o--------------------------------------------------------------
Info Pregunta: c588c6f1-0980-478a-a614-534d74c70951

257. En uno de los siguientes trastornos NO se ha demostrado todavía una relación con
el H. pylori como agente causal. Señálelo:

1. Úlcera de estrés.
2. Úlcera gástrica.
3. Adenocarcinoma gástrico.
4. Gastritis antral.

Resp. Correcta: 1

Comentario: H.pylori se ha demostrado que guarda una relación muy estrecha con la aparición de gastritis
antral y atrofia gástrico, y sobre ellas pueden desarrollarse úlceras pépticas crónicas, gástricas y duodenales.
Además, se asocia a linfoma y adenocarcinoma gástrico, sobre todo tipo intestinal. No se reconoce una
asociación con la úlcera de estrés, que, a su vez, guarda relación con la hipoperfusión propia de las
situaciones de extrema gravedad, como politraumatizados, sépticos o grandes quemados.

-----------------------------------------------------------o--------------------------------------------------------------

Info Pregunta: fc97205a-4e82-49a9-a30a-538078e606d9

258. Andrés es un paciente de 43 años de edad que padece enfermedad de Crohn desde
los 16 años. Ha sido tratado con múltiples tratamientos manteniéndose durante años
con mesalacina y azatioprina. Tras sufrir hace unos meses un divorcio y comenzar a
fumar diariamente ha sufrido un brote de su enfermedad que le ha llevado a estar 1
mes ingresado en el hospital. Andrés mejora adecuadamente pero su médico digestivo
le recomienda un fármaco llamado Adalimumab. Le explica que es un nuevo fármaco
muy cómodo de usar que no ha utilizado antes pero que está teniendo muy buenos
resultados. ¿Cuál de las siguientes infecciones no es necesaria descartar antes de
someter al paciente al tratamiento de anti-TNF como el Adalimumab?

1. VIH
2. VHC
3. Streptococcus pneumoniae
4. M.Tuberculosis

Resp. Correcta: 3

Comentario:

El tratamiento con anti-TNF es cada vez más frecuente en pacientes con enfermedad de Crohn tanto en
brotes corticorresistentes como de mantenimiento. Sin embargo no debemos olvidar que antes de
comenzarlo deberemos descartar que el paciente posea VIH porque no podremos usar estos fármacos. VHB
o VHC porque deberemos tratar estas infecciones a la vez que damos el fármaco. M. Tuberculosis donde
también deberemos de tratarla. El motivo es porque el fármaco produce una depleción de linfocitos T que
provoca que estas infecciones que se encontraban latentes se reactiven o avancen con dicho déficit de la
inmunidad celular. No es necesario descartar el neumococo (respuesta correcta 3). Si no recordabas esto
podrías haber acertado la pregunta por técnica de examen con “oveja negra”. VIH,VHC y tuberculosis son
controladas por la inmunidad celular sin embargo el neumococo pertenece más a la inmunidad humoral.

-----------------------------------------------------------o--------------------------------------------------------------
Info Pregunta: c6d9c218-9759-4547-9ee7-538b932bcbdc

259. Nos encontramos ante un varón de 60 años diagnosticado de cirrosis hepática con
serología positiva para el virus C y ascitis que no responde al tratamiento con 400 mg
de espironolactona con restricción de sal (< 800 mg de sodio). ¿Cuál debe ser nuestra
actitud si queremos controlar la ascitis?

1. Estableceremos un cortocircuito portocava laterolateral.


2. Dispondremos un shunt peritoneovenoso, conscientes de las complicaciones infecciosas y de
coagulación que puede tener el enfermo.
3. Duplicaremos la dosis de espironolactona con especial atención al nivel de potasio en sangre.
4. Añadiremos al tratamiento furosemida, aumentando dosis progresivamente (desde 40 hasta 160 mg al
día como máximo).

Resp. Correcta: 4

Comentario:

En el tratamiento escalonado de la ascitis se debe iniciar con restricción de sodio. A continuación se deben
añadir diuréticos, aumentando progresivamente la dosis hasta llegar al máximo permitido. Aunque la
espironolactona es el diurético de elección, su inicio de acción es lento, y tiene potencia diurética baja; por
ello, se debe asociarse furosemida, aumentando la dosis progresivamente hasta llegar al máximo tolerado o
hasta 160 mg al día.

-----------------------------------------------------------o--------------------------------------------------------------

También podría gustarte